You are on page 1of 266

Prescribed by the National Curriculum and Textbook Board as a

Textbook for class VIII from the academic year 1998

JUNIOR SECONDARY MATHEMATICS


(Nimna Madhyamic Ganit)

[For class VIII ]

Translated by
Saleh Matin
A. S. M. Noman Alam

Edited by
Surujuddin Ahmed

National Curriculum and Textbook Board, Dhaka.

Published by

National Curriculum and Textbook Board,


69-70, Motijheel Commercial Area, Dhaka-1000

[ All rights reserved by the Publisher ]

First Edition : 1998

Reprint : 2009

Computer Compose
Ananda Printers

Cover
Md. Abdul Halim

Design
NCTB, Dhaka

For free distribution from academic year 2010 by the Government of Bangladesh

Website Version developed by : MARS Solutions Limited

Preface
Education is the key to development. A progressively improved education system
largely determines the pace and the quality of national development. To reflect the
hopes and aspirations of the people and the socio-economic and cultural reality in
the context of the post independent Bangladesh, new textbooks were introduced in
the beginning of the 1980s following the recommendations of the National
Curriculum and Textbook Committee.
In 1994, in accordance with the need for change and development, the textbooks
of lower secondary, secondary and higher secondary were revised and modified.
The textbooks from classes VI to IX were written in 1995. In 2000, almost all the
textbooks were rationally evaluated and necessary revision were made. In 2008,
the Ministry of Education formed a Task Force for Education. According to the
advice and guidance of the Task Force, the cover, spelling and information in the
textbooks were updated and corrected.
The study of arithmetic has been limited to class VI and more importance has been
given to the study of algebra, so that application and use of mathematics can be
made simple and easy. For this reason, the study of algebra has been introduced
from class six. It is hoped that students can easily solve mathematical problems
through algebraic formulas. It is necessary to develop mathematical skill in the
learners so that they can study science in a better manner. Therefore, new
techniques and methods has been presented in a simple and concrete way. As a
result, learners can themselves solve their problems without depending fully on
teachers. Mathematics is a subject where practice is needed. It cannot be
memorised. Exercises after each chapter, therefore, includes both traditional
practice questions as well as creative questions.
This book of Junior Secondary Mathematics for class VIII is the English Version
of the original textbook entitled Nimna Madhyamik Ganit written in Bangla.
We know that curriculum development is a continuous process on which
textbooks are written. Any logical and formative suggestions for improvement will
be considered with care. On the event of the golden jubilee of the Independence of
Bangladesh in 2021, we want to be a part of the ceaseless effort to build a
prosperous Bangladesh.
In spite of sincere efforts in translation, editing and printing some inadvertent
errors and omissions may be found in the book. However, our efforts to make it
more refined and impeccable will continue. Any constructive suggestion towards
its further improvement will be gratefully considered.
I thank those who have assisted us with their intellect and efforts in the writing,
editing and rational evaluation of this book. We hope that the book will be useful
for the students for whom it is written.
(Prof. Md. Mostafa Kamaluddin)
Chairman
National Curriculum and Textbook Board
Dhaka

Index
Chapter

Content

Page

Arithmetic
One

Recurring Decimal, Rational and Irrational Numbers

Two

Percentage, Profit-Loss

37

Three

Measurement and Unit

73

Four

Statistics

89

Algebra
One

Algebraic Formulas and their applications

116

Two

Algebraic Fraction

137

Three

Linear Equations and their applications

155

Four

Simultaneous Linear Equations and their applications.

173

Five

Drawing of Graphs and applications

185

Geometry
One

Parallelism of Lines, Congruency of Triangles

202

Two

Theorems related to Quadrilaterals

218

Three

Area

227

Four

Problems

239

Answers

251

JUNIOR SECONDARY MATHEMATICS

ARITHMETIC

Chapter -I
Recurring Decimal, Rational and
lrrational Numbers
1.1 Rational and Irrational Numbers
1, 2, 3, 4, --- etc. are natural numbers. The numbers which can be expressed in the
a

form of a fraction b (where, a and b are natural numbers), are called rational
a

numbers. Therefore rational number is a fraction in the form of b , where a and b


a

are natural numbers. Each whole number (integer) is a rational number : a = 1 .


The fractions which have equal value are the same rational number of different
3 6

type. For example, the fractions 4 , 8 , 12 , ------etc. represent the same rational
number.
1 4 5 6 10 8
Again, 2 , 3 , 3 , 1 , 3 , 9 , ------etc. are rational numbers. Zero, natural numbers,
proper and improper fractions are rational numbers.
a

The numbers that cannot be expressed in the form of fractions b , are all irrational
3

numbers. For example, 2 , 3 , 5 , 2 , 3 , ...... etc are irrational numbers.


There are so many irrational numbers, which are not the roots (Square root, cubic
root, .......... ) of either natural or rational numbers. All the fractions are rational
numbers. Natural numbers are all rational numbers but those are not generally
written in the form of fraction.
1.2. Classification of Decimal Fractions
Rational and irrational numbers are all real numbers. Each real number can be
expressed in the form of a decimal fraction. There are three types of decimal
factions : terminating decimals, recurring decimals and non-terminating decimals.
Terminating Decimals : Here the finite number of digits in the right side of a
decimal points. For example, 3.45, 7.893, 12.6032, .... etc. are terminating
decimals.
Recurring Decimals : Here the digits or some of the digits will occur again and
again in the right side of the decimal points.

For example, 3.333 ......., 8.4567567 ........, 6.4345674567 ......, etc. are recurring
decimals.
Non-terminating Decimals: Here the digits in the right side of a decimal point
never terminate that is the number of digits in the right side of decimal point will
not be finite or some of the digits will not occur again and again and digits in the
right side will be unlimited. For example. 3.4513681437 ...........,8.26104035
............, etc.
Terminating decimals and recurring decimals are rational numbers and nonterminating decimals are irrational numbers. The value of an irrational number can
be determined upto the required number of places after the decimal point. If the
numerator and the denominator of a fraction can be expressed in natural numbers,
then that fraction is a rational number.
Decimal fraction
In class six we have discussed about decimal fractions. Here again those are
repeated. In the formation of numbers of base ten is generally used. Here ten's
place, hundred's place, ............ etc. occur irrespectively in the left hand side of
unit's place and the place value of each digit is ten times the place value of that
digit if it occurs immediately in the right side. The digit in right side is less than
ten times that of in units place. The digits in hundred's place is ten times that of
tens,..........etc. Therefore digits of tens place is one tenth part of digit in hundred's
place digits. Units is one tenth of part ten. In the formation of many number digits
tens, hundreds,............... etc. are respectively placed in the left side of units place.
Similarly tenth, hundredth, thousandths,..............etc. are placed respectively in the
right hand side of units place.
In order to identify the digit of units place while thousandth, hundredth, tenth,
units, tens, hundreds, ...........etc. are all written together, to express a number of
point (.) is placed in the right side of units place. This point is called the decimal
point.
The portion of the number in the left side of decimal point is a whole number
(integer) and the right side is a fraction. For this reason number with a decimal
point is called decimal fraction. 94.5372 is a decimal fraction. Here 94 is a whole
number and .5372 is a fraction. This decimal fraction is read as ninety four point
five three seven two. Here 5 is tenth, 3 is hundredth, 7 is thousandth and 2 is ten
thousandth. 0.78 is a proper decimal fraction, where the whole number is zero..
Sometimes 0.78 is written as .78. But it is not better to write like this.

The value of any number does not change if any number of zeros, without a
significant digit is used after the decimal point. Again any number of zeros is used
at the left side of a number, then its value does not also change.
For example, 12 = 12.000, 734.52 = 734.5200, 25.003 = 25.00300, 12 = 00012 etc.
When one, two,.........zeros are placed at the right side of decimal point before a
significant digit, then the value will be respectively becomes hundredth,
thousandth,........... etc. of its original value. For example, .1 = One tenth of unit.
.01 = One hundredth of unit, .001 = One thousandth of unit,..... etc.
When decimal point is moved by one, two, three, ..........places to the right side,
then the value of decimal increases by 10, 100, 1000 times..........and again if it is
moved to the left side by one, two, three, ... places then the value decreases by 10,
100, 1000, ..........times.
Therefore when a decimal is multiplied by 10, 100, 1000, ............then the decimal
point is to be placed respectively at the right side after one, two,..........digits and
when it is divided by 10,100,............ the decimal point is placed respectively
before one, two, ............digits at the left side. For example,
17.4 10 = 174, 39.478 10 = 394.78, 124.6789 100 = 12467.89, etc.
Again, 78.453 10 = 7.8453, 6789.23 100 = 67.8923. etc.
Those decimals whose values are the same are called equivalent decimal
fractions. For example, 12.34, 12.340, 12.3400,...........etc. are equivalent decimal
fractions. Again, those decimals which have equal number of digits to the right
side of decimal point are called similar decimal fractions. For example, 0.452,
12.467, 8.350, ............etc. are similar decimal fractions. If necessary, by placing
required number of zeros at the right side of decimal fractions, two or more
decimal fractions can be made similar.
Conversion of Simple Fractions into Decimal Fractions
To convert a simple fraction into a decimal fraction, the numerator is to be divided
by denominator. Beside multiplying denominator by a digit and changing it into
10, 100 or 1000, ..........etc. and then multiplying the numerator by that number, the
simple fractions can be converted into decimal fractions.
Remark
After dividing numerator by denominator if there is any remainder, it will have to
be changed into tenth part. At the time of changing into tenth part, the decimal
point should be placed in the quotient and zero should be placed at the right side of
the remainder. In this way till the process of division is not completed, a zero will
have to be placed at the right side of the number that occurs in the remainder etc.

and thus the digits of the hundredth, thousandth, ............parts are obtained in the
quotient.
1.3 Conversion of Terminating Decimal Fractions into Simple Fractions
To convert a digit decimal fraction into a simple fraction, decimal point is first to
be to the extreme right side of all digits. Thus the number will be whole number. If
there is one digit at the right side of the decimal point i.e. if it is tenth part, then
after multiplying by ten, the decimal point will move at the extreme right side.
Similarly, if it is hundredth part it is to be multiplied by 100, in case of thousandth
part it is to be multiplied by 1000, according the necessity successive
multiplication is to be made by 10000, 100000,.......etc. As a result the decimal
point can be moved at the extreme right side. But value of the original decimal
fraction will remain unchanged when converted into simple fraction if the whole
number thus found is divided by the same number by which the decimal fraction is
multiplied. This number will be the denominator of the simple fraction and the
numerator of the fraction is obtained when the decimal fraction is multiplied by
this number, the whole number that in obtained by replacing the decimal point to
the extreme right side the decimal point of the decimal fraction.
For example, 11245 100 = 11245, 11245 =

11245 100
. After changing it into
100

lowest terms fraction thus formed is the fraction of 112.45.


Again. 112.45 = 112 + tenth of 4 + hundredth of 5
4
5
= 112 + 10 + 100
11200 + 40 + 5
=
100
11245
= 100
Remark
To convert a terminating decimal fraction into fractions, its denominator is
obtained by putting at the right side of 1 the number of zeros equal to the number
of digits at the right side of the decimal point and the numerator will be the whole
number that is obtained when decimal point is omitted from the decimal fraction.
In this way terminating decimal fraction are to be converted into simple fractions.
All the terminating decimals are rational numbers.

Addition and Subtraction of Terminating Decimal Fractions


In the process of addition and subtraction of decimal fractions, those are first made
into similar decimal fractions, then the same procedure like the process of addition
and subtraction of whole number is followed.
Multiplication of Terminating Decimal Fractions
Converting decimal fractions into simple fractions, numerator and denominators
are respectively multiplied by numerators and denominators and then, is to be
changed into decimal fraction, thus the product of decimal fractions is obtained.
To multiply decimal fraction of tenth part by the decimal fraction the process of
multiplication of whole number is to be applied in case of numbers ignoring
decimal point of decimal fraction and then the decimal point is placed in the
product in such a way that the product remains as decimal of (tenth tenth) or
decimal of hundredth.
To multiply decimal fraction of tenth part by the decimal fraction of hundredth
part, the product remains as decimal of (hundredth tenth) or decimal of
thousandth. Similarly multiplication of other decimal fractions are to be made.
12 38 456
For example, 12 38 = 10 10 = 100 = 456
1.2 3.8 = 4.56
Again, 3.43 1.2 = 4.116
Remark : Here in two numbers there are two digits in the right side of decimal
point of the first number and there is one digit in the right side of the decimal point
of the second number. Considering the two numbers as whole numbers,
multiplication is made and then putting decimal point after (2 + 1) or 3 digits
towards the left from the extreme right hand digit of the product, the actual
product is found.
Rule of Placing Decimal Point in the Product
In the product number of digits in the right side of decimal point = In the
multiplier number of digits in the right side of decimal point + In the
multiplication the number of digits in the right side of decimal point.
Division of Terminating Decimal Fractions : In the process of division of
decimal fractions the dividend and the divisor is first changed into simple fractions
and then the dividend fraction is divided by divisor fraction and changed into
simplest form and then by dividing numerator by denominator, the quotient will be
found.

648
36
To divide 6.48 by 3.6, it is required to divide 100 by 10 .
648 36 648 10 9
100 10 = 100 36 = 5 = 18
5 ) 9 ( 18
5
40
40
0
Alternative Method
648 10 648
648 36 =
= 36 = 18
36 10
36 ) 648 ( 18
36
288
288
0
Remark
To make divisor a whole number the decimal point is required to be towards the
right side after some digits then the decimal point in the dividend to be shifted
towards then the right side of the same number of digits. Then the quotient is
found after dividing numerator by denominator.
Terminating Decimal Numbers
If the simple fraction is converted into decimal fraction, it will be a terminating
decimal fraction if and only if denominator of simple fraction has no prime factors
other than 2 and 5. In the process of division for converting simple fraction into
decimal no remainder will remain at a stage and therefore a definite number of
digits after the decimal point in the quotient will exist.
Example 1. Express the following fractions into decimal fractions:
41
627 24 9
,
,
and
32.
100 5 20
627
Solution : 100 = 6.27

24 24 2 48
5 = 52 = 10 = 4.8
95
45
9
20 = 20 5 = 100 = 0.45
32) 41(1.28125
32
90
64
260
256
40
32
80
64
160
160
0
41
32 = 1.28125
Ans: 6.27, 4.8, 0.45, 1.28125
Example 2. Express the decimal fractions into simple fractions :
3.57. 12.045. 9.825 and 0.05
3.57100 357
57
Solution: 3.57 = 100 = 100 = 3 100
12.045 =
9.825 =

12.045 1000 12045 2409


9
=
=
=
12
1000
1000
200
200

9.825 1000 9825 393


33
=
=
=
9
1000
1000 40
40

0.05 100
5
1
=
=
100
100 20
57
9
33
1
Ans : 3100, 12 200 , 9 40 and 20
0.05 =

N.M.G. -3

Example 3. Find out the sum of 42.304 + 23.4 + 15.02 + 121.657 + 12


Solution : 42.304
23.400
15.020
121.657
12.000
214.381 (All decimals are made similar and then added)
Ans: 2l4.38l
Example 4.

(a) Subtract 87.7456 from 123.43


(b) Subtract 298.45 from 523.7083

Solution :
(a)
123.4300
87.7456
35.6844 (Decimals are made similar and then subtracted)
Ans 35.6844
(b)

523.7083
298.4500
225.2583 (Decimals are made similar and then subtracted)

Ans : 225.2583
Example 5. (a) Multiply 43.28 by 22.674
(b) Multiply 123.35 by 2.568
(c) Multiply 3.674 by 0.26
(d) Multiply 0.4325 by 0.16
Solution:
(a)

4328 22674 98133072


43.28 22.674 = 100 1000 = 100000 = 981.33072

10

Alternative method :
43.28
22.674
17312
302960
2596800
8656000
86560000
981.33072
Ans: 981.33072.
(Here there are 2 digits in the right side of decimal point of multiplicand and 3
digits in the right side of decimal point of multiplier. The decimal point is to be
placed in the left side of (2 + 3) or 5 digits of the product).
(b)
12335 2568 = 31676280.
123.35 2.568 = 316.76280. (Here the decimal point is placed at the left side of
(2 + 3) or 5)
123.35 2.568 = 316.7628 (In decimal fractions '0' in the extreme right side is
omitted)
Ans: 316.7628.
(c) 3674 26 = 95524
3.6740.26 = 0.95524. (Here the decimal point is placed at the left side of (3+2)
or 5 digits)
Ans: 0.95524
(d) 4325 16 = 69200

0.4325 0.16 = 0.069200 [Here the decimal point is to be placed at the left side
of (4 + 2) or 6 digits. But the product consists of 5 digits and so the product has
been made of 6 digits by placing 0 at the extreme left and then decimal point has
been placed.]
Again 0.069200 = 0.0692
0.4325 0.16 = 0.0692.
Ans : 0.0692

11

Example 6. (a) Divide 42.64 by 3.2


(b) Divide 1.656 by 0.36
(c) Divide 1209.11 by 4.025
(d) Divide 0.0005 by 0.008
42.64 42.64 10 426.4
Solution : (a) 42.64 3.2 = 3.2 =
= 32
3.2 10
32) 426.4(13.325
32
106
96
104
96
80
64
160
160
0
Ans: 13.325
1.656 1.656 100 165.6
(b)
1.656 0.36 = 0.36 =
= 36
0.36 100
36) 165.6(4.6
144
216
216
0
Ans : 4.6
1209.11 1209.11 1000 1209110
(c) 1209.11 4.025 = 4.025 =
= 4025
4.025 1000
4025)1209110(300.4
12075
16100
16100
0
Ans: 300.4

12

(d) 0.0005 0.008 =


=

0.5 10
= ,
8 10
80)500(0.0625
480
200
160
400
400
0

0.0005 1000 0.5


= 8
0.008 1000
0.0005 10000 5
[
=
]
0.008 10000 80
or,

8)0.50( 0.0625
48
20
16
40
40
0

Ans : 0.0625
Remark : At present in our country terminating decimals are used for currency, weight,
measurement of length and area, etc. Therefore, it is essential to acquire knowledge about
addition, subtraction, multiplication and division of terminating decimals.
1.4 Recurring Decimal Fractions
Some times it is found that, when a numerator of a fraction is divided by its denominator
to convert it into decimal fraction the process of division is never terminated. In such
cases the same number appears again and again or some numbers repeat alternately in the
quotient.
When in the right side of the decimal point of decimal fractions a digit appears, again and
again or more than one digit repeat successively, then these are called recurring decimal
fractions.
In recurring decimal fractions, the portion which occurs again and again is called
recurring part.

3.3333 ------,10.23456456 ------, etc. are recurring decimal fractions. In the first
recurring decimal 3 is recurring part and in the second decimal 456 is the recurring
part. To indicate presence of the digit (s) of the recurring part a symbol (.) once
again recurring point is used upon the digit (s).


For example, 3.333 ------- = 3. 3 and 10.23456456 ....... = 10.23456
Remark : When only one digit recurrs, recurring point is placed upon that digit.
When more than one digit recurrs, in that case the recurring points are placed upto
the first and the last digit.

13

In recurring decimal fractions, if after decimal point there is no other digit except
recurring one then it is called pure recurring decimal and in recurring decimal
fractions, if there are one digit or more than one digit after decimal point in
addition to recurring one, then it is called mixed recurring decimal. For example,

5. 3 is pure recurring decimal and 8.23512 is mixed recurring decimal. If there


exists prime factors other than 2 and 5 in the denominator of the fraction, the
numerator will not fully be divisible by denominator. As the last digit of
successive divisions are 1, 2, ........ , 9 and nothing else, so at one stage the same
number will repeat in the remainder. The number in the recurring part is always
smaller than that of the denominator.
1
Example 7. Express 3 into decimal fraction.
Solution: 3)1.0( 0.333..............
9
10
9
10
9
1
Here in the quotient the number after the decimal points is 3 and the remainder is
1. Now if the process of division is continued in each time 3 will occur in the
quotient and the remainder will be 1. The process of division will never be
completed.

The required decimal fraction = 0.333 ............= 0. 3

Ans : 0. 3
1
Example 8. Express 13 into decimal fraction.

14

Solution:

13) 1.00(0.07692307........
91__
90
78___
120
117__
30
26__
40
39__
100
91__
9

The required decimal fraction = 0.07692307..........= 0. 076923

Ans : 0. 076923
101
Example 9. Express 26 into decimal fraction.
Solution:
26) 101 (3.88461538461
78_
230
208__
220
208__
120
104__
160
156__
40
26__
140
130__
100
78__
220
208__
120
104__
160
____156__
40
26__
14

15

The required decimal fraction = 3.8846153

Ans : 3.8846153
Conversion of Recurring Decimal into Simple Fraction

Example 10. Express 0. 3 into simple fraction.

Solution : 0.3 = 0.3333 .................

0. 3 10 = 0.333........ 10 = 3.333 .................

and 0. 3 1 = 0.333......... 1 = 0.333 .................

By subtracting, 0.3 10 0. 3 1 = 3 or, 0. 3 (10 1) = 3 or, 0. 3 9 = 3


3 1
Therefore, 0.3 = 9 = 3
1
Ans : 3

Example 11. Express 0. 24 into simple fraction.

Solution : 0.24 = 0.24242424 ..............

Therefore, 0.24 100 = 0.242424 .......... 100 = 24.2424...............

and
0.24 1 = 0.242424 ........... 1 = 0.242424 ...............

By subtracting, 0. 24 99
24 8
0.24 = 99 = 33
8
Ans. 33

Example 12. Express 5.1 345 into simple fraction.

Solution: 5.1 345 = 5.1345345345..........

Therefore, 5.1 345 10000 = 5.1345345........... 10000 = 51345.345...........

and
5.1 345 10
= 5.1345345 .......... 10
= 51.345.................

16


By subtracting, 5.1 345 9990 = 51294
8549
224
51294
5.1 345 = 9990 = 1665 = 5 1665
224
Ans : 5 1665

Example 13 (a). Express 42.34 78 into simple fraction.

Solution : 42.3478 = 42.34787878 ............................

Therefore, 42.3478 10000 = 42.347878........ 10000 = 423478.7878 .......

and
42.3478 100 = 42.347878 ....... 100 = 4234.7878...........

By subtracting, 42.3478 9900 = 419244


287
419244 34937
42.3478 = 9900 = 825 = 42 825
287
Ans: 42 825
Explanation : From the examples given above, it appears that,
* The recurring decimal has been multiplied by the number formed by putting at
the right side of 1 the number of zeros equal to the number of digit in the right
side of decimal point of the recurring decimal.
* The recurring decimal has been multiplied by the number formed by putting at
the right side of 1 the number of zeros equal to the number of digits which are
nonrecurring after decimal point of the recurring decimal.
* Second product has been subtracted from the first product. By subtracting
second product from first product the whole number has been obtained at the
right side. Here it is observed that, the number of non recurring part has been
subtracted from the number obtained by removing the decimal and recurring
points of recurring decimal fraction.
* The result of subtraction that is obtained above which is a whole number has
been divided by the number formed by writing the same number of 9 equal to
the number of digits of recurring part and number of zeros equal to the number
of digits of nonrecurring part (all 9's at the left and all 0's at the right has been
written in the formation of number).
N.M.G. -4

17

* The denominator of the fraction that is obtained consists of the number of 9


which is equal to the number of digits in the recurring part and number of zeros
equal to the number of digits in the nonrecurring part, all these zeros are placed
in the right side of all 9's. And the numerator of the fraction in the result that is
obtained by subtracting the number of the digits formed by omitting the digits
of recurring part from the number formed by removing the decimal and
recurring points of recurring decimal.
Remark : Any recurring decimal can also be converted into a simple fraction. All
recurring decimals ate rational numbers.
. .
Example 13 (b). Express 5.234 57 into simple fraction.
. .
Solution : 5.234 57 = 5.23457457457 ..............
. .
Therefore, 5.234 57 100000 = 523457.457457
. .
and
5.234 57 100
=
523.457457
. .
By subtracting, 5.234 57 99900 = 522934
. . 522934 261467
5.234 57 = 99900 = 49950
261467
Ans : 49950
Explanation : In the decimal part there are 5 digits, recurring decimal has been
multiplied first by 100000 (5 zeros at the right side of 1). As there are two digits at
the left side of recurring part in the decimal portion, so the recurring decimal has
been multiplied by 100 (two zeros at the right side of 1). Second product has been
subtracted from the first product. In one side of the result of subtraction is a whole
number and at the other side the result is (100000100) = 99900 times the value of
the given recurring decimal. Dividing both sides by 99900, the required fraction is
obtained.
Rule of Transfomation of Rucurring Decimals into Simple Fractions
Numerator of the required fraction = The result obtained by subtracting the
number formed by omitting the digits of recurring part and decimal point of the
given recurring decimal form. The number found by removing decimal point of
the given recurring decimal.

18

Denominator of the requried fraction = Number formed by putting the number of 9


equal to the number of digits in the recurring part of given recurring decimal at the
left and the number of zeros equal to the number of digits in the nonrecurring part.
Now the above rule is directly applied to express some recurring decimals into
simple fractions.

Example 14. Express 45.2346 into simple fraction.
1172
452346 452 451894 225947
Solution : 45.2346 =
=
=
=
45
9990
9990
4995
4995
1172
Ans : 45 4995

Example 15. Express 32. 567 into simple fraction.
21
32567 32 32535 3615 1205
Solution : 32.567 =
=
=
=
=
32
999
999
111
37
37
21
Ans : 32 37

Example 16. Express 0.634 into simple fraction.


634 6 628 314
Solution : 0.634 = 990 = 990 = 495
314
Ans: 495

Example. 17. Express 2.637 into simple fraction.


287
2637 263 2374 1187
Solution : 2.637 =
= 900 = 450 = 2 450
900
287
Ans : 2 450

Example 18. Express 1.5326 into simple fraction.


5273
15326 153 15173
Solution : 1.5326 =
=
=
1
9900
9900
9900
5273
Ans : 1 9900

19

Similar Recurring Decimals


If the number of digits in the nonrecurring part of recurring decimals are equal and
also the numbers of digits in the recurring part are equal, then those are called
similar recurring decimals. Other recurring decimals are called non-similar

recurring decimals. For example, 0.3456 and 12.6793 ; 12.45 and 6.32; 9.453 and

125.897 are similar recurring decimals.




Again, 0.3456 and 7.45789 ; 6.4357 and 2.89345 are non-similar recurring
decimals.
The Rule of Changing Non-Similar Recurring Decimals into Similar
Recurring Decimals:
The value of the recurring decimal is not changed, if the digits of its recurring part
are written again and again. For example:

6.4537 = 6453737 = 645373 = 6453737. Here each one is recurring decimal,


6.45373737....... is non-terminating decimal.
It will be seen that each recurring decimal if converted into a simple fraction has
the same value.
64537 645 6453737 645 645373 6453 6453737 64537
=
=
=
9900
999900
99000
990000
63892 6453092 638920 6389200
or, 9900 = 999900 = 99000 = 990000
Multiplying by 101 the numerator and denominator of the frist fraction, the
numerator and denominator of the second fraction is obtained.
To get the necessary numbers of digits of the recurring and nonrecurring parts, the
recurring point can be transfered in such a way that the value of the recurring
decimal is not changed. In order to make the recurring decimals similar, number of
digits in the nonrecurring part of each recurring decimal is to be made equal to the
number of digits of nonrecurring part of that recurring decimal in which greatest
number of digits in the nonrecurring part exists and the number of digits in the
recurring part of each recurring decimal is also to be made equal to the lowest
common multiple of the numbers of digits of recurring part of recurring decimals.


Example 19. Convert 5. 6, 7.345 and 10.78423 into similar recurring decimals.

20


Solution : The numbers of digits of nonrecurring parts of 5. 6, 7.345 and 10.78423
are respectively 0,1 and 2. Here the number of digits in the nonrecurring part

occurs in 10.78423 and that number is 2.
Therefore to make the recurring decimals similar the number of digits in the
nonrecurring part of each recurring decimal is to be made 2. Again the numbers of


digits to recurring part of 5. 6, 7.345 and 10.78423 are respectively 1,2 and 3.
The lowest common multiple of 1,2 and 3 is 6. So the number of digits in the
recurring part of each recurring decimal would be 6 in order to make them similar.

So, 5. 6 = 5.66666666, 7.345 = 7.34545454,


10.78423 = 10.78423423.

Ans : 5.66666666, 7.34545454, 10.78423423.


Remark : In order to make the terminating fractions similar the required ; number
of zeros is placed after the digits at the extreme right of decimal point of each
decimal fraction and thus the number of nonrecurring part in each decimal after
the decimal point has been made equal. Again in recurring decimals the numbers
of digits or nonrecurring part of decimals after the decimal points are equal and the
numbers of digits of recurring parts made equal by using the recurring digits.
Beginning from any digit after all the digits of nonrecurring part the recurring part
can be taken.
Addition and Subtraction of Recurring Decimals
In the process of addition or subtraction of recurring decimals, the recurring
decimals are to be made similar. Then the process of addition or subtraction as that
of terminating decimals are followed. If addition or subtraction of terminating
decimals and recurring decimal together are done, then in order to make recurring
decimals similar the number of digits of nonrecurring part of each recurring
decimals should be equal to the number of digits between the numbers of digits
after the decimal points of terminating decimals and that of the nonrecurring parts
of recurring decimals. The number of digits of recurring part of each recurring
decimal will be equal to L. C. M. as obtain by applying the rule stated earlier and
in case of terminating decimals, necessary numbers of zeros are to be used in its
recurring parts. Then the same process of addition and subtraction as done in case
of terminating decimals are to be applied. The sum or the difference obtained in

21

this way will not be actual one. It should be observed that in the process of
addition of similar decimals if any number is to be earring over after adding the
digits at the extreme left of the recurring part of the decimals then that number is
added to the sum obtained and thus the actual sum is found. In case of subtraction
the number to be carried over is to subtract from the difference obtained and thus
actual result is found. The sum or difference which is found in this way is the
required sum or difference.
Remark : (a) The sum or difference of recurring decimals is also a recurring
decimal. In this sum or difference of the number of digits in the nonrecurring part
will be equal to the number of digits in the nonrecurring part of that recurring
decimal which have the highest number of digits in its nonrecurring part.
Similarly, the number of digits in the recurring part of the sum or the result of
subtraction will be the equal to L.C.M. of the numbers of digits of recurring parts
of recurring decimals. If there is any terminating decimal, the number of digits in
the nonrecurring part of each recurring decimal will be equal to the highest
numbers of digits that occurs either in the nonrecurring parts of recurring decimals
or in the terminating decimals after the decimal point.
(b) Converting the recurring decimals into simple fractions, addition and
subtraction may be done according to the rule as used in case of simple fractions
and then the sum or difference is converted into decimal fractions. But this process
needs more time.


Example 20. Convert 1.7643, 3. 24 and 2.78346 into similar recurring decimals.
Solution : In 1.7643 the number of digits in the nonrecurring part means 4 digits
after decimal point and here there is no recurring part.

In 3.24 the number of digits in the recurring and nonrecurring parts are
respectively 0 and 2.

In 2.78346 the number of digits in the recurring and nonrecurring parts are
respectively 2 and 3.
The highest number of digits in the nonrecurring parts is 4 and the L.C. M. of the
numbers of digits in the recurring parts i. e. 2 and 3 is 6. The numbers of digits in
the recurring and nonrecurring parts of each decimal will be respectively 4 and 6.

1.7643 = 1.7643000000; 3.24 = 3.2424242424; 2.78346 = 2.7834634634.

Ans: 1.7643000000, 3.2424242424 and 2.7834634634.

22


Example 21. Add : 3. 89, 2.178 and 5.89798.
Solution : Here the number of digits in the nonrecurring part will be 2 and the
number of digits in the recurring part will be 6 which is L. C. M. of 2, 2 and 3.
At first three recurring decimals are made similar.

3. 89
=
3.89898989

2.178
=
2.17878787

5.89798
=
5.89798798
11.97576574
+2

11.97576576
[ 8 + 8 + 7 + 2 = 25. Here 2 is the number to be carried over. 2 of 25 has been
added].


The required sum = 11.97576576 or, 11.97576.

Ans 11.97576.
Remark : In the sum the number in the recurring part is 576576. But the value is
not changed if 576 is taken as the number of recurring part.
Note : To make clear the concept of adding 2 at the extreme right side this
addition is done in another method:

3. 89
= 3.89898989|89

2.178
= 2.17878787|87

5.89798
= 5.89798798|79

11.97576576|55
Here the number is extended upto 2 more digits after the completion of recurring
part. The additional digits are separated by drawing a vertical line. Then it is added
2 has been carried over from the sum of the digits at the right side of the vertical
line and this 2 is added to the sum of the digit at the left side of the vertical line.
The digit in the right side of the vertical line is the same as the digit from which
recurring point begins. Therefore both the sums are the same.

23

Example 22. Add : 8.9478, 2.346 and 4.71


Solution : To make the decimals similar, the number of digits of nonrecurring
parts would be 3 and that of recurring parts would be 6 which is L. C. M. of 3 and 2.

8.9478
=
8.947847847

2.346
=
2.346000000

4.71
=
4.717171717
16.011019564
+1

16. 011019565
[8 + 0 + 1 + 1 =10. Here the digit in the second place on the left is 1 which is to be
carried over. Therefore 1 of 10 is added.]

The required sum = 16. 011019565.

Ans: 16. 011019565.


Example 23. Subtract 5.24673 from 8.243.


Solution : Here the number of digits in the nonrecurring part would be 2 and that
of recurring part is 6 which is L.C.M of 2 and 3. Now making two decimal
numbers similar subtraction is done.

8.243
= 8.24343434

5.24673
= 5.24673673
2.99669761 [ Subtracting 6 from 3,1 is to be carried over.]
1

2.99669760

Ans : 2.99669760
Remark : If the digit at the beginning place of recurring point in the number from
which deduction to be made is smaller than that of the digit in the number which is
to be deducted then 1 is to be subtracted from the extreme right hand digit of the
result of subtraction.

24

Note : In order to make the conception clear why 1 is subtracted, subtraction is


done in another method as shown below :

8.243
=
8.24343434|34

5.24673
=
5.24673673|67

2.99669760|67
Here both the difference are the same.


Example 24. Subtract 16. 437 from 24.45645.


Solution : 24.45645 = 24.45645


16. 437 = 1643743
801902 [7 is subtracted from 6,1 is to be carried over. ]
1

801901

or, 8019

Ans: 8019.


Note : 2445645
=
2445645/64


16437
=
1643743/74


801901/90 or, 8019
Example 25.

(a) Subtract 10418 from 1312784

(b) Subtract 82438 from 24435
Solution :


(a)
1312784
=
13127847

10418
=
10418000

2709847

Ans : 2709847
N.M.G. -5

25

(b)

24435

82438


Ans : 16191156.


= 24435000

= 8243 843
16191157
1

16191156

Multiplication and Division of Recurring Decimals: Converting recurring


decimals into simple fraction and completing the process of their multiplication or
division, the simple fraction thus obtained when expressed into a decimal fraction
will be the product or quotient of the given recurring decimals. In the process of
multiplication or division amongst terminating and recurring decimals the same
method is to be applied.
But in the case of division, if both the divisor and dividend are recurring decimals
and those are made similar, then the process of division can easily be completed.
Remark : The direct method similar to addition or subtraction of recurring
decimals is not used for multiplication or division.

Example 26. Multiply 43 by 57


43 4 39 13
Solution : 43 = 9 = 9 = 3
57 5 52
57 = 9 = 9

13 52 676

43 57 = 3 9 = 27 = 25037


Ans : 25 037

Example : 27. Multiply 0.28 by 4218


28 2 26 13
Solution : 0.28 = 90 = 90 = 45

26

4218 42 4176 464


4218 =
= 99 = 11
99

13 464 6032
The required product = 45 11 = 495 = 12185

Ans: 12185.

Example 28. Multiply 146 by l218


146 14 132 44
Solution : 146 =
= 9 = 3
9
1218 12 1206 67
1218 = 990
= 990 = 55
4

44
67
268
The required product = 3 55 = 15 = 1786
5

Ans : 17 86

Example 29. 25 435 1234 = how much?


25 5
Solution : 25 = 10 = 2
435 43 392
435 = 90
= 90
1234 12 1222 611
1234 = 990
= 990 = 495
1
196
5
392 611 196 611 119756
The required product = 2 90
495 = 8910
= 8910
1
18
= 13.44062---Ans: 1344062

Example 30. Divide 732 by 027

27

732 7 725
Solution : 732 = 99 = 99
27 2 25
5
027 = 90 = 90 = 18
145 2

725

5
725
18 290
732 027 = 99 18 = 99
5 = 11 = 26. 36
11
1

Ans : 26 36

Example 31. Divide 22718 by 1912


22718 2 22716
1912 19 1893
Solution : 22718 = 9999 = 9999 , 1912 = 990
= 990
12
10

22716 1893

22716
990 120
22718 1912 = 9999 990 = 9999
1893 = 101 = 11881
101
1

Ans: 11881.

Example 32. Divide 945 by 2 863


189
99
945 2863 28 945
990
Solution : 945 2863 = 100
= 100
2835
990
10
2
1
33
189 99
33
=
= 10 = 33
2 2835
945
5
Ans: 33
Example 33. Simplify :

35 1602
42 314 22

of
+

22 5972
42 + 314 15

28

Solution : Given Expression :


42 4 314 31
22 2
35
1602 1

90
9
9
10
999
= 42 4

of

314 31
15 1
22 2
5972 5
90
9 +
9
9
999
38 283
1601
10
7
1

20 9 35 9 999
9 90
= 38 283 9 14 of 10 20 + 5967
1
7

4
9 + 90
999
380 283
1
9
1

90
10
63 1601 999
= 380 + 283 7 of 40 + 999 5967
1
4 1

90
1

90
9 1601
97
4 1601
97

= 90 663 4 + 5967 = 663 9 + 5967


1

388
1601 1989
1
= 5967 + 5967 = 5967 = 3 = 03
3

Ans : 03
Remark: The product of recurring decimals, may be a recurring decimal or not.

For example, 13 299 = 4. The quotient of recurring decimals may also be a


recurring decimal or not.
1.5. Non Terminating Decimals
There are so many decimal fractions in which the number of digits after its
decimal point is un-limited, again one or more than one digit does not come
successively again and again. Such decimal fractions are called non terminating
decimal fractions. For example,
5134248513942307................. is a non terminating decimal number. The square
root of 2 is a non terminating decimal.
Now if we want to find out the square root of 2 then it will be seen that the number
of digits after the decimal point never ends.

29

1) 2 (14142135 .....................
1
100
24 96
281

400
281

11900
2824 11296
60400
28282 56564
383600
282841 282841
10075900
2828423 8485269
159063100
28284265 141421325
17641775
If the above process is continued for ever then it will never ends.
The value upto the definite number of decimal places/approximate value upto
some decimal places
It is required to find out the value of such non terminating decimals upto definite
number of decimal place. Sometimes it is asked to find out the approximate value
upto a definite number of decimal places. But the meaning of both is not the same.
The value of the decimal, 5.4325893 ------- upto four decimal places will be
5.4325, but the approximate value of the decimal, 5.4325893-------upto four
decimal places will be 5.4326.
Here the value upto 2 decimal places and the approximate value upto 2
decimal places are the same. This value is 5.43. In this way the approximate
terminating decimals can also be found out.
Remark : When it is wanted to find out the value upto some decimal places, then
the digits that occur in those places are to be written without any alteration of
those digits.

30

Example 34. Find out the square root of 13 and write down the approximate
value upto 3 decimal places.
Solution :
3)13(3.605551---------9
400
66 396
40000
7205 36025
397500
72105 360525
3697500
721105 3605525
9197500
7211101 7211101
1986399
The required square root = 3.605551 ............
The required approximate value upto 3 decimal places = 3.606.
Ans: 3605551................, 3606.
Remark : The square roots of those numbers which are not square numbers,
i.e. which are not 1,4, 9, 16, 25, ------- or 12, 22, 32, 42, 52-------, these square roots
are non terminating decimals, i.e. the numbers are irrational.
Example 35. Find out the value and approximate value of 44623845 .............
upto 1, 2, 3, 4 and 5 decimal places.
Solution : The value of 44623845 upto 1 decimal place is
44
and approximate value
" 1
"
" "
45
Value upto 2 decimal place is
446
and approximate value upto
2
"
" "
446
Value upto 3 decimal place is
4462
and approximate value upto
3
"
" "
4462
Value upto 4 decimal place is
44623
and approximate value upto
4
"
" "
44624
Value upto 5 decimal place is
446238
and approximate value upto
5
"
" "
446238

31
i

Example 36. Which numbers are rational and which are irrational of the following

13
(a) 325 (b) 5 (c) 7 (d) 324 (e) 15 (f) 25 (g) 14
3
4
165
5
2
(h) 0
(i) 5 (j) 2
(k) 17
(l)
18
6
Solution :
325
5
(a) 325 = 100 , rational (b) 5 = 1 , rational (c) 7 , irrational, since 7 is not a
square number.
324 32 292
13
(d) 324 = 90
= 90 , rational, (e) 15 , rational (f) 25 = 5, rational.
0
(g) 14 = 2 7 , irrational. (h) 0 = Any number except zero , rational
3
4 3 5 3 6
9
1
5
(i) 5 = 4 6 = 4 5 = 10 , rational. (j) 2 = 5 2 , irrational [... The product
6
is irrational, when the irrational number multiplied by rational number.]
165
1
2
2
(k) 17 , rational (1)
=
= 3 , rational.
18
2 9
Remark : It is noted that, multiplication or division of two irrational number may
be rational.

1.

2.

EXERCISE 1
Express the following fractions into decimal fractions:
1
3
8
3
7
7
9
1216
87
(i) 2 (ii) 4 (iii) 5 (iv) 116 (v) 10 (vi) 220 (vii) 232 (viii) 1280 (ix) 3100
Express the following decimal fractions into simple fractions:
(i) 03
(ii) 025
(iii) 125
(iv) 347
(v) 2125
(vi) 2025 (vii) 3675 (viii) 10005 (ix) 122255

32

3.

4.

5.

6.

7.

8.

9.

Find out the sum of the following decimal fractions :


(i) 012, 302, 28 (ii) 13, 312, 5023, 3 (iii) 423, 6345, 88792, 63458
(iv) 0005, 500, 5505 (v) 1 + 11 + 1111 + 1001101
(vi) 00507 + 0507 + 507 + 50007 + 5 (vii) 7 + 0873 + 001 + 00837
Subtract the following decimal fractions:
(i) 5321 2109 (ii) 601 42301 (iii) 120145 893
(iv) 600004 0678523 (v) 0001 0000635 (vi) 20345 124936
Simplify:
(i) 1532 4689 + 15639 4324 + 6342
(ii) 02 3567 + 632 16324 00034 13025
(iii) 400001 12001 50001 16456
(iv) 26003 3423 + 55 + 3002 5732
Find out the product :
(i) 432 6 (ii) 675 12 (iii) 4005 200 (iv) 08 08 (v) 52 08
(vi) 40 37 (vii) 724 56 (viii) 832 005 (ix) 0006 0025
(x) 34025 128 (xi) 00306 2402 (xii) 05 05 05
(xiii) 19 101 1001 (xiv) 002 005 004 (xv) 02 002 0002 20
Find out the quotient:
(i) 1225 5 (ii) 432.5 25 (iii) 12096 112 (iv) 28426 233
(v) 245 54635 (vi) 000281 1405 (vii) 00006 0005
(viii) 4 0625 (ix) 84375 00375
Simplify:
i. (345 + 26) 634 + 265 125 10345
ii. (627 05) 05 of 075 836
iii. 1175 0116 of (175 35) { 01 of (120 20) 055}
iv. 24 05 of 05 375 + 376 016 of 5
Express into recurring decimal fractions :
1
2
1
5
7
5
(a) 6
(b) 3
(c) 9
(d) 9
(e) 11
(f) 16
2
11
1
8
17
647
(g) 39
(h) 412
(i) 15
(j) 315
(k) 18
(l) 1980

N.M.G. -6

33

10. Express into simple fractions :

(a) 0 2
(b) 035
(c) 0456
(d) 013
(e) 142 (f) 378

(g) 5243 (h) 8. 893 (i) 4032


(j) 62304 (k) 4.678
11. Express into similar recurring decimal fractions :
.
.
. .

(a) 1.3, 3.3 2 (b) 2. 3, 5.235 (c) 7.26, 4.23 7 (d) 5. 7, 8.3 4, 6.245

(e) 3.23, 9.2348, 1.2576 (f) 12.32, 2.19, 4.3256


12. Add:

(a) 02 + 03 (b) 0 7 + 0 8 (c) 012 + 0243 (d) 045 + 0134


.


(e) 8 + 234 + 7238 (f) 205 + 804 + 7018 (g) 2245 + 589 + 0341


(h) 0006 + 092 + 00134 (i) 27 + 0 395 + 512746
13. Subtract:

(a) 34 213
(b) 5.12 345
(c) 18.325 956

(d) 6432 275346


(e) 15 12345
(f) 849 5356


(g) 19345 132349
(h) 71607 5234 (i) 9076 4234
.

(j) 485 14693


(k) 38972 00089
14. Multiply:
.

(a) 03 06 (b) 113 26 (c) 24 081 (d) 718 403 (e) 0 09 073
.

(f) 075 16 (g) 062 03 (h) 4218 028 (i) 12 112 0081
15. Divide :

(a) 06 09 (b) 03 06 (c) 24 004 (d) 035 17 (e) 237 045

(f) 0732 0027 (g) 12185 4218 (h) 03 075 (i) 55851 96

(j) 06 009 (k) 016 13 (l) 1185 024

34

16. Find out the square root (upto four decimal places) and write down the
approximate values of the square roots upto three decimal places.

(a) 12
(b) 125
(c) 05
(d) 025
(e) 134



(f) 7036
(g) 143456
(h) 7
(i) 3437
(j) 51302
17. Write down which are rational and irrational of the following numbers :

7
6
8
(a) 04 (b) 9 (c) 135 (d) 11 (e) 13 (f) 3 (g)
7
2


3
126
27
(h)
(i) 3
(j) 315 (k) 12039
(l) 5639
48
7
18. Simplify:

(a) (03 of 083) (05 01) + 035 + 008,


.
(b) 15 0075 325 13 33 225 136
.
(c) [(627 05) {(05 of 075) 836}] {(025 of 01) (075 of 213)
05}

28 of 227
44 283
(d)
+
of 82

136
13 + 2629
Multiple Choice Questions: [Mark () on the correct answer]
1.
Which one of the following is a rational number?
(a)

(b) 2

5
(d) 4
Which one of the following decimal fractions are similar?
(a) 4.37, 43.7
(b) 0.530, 0.817
(c) 12.34, 12.346
(d) 0.20, 20
(c)

2.

35

3.

Which one of the following fractions can be expressed in to


terminating decimal fraction ?
(a)
(c)

4.

5.

8
?
33

(b) 0.024&
(a) 0.24&
..
(c) 0.240
(d) 0.2& 4&
If 0.47& is converted in simple fraction then which one of the
following would be the result?

(c)

7.

1
3
5
(d)
13

(b)

Which one of the following is in recurring decimal form of

(a)

6.

3
7
41
16

47
90
43
99

(b)
(d)

43
90
47
99

Which one of the following is the correct value of 4 0.125?


(a) 0.64
(b) 6.4
(c) 3.2
(d) 32
(i) The product of two irrational numbers will be irrational.
(ii) 0 is a rational number.
(iii) All those numbers which are not square number, the square
root of those numbers are irrational.
Which one of the following is correct on the basis of the above
informations?
(a) i and ii
(b) i and iii
(c) ii and iii
(d) i, ii and iii
Answer the questions (810) on the basis of the following
informations:
.
.
.
8
0.6, 0.9, 0.5 are three recurring decimal fractions and
is a
98

rational fraction.

36

8.

If 0.9& is expressed in simple fraction then which one of the


following would be the result?
(a) 1

(b) 10

1
1
(d)
10
9
&
&
If 0.6 is divided by 0.5 then which one of the following would be
the result?
(c)

9.

(a) 10
(c) 1.2&
10.

(b) 1.2
(d)

0.83&

Which one of the following is the approximate value of


three decimal place?
(a) 0.283
(c) 0.285

8
98

upto

(b) 0.284
(d) 0.286

CREATIVE QUESTIONS
8.04& , 0.39& 5& and 5.13& 02& are three recurring decimal fractions.
1.
(a) convert 8.04& into simple fraction.
(b) Add the above three fractions after expressing into similar
recurring decimal fractions.
. .
(c) Find the square root of 5.1302 upto four decimal place and
find the approximate value of the square root upto three
decimal places.
. .
.
..
2.
12185, 4218 and 028 are three recurring fractions.
(a) Convert 12.1& 85& into simple fraction.
(b) Divide 12.1& 85& by 42.1& 8&
(c) Find the product of the three numbers.
..
3.
2.8 of 2.2& 7& , 136, 4.4& 2.83& , 1.3& + 2.629& and 8.2 are some fractions.
(a) Convert 2.8 of 2.2& 7& into simple fraction.
(b) Divide 4.4& 2.83& by 1.3& + 2.629& and then multiply the quotient by 8.2.
(c) Divide the result of (a) by 1.3& 6& . Sum this quotient with the
result of (b) and find the square root of the sum upto two
decimal place.

Chapter II

Percentage, Profit - Loss


2.1 Percentage
'Percent' means per hundred, i.e. per one hundred. The word, 'percent' is used to
determine the quantity per one hundred. 15 percent means 15 per 100.15 percent,
15
100 or, 0.15 have the same meaning. The symbol (%) is used for percent. In short,

15 percent is written as 15%.


Percentage is a fraction. Its denominator is 100 and numerator is the number,
calculated as percentage. Percentage is used in as calculating interest, gain and
loss, in increasing and decreasing rate of population etc. 23 parts of 100 parts of
23

Tk. 1 can be expressed in any one of forms Tk. 100 , Tk. 23 or, 23% of Tk. 1.
Some simple fractions and decimal fractions are expressed as percentages:
Simple fraction
Decimal fraction
Hundredth Percentage
1
25
025
25%
4
100
1
50
0

50
50%
2
100
3
75
0

75
75%
4
100
3
150
1

50
150%
2
100
18
360
3.60
360%
5
100
Any simple or decimal fraction can be expressed as percentage and also
percentage can be expressed either in simple or decimal fraction.
Some percentages are expressed in simple and decimal fractions.
Percentage Hundredth Decimal fraction
Simple fraction
20
1
20%
0

20
100
5
66
33
66%
0

66
100
50
375
15
3
375%
3.75
or,
3
100
4
4

38

Remark :
(a) As improper fractions are greater than 1 so percentages can be greater than
100%.
(b) 100% expresses whole of the quantity. 200% or, 300% express twice or thrice
of the whole of the quantity respectively. .
Expression of Simple Fractions as Percentages
3
7
Example 1 : Express 5 and 8 as percentages.
20
3 3 100
60
Solution : 5 =
= 100 = 60%
5 100
1
175
25
2
7 7 100
175
175
1
=
=
=
=
%
or,
87
8
2
2%
2 100 100
8 100
2
1
Ans : 60% and 872 %
21
9
Example 2 : Express 5 25 and 3 16 as percentages.
Solution :
4
21 146 146 100
584
5 25 = 25 =
= 100 = 584%
25 100
1
1425
25
4
9
57 57 100
1425
1425
1
3 16 = 16 =
=
= 100 = 4 % or, 356 4 %
4 100
16 100
4
1
Ans : 584% and 3564 %
Expression of Decimal Fractions as Percentages
Example 3. Express 055 and 436 as percentages.
0.55 100
55
Solution : 055 =
=
100
100 = 55%
4.36 100 436
4.36 =
= 100 = 436%
100
Ans : 55% and 436%

39

Example 4. Express 2678 and 6.0345 as percentages.


2.678 100 267.8
Solution : 2678 =
= 100 = 2678%
100

60345 =

60345 100
= 60345%
100

Ans : 2678% and 60345%


Expression of Percentages in Simple Fractions and Decimal Fractions
1
Example 5. Express 64% and 2246 % in simple fractions.
16
64
16
Solution : 64% = 100 = 25
25
1345
269
6
1345
1
269
29
2246 % = 100 =
= 120 = 2 120
6 100
20
16
29
Ans : 25 and 2 120
1
Example 6. Express 243 % and 2566% in simple fractions.
73
3
1
73
73
73
Solution : 243 % = 3 % = 100 =
= 300
3 100
2566
1283
10
2566 1283
6
2566
283
256.6% = 25610 % = 10 % = 100 =
= 500 = 2 500
10 100
5
73
283
Ans : 300 and 2 500
Example 7. Express 56% and 126% in decimal fractions.
56
Solution : 56% = 100 = 0.56.
126
126% = 100 = 126.
Ans : 056 and 126.

40

1
Examples 8. Express 1238 % and 567% in decimal fractions.
985
8
1
985
123125
Solution : 1238 % = 8 % = 100 = 100 = 123125
567
567% = 100 = 0567
Ans: 123125 and 0567.
Percentage of Numbers
Example 9.
(a) What is 5% of 100?
(c) What is 22% of 450?
5
Solution : (a) 5% = 100

(b) What is 8% of 150?


(d) What is 25% of 680?

5
1
Therefore, 5% of 100 = 100 100 = 5.
1
8
(b) 8% = 100
2
6
8
30
8% of 150 =
150 100
= 12
20
4
1
22
(c) 22% = 100
22
22% of 450 = 450 100 = 99
25
(d) 25% = 100
25
25% of 680 = 680 100 = 170
Ans: (a) 5
(b)12 (c) 99 (d) 170.

N.M.G. -7

41

One Number as a Percentage of Another Number


Example 10.
(a) What percentage is 5 of 25?
(b) What percentage is 24 of 40?
(c) What percentage is 37 of 45?
5
Solution : (a) 5 is 25 of 25.
5
When the fraction 25 is expressed as percentage, it becomes
5
5
4
25 = (25 100 )% = 20%
1
Ans : 20%
24
(b) 24 is 40 of 40.
24
When the fraction 40 is expressed as percentage, it becomes
6
24
24
10
)% = 60%
=
(

100
40
40
10
1
Ans : 60%
|

37
(c) 37 is 45 of 45
37
When the fraction 45 is expressed as percentage, it becomes
740
37 37
2
20

%
=
=

100
%
=
82
9
45 45
9%

2
Ans : 829 %
Number from the Percentage
Example 11. Find the number whose 60% is 48.

42

Solution : 60% = 48
48

1% = 60

4
12
48
20
100% = 60
100 = 80.
155
1

Ans : 80.
Example 12. Find the number whose 75% is 90.
Solution : 75% = 90
90

1% = 75

30
90
4
100% = 75 100 = 120
3
1

Ans : 120
To Express the Ratio as Percentage
If one of two quantities of the same kind is expressed as a fraction of the other
quantity, this fraction is said to be the ratio of their sizes provided that both the
quantities are in the same unit.
2
For example, the ratio of Tk. 2 and Tk. 3 is written as 2 t 3 = 3
The symbol t is the mathematical symbol of ratio. Percentage is also a fraction,
whose numerator is the required number calculated as percentage and the
denominator is 100. In case of ratio if the denominator is arranged to be 100, then
the first quantity can simply be expressed as the percentage of the second. For
15
example, 15 percent means 100 i. e., 15 t 100.
Example 13. Express 17 t 25 as a percentage.
4
17 17 100
68
Solution : 17 t 25 = 25 =
= 100 = 68%
25 100
1
First quantity is 68% of the second quantity.
Ans : 68%

43

Example 14. Express 32 t 60 as a percentage.

32 32 100
160
160
1
Solution : 32 t 60 = 60 =
=
= 3 % = 533 %
60 100 3 100
1
First quantity is 533 % of the second quantity.
1
Ans : 533 %.
Example 15. If one quantity is 64% of another quantity, then what is the ratio of
two quantities?
64
16
Solution : 64% = 100 = 25 = 16 t 25
Ratio of the two quantities is 16 t 25
Ans : 16 t 25

1
Example 16. If one quantity is 1233 % of another quantity, then what is the ratio
of two quantities?
1
370
370
37
Solution : 1233 % = 3 % =
= 30 = 37 t 30.
3 100
Ratio of the two quantities is 37 t 30.
Ans : 37 t 30.
Some Problems Relating to Percentage
Example 17. There are 450 pupils in a school and 18% of them are girls. How
many girls are there in that school?
450 18
Solution : 18% of 450 pupils =
100 pupils = 81 pupils.
The required number of girls = 81.
Ans : 81 girls.
Example 18. Rangan got 576 marks out of 800 marks in the examination. Find his
marks in percentage.
576
Solution : 576 is 800 of 800.

44
576

Expressing the fraction, 800 as percentage, we get


576 576

800 = 800 100% = 72%


The required percentage marks = 72
Ans : 72%
Second method :
Out of
800 marks he got 576 marks
576
"
1
"
"
"
"
800
576 100
"
100 "
"
"
" or, 72 marks.
800
The required percentage marks = 72
Ans : 72%.
Example 19. 10% of monthly salary of Mr. Zafar was deducted for provident fund
and then he received Tk. 2700.00. How much was his monthly salary?
Solution : If Mr. Zafar' s monthly salary is Taka 100, then after deducting Taka 10
for provident fund he receives Tk. (100 10) or, Tk. 90
When Mr. Zafar receives Tk. 90 then his monthly salary is Tk. 100
100
" "
"
"
" 1 " "
"
" " " 90
300
10
100 2700
" "
"
"
" 2700 " "
"
" " "
90
9
1
or, Tk. 3000
The required monthly salary is Tk. 3000.
2nd method : If Tk. 10 percent is deducted for provident fund, Mr. Zafar receives
from his monthly salary Tk. (100 10) or, Tk. 90 percent.
Now 90% of Mr. Zafar's salary = Tk. 2700
2700
" 1% " "
"
" = Tk. 90
2700
" 100% " "
"
" = Tk. 90 100 or, Tk. 3000
Required monthly salary is Tk. 3000.
Ans : Tk. 3000.

45

Example 20. A book of mathematics is sold for Tk. 68 at 85% of the actual price.
What is the actual price?
Solution : 85% of the actual price of the book = Tk. 68
68

1% " "
"
" " " " = Tk. 85
20
4
68 100

100% " "


"
" " " " = Tk.
or, Tk. 80.
85
17
1
Required actual price Tk. 80
2nd method : If the actual price of the book is Tk. 100, then the selling price is
Tk. (100 15) = Tk. 85.
Now if the book's selling price is Tk. 85, actual price is Tk. 100
100
" " " "
"
"
Tk. 1
"
" " Tk. 85
20
4
68 100
" " " "
"
"
Tk. 68
"
" " Tk.
85
17
1
or, Tk. 80.
Required actual price Tk. 80.
Ans : Tk. 80.
Example 21. The mortality rate per annum of a village is 4% of its population. If
the total population of that village is 1250, how many people die per annum?
4
Solution : 4% of 1250 persons = 1250 100 persons = 50 persons.
Required number of persons = 50.

Ans : 50 persons.

Example 22. When the price of tea leaves decreased by 25%, what is the present

price of tea leaves per kg. which was Tk. 72 per kg. before decrease?

46

18
Solution : 25% of Tk. 72 = Tk.

1
72 25
= Tk. 18
100
4
1

Required price = Tk. (72 18) = Tk. 54.


Ans : Tk. 54.
Example 23. Rajat's monthly income increases from Tk.6600 to Tk. 7260. Find
percentage of the increase in his income.
Solution : Increase in Rajat's income = Tk. (7260 6600) = Tk. 660.
660 100
Required precentage increase in income =Tk. 6600
= Tk. 10
Ans : 10%
Example 24. For 15% increase in Bakar's salary it becomes Tk. 5750. What was
his salary before?
Solution : Salary of Bakar increases to (100 + 15)% or, 115%.
Previous salary of 115% = Tk. 5750
5750
"
"
1% = Tk. 115
20
250
5750 100
"
"
100% = Tk.
or, Tk. 5000
115
23
1
The required previous salary is Tk. 5000.
Ans : Tk. 5000
Example 25. When the price of pulse is increased by 25% then at what rate in the
use of pulse be reduced so that there would be no change in the expenditure for
use of pulse?
Solution : For 25% increase in the price of pulse it becomes (100 + 25)% or,
125%. In order to keep the expenditure for pulse unchanged that quantity of pulse
in the increased price obtained by the amount equivalent to the amount as before is
to be used.

After the increase of price be 25%


125 expenditure of previous can provide Tk. 100

use of previous

47

"

"

"

"

"

100
Tk. 125

" "

"

100 100
"" "
125
= Tk. 80 " "
"
Reduction in use of pulse = (100 80)% = 20%
That is, 20% use of previous should be reduced for the no change in expenditure.
Ans : 20%
100

"

"

"

"

"

Tk.

Example 26. Mr. Farid paid 15% per annum income tax amounting to Tk. 8250 .
What was the annual income of Mr. Farid?
Solution : 15% of Mr. Farid's income = Tk. 8250.
8250

1% " "
"
"
= Tk. 15
20
2750
8250 100

100% " "


"
"
= Tk.
or, Tk. 55000.
15
3
1
Required annual income = Tk. 55000.
Ans : Tk . 55000.
Example 27. The population of a town increases by 4% per annum and the
population of that town was 2000000 persons. What will be the population of that
town after 3 years?
Solution : If the number of persons is 100 at the beginning of the year, number of
persons become 104 at the end of the year.
Therefore, 100 persons increase to 104 at the end of one year
104

1 persons increase " 100 " " " " " "
104

2000000 persons increase at 100 2000000 persons

104
104 104
Similarly, 100 2000000 persons increase to 100 100 2000000
1042
or, 100 2000000 person at the end of second year.

48
1043
At the end of third year population becomes 100 2000000 persons.

104 104 104 2000000


persons
100 100 100
or, 2249728 persons.
The required population = 2249728 persons.
Ans : 2249728 persons.
or,

EXERCISE 2.1

1. Express as percentages:
3
7
6
11
5
7
9
(a) 8 (b) 10 (c) 225 (d) 416 (e) 6 (f) 212 (g) 232 (h) 006
(i) 0567
(j) 467 (k) 8459 (l) 00003 (m) 7045 (n) 95 (o) 346
(p) 9t10 (q) 7 t 12
(r) 13 t 25 (s) 25 t 32 (t) 65 t 125.
2. Express in simple fractions :
5
2
3
2
(a) 25% (b) 125% (c) 1236 % (d) 253 % (e) 3427 % (f) 163 %
1
(g) 314 %

1
2
(h) 372 % (i) 663 %.

3.

Express in decimal fractions :


1
1
1
5
(a) 12% (b) 37% (c) 122 % (d) 84 % (e) 1238 % (f) 1158 %
5
3
3
(g) 228 % (h) 1432 % (i) 4216 %
4. (a) What is 25% of 124?
(b) What is 20% of 300?
(c) What is 4% of 250?
(d) What is 10% of 670?
(e) What is 12% of 550?
(f) What is 32% of 750%?
(g) How many is 20% of 60 pencils? (h) How much 25% of Tk. 150 ?
(i) How much is 75% of 3 kg?
(j) How many is 80% of 600 students?
(k) How much is 80% of 10 kilometers?
5. (a) What percentage is 20 of 100?
(b) What percentage is 40 of 80?
(c) What percentage is 60 of 90?
(d) What percentage is 125 of 75?
(e) What percentage is 20 pencils of 150 pencils?
N.M.G. -8

49

(f) What percentage is Tk. 25 of Tk. 60?


(g) What percentage is 5 kilometers of 8 kilometers?
(h) What percentage is 20 kg. of 120 kg?
(i) What percentage is Tk. 10 of Tk. 250?
6. (a) Find the number of which 50% is 25.
(b) Find the number of which 20% is 64.
(c) Find the number of which 60% is 72.
(d) Find the number of which 55% is 132.
(e) Find the amount of which 16% is Tk. 20
(f) Find the distance in km. of which 30% is 24 km.
2

(g) Find the weight in kg. of which 663 % is 160 kg.


(h) Find the amount of which 30% is Tk.70
(i) Find the weight in kg. of which 15% is 45 kg.
7. Percentage of 1st quantity as the 2nd quantity is given below. Find the ratio of
the two quantities:
(a) 25% (b) 32% (c) 48% (d) 60% (e) 125% (f) 350%.
8. The monthly salary of Mr. Hanif is Tk. 5000. He saves 24% of his salary per
month. How much does he save in a year?
9. In a certain school number of male students is 55% of the number of pupils. If
the number of male students is 1100, then what is the number of pupils in that
school?
10. Mr. Helal's monthly income and expenditure are Tk. 4200 and Tk. 2940
respectively. What is the percentage of expenditure of his income?
11. 15% commission is allowed for buying a book on Mathematics. The actual
price of the book is Tk. 120. How much will be needed to buy the book?
12. In the annual examination Rasel obtained 600 out of 800 marks. What is the
percentage of his marks?
13. The monthly salary of Mr. Belal is Tk. 4820. He has to deposit 10% of his
salary to the provident fund. How much is deposited by him to the provident
fund?
14. Mr. Bakar's salary increases from Tk. 4000 to Tk. 4500. Find the rate of
increase in his salary.
15. Rahim Miah bought a cow for Tk. 5000 and sold it for Tk. 4000. How much
his loss in percent?

50
1

16. Popy has to pay house-rent at 72 % of his basic pay. After deduction of his
house-rent she gets Tk. 3700 per month. What is her monthly basic pay?
17. The population of a village increased by 8% and become 21600. What was
the population of that village before?
18. Mr. Moin Uddin transferred 12% of his property to his wife, 20% to his son
and the rest of Tk. 8,16,000 to his daughter. What was the value of his whole
property?
19. 68% of S.S.C Examinees in a school came out successful. The percentage
pass would have been 75% if 14 more students passed. What is the number of
the Examinees?
20. The number of male and female students in a school is 900. 4% male students
left the school and 5% female students got themselves admitted afresh. Thus
there was no change in the original total number of male and female students'.
How many female students were in that school before and at present how
many male students are there?
21. If 6% commission is allowed the amount to be paid for the price of a book is
less by Tk. 15 than the amount to be paid for the price if 5% commission is
allowed. What is the actual price of the book?
22. In a pooling booth, a candidate has been elected by getting 55% vote of the
voters present. He got 10000 votes more than number of votes of his only
competitor. How many voters were present in the pooling booth?
23. In an electoral college 75% voters were present. One of the two
candidates got 55% votes of voters present and it was seen that his number of
votes was more by 75000 than the number of votes of the other candidate.
What was the total number of voters?
24. When the price of tea-leaves is reduced by 20%. Karim can buy tea-leaves of
15 kg. more than previous quantity by Tk. 4500. Find the present and the
previous prices of per kg. tea-leaves.
2

25. When the price of banana is reduced by 163 % then by Tk.75 five more
bananas can buy. What is the present price of each dozen of banana?
26. When the price of suger is increased by 10 percent, then what
percentage in the use of suger should be reduced so that there would be no
change in the expenditure for use of sugar?

51

27. When the price of fish is reduced by 25%, then at what percentage in the use
of fish should increased so that there will be no change of expenditure for use
of fish in the family?
28. When the tax on tea-leaves is reduced by 10 percent, then at what percentage
in the use of tea-leaves is to be increased so that the Government will get tax
on tea-leaves at Tk. 8 percent more than the previous rate?
29. In an examination 52% and 42% examinees failed in English and
Mathematics respectively. If 17% examinees failed in both the subjects, then
at what percentage examinees were successful in both the subjects?
30. In an examination 60% and 50% examinees passed in Bangla and
Mathematics respectively. If 40% examinees passed in both the subjects and
60 examinees failed in both the subjects, what is the total number of
examinees?
31. In an examination 80% and 60% examinees passed in Bangla and Mathematics
respectively and 160 examinees passed in both the subjects. If no body failed
in both the subjects, then what is the total number of examinees?
32. The population of a city increases by 8% in each year. At present the
population of the city is 50000. What will be the population of the city after
two years?
33. In a hostel 25 percent seats are increased in each year. At present there are
512 seats. After 4 years, how many seat will be there?
2.2 Calculation of Interest
When money is deposited in a bank or money is lent to an institution or to a
person or invested in a business, an excess sum of money depending on the total
amount and period of time is obtained. This excess sum of money is called
interest. The sum of money deposited or lent is called the Principal or Capital. The
total sum of money received as interest and principal together is called InterestPrincipal or Increased Principal.
...Interest + Principal = Amount...

The sum of interest on a definite sum of money for a limited period of time is
called the rate of interest. Generally sum of interest on Tk. 100 for a period of one
year is called the rate of interest. This rate is mentioned as the rate of interest
percent per annum. So if in the rate of interest percent yearly or monthly not
mentioned in that case interest is to be calculated on the basis of percent per
annum. If Tk. 4 is the rate of interest percent per annum, then it is written as 4%

52

interest. When interest is calculated on principal only, then it is called simple


interest. The method by which interest of a definite sum of money for definite
period of time determined is called calculation of interest. The problems relating to
interest is generally solved by unitary method. In the discussion of calculation of
simple interest, if three out of four data as principal, interest, rate of interest and
time is known, then the 4th one can be found with the help of unitary method.
To Find Interest and Amount
Example 1. How much is the interest on Tk. 850 in 5 years at 4% per annum?
How much is the amount?
Solution : Interest on Tk. 100 in 1 year is Tk. 4

"

"

" 1

" 1 "

"

"

" 1

" 5 "

"

"

" 850 " 5 "

4
" Tk. 100
45
" Tk. 100
1
1 170
850
4 5
" Tk.
or, Tk. 170
100
25
51

The required interest = Tk. 170 and amount = Tk. (850 + 170) or, Tk. 1020
Ans : Tk. 170 and Tk. 1020.
1
Example 2. How much is the interest on Tk. 600 in 3 years at 52 % per annum?
How much is the amount?
1
11
Solution : Interest on Tk. 100 in 1 year is Tk. 52 or Tk. 2
11

"
" " 1 " 1 " " Tk.
2 100
11 3

"
" " 1 " 3 " " Tk.
2 100
3
6
11 3 600

"
" " 600 " 3 " " Tk.
or, Tk. 99
2 100
1
1
The required interest = Tk. 99 and amount = Tk. (600 + 99) or, Tk. 699.
Ans : Tk. 99 and Tk. 699.

53

rate of interest per annum time principal


100
[Time is always to be expressed in years.]
Example 3. How much is the interest on Tk. 525 in 1 year 6 months at 12% per
annum? How much is the amount?
Solution : Here, time = 1 year 6 months = 1 year + 6 months
1
1
3
= 1 year + 2 year = 12 year = 2 year
Interest on Tk. 100 in 1 year is Tk. 12
12
" " " 1 " 1 " " Tk. 100
3
12 3
" " " 1 " 2 " " Tk.
100 2
3
21
12 3 525
3
189
" " " 525 " 2 " " Tk.
or, Tk. 2 or, Tk. 9450
2
100
25
1
The required interest = Tk. 9450 and amount = Tk. (525 + 94.50)
or, Tk. 61950
Ans : Tk. 9450 and Tk. 61950
Remark : Amount of interest =

To Find the Principal


Example 4. How much will be the principal for which interest is Tk. 200 in 10
1

yerars with rate of interest 22 % per annum?


1
5
Solution : Interest on Tk. 100 in 1 year is Tk. 22 or, Tk. 2

"

" " 100 " 10 " " Tk.

5 10
or, Tk. 25
2

So, if interest is Tk. 25 then principal is Tk. 100


100
"
"
"
1 "
"
" Tk. 25
100 200
"
"
" 200 "
"
" Tk.
or, Tk. 800
25
The required amount of principal = Tk. 800.
Ans : Tk. 800

54

Remark : Principal =

100 interest
time rate of interest per annum

1
Example 5. How much sum of money at the interest of 62 % per annum in 3 years
4 months will be Tk. 78840 as amount?
1
10
Solution : Here, time = 3 years 4 months = 33 years or, 3 years
7884
and amount = Tk. 78840 or, Tk. 10
1
13
Interest on Tk. 100 in 1 year is Tk. 62 or, Tk. 2
5
10
13 10
65
"
" " 100 " 3 " " Tk. 2 3 or, Tk. 3
1
65
So, the principal of Tk. 100 in 3 years 4 months amounts to Tk. (100 + 3 )
365
or, Tk. 3
365
If amount is Tk. 3 then principal is Tk. 100
100 3
"
" " " 1
"
"
" Tk. 365
2
10
108
100 37884
7884
"
" " "
"
" Tk.
or, Tk. 648
10 "
365 10
73
1
1
The required principal = Tk. 648
Ans : Tk. 648
100 amount
Remark : Principal =
(time rate) + 100
To Find the Rate of Interest
Example 6. How much will be the percentage rate of interest per annum if the
interest on Tk. 700 in 5 years is Tk. 105?
Solution : Interest on Tk. 700 in 5 years is Tk. 105

"

" " 700 " 1 "

100

" Tk. 5

55

"

" " 1

"

" " 100 " 1 "

" 1 "

105
5 700
3
21
1
105 100
" Tk.
or, Tk. 3
5 700
1
7
1

" Tk.

The required rate of interest is Tk. 3


Ans : 3%
Example 7. How much will be the percentage rate of interest per annum if Tk.
431 in 8 years becomes Tk. 862 as amount?
Solution : Here, interest on Tk. 431 in 8 years is Tk. (862 431) or, Tk. 431.
Interest on Tk. 431 in 8 years is 431

"

" " 431

"

1"

"

" " 1 " 1

" "

"

" " 100

1"

"

"

431

Tk. 8

431
8 431
1
25
431 100
" Tk.
8 431
2
1
25
1
or, Tk. 2 or, Tk. 122

Tk.

The required rate of interest is Tk. 12 2


1

Ans: 12 2 %
Remark : ...Rate of Interest (in percentage) =

100 interest
time principal

To Find Time
Example 8. In how many years interest on Tk. 175 at 5% per annum will be
Tk. 8750?
175
Solution : Interest = Tk. 87.50 or, Tk. 2
Interest on Tk 100 in 1 year is Tk. 5
5
"
" " 1 " 1 " " Tk. 100

56

7
5 175
35
"
" " 175 " 1 " " Tk. 100
or, Tk. 4
4
2
5
175 4
175 35

The required time = 2 4 years = 2 35 years = 10 years.

1
1
Ans : 10 years.
Example 9. In how many years Tk. 450 at the interest of 6% per annum will be
Tk.558 as amount ?
Solution : Here, interest = Tk. (558 450) = Tk. 108
Interest on Tk. 100 in 1 year is Tk. 6
6
" " " 1 " 1 " " Tk. 100
"

" " 450 " 1 " " Tk.


4

6 450
100 or, Tk. 27.

108
The required time = 27 years = 4 years.
1
Ans : 4 years.
Second method :
Here, interest = Tk. (558 450) = Tk. 108
Interest on Tk. 100 in 1 year is Tk. 6
6
" " " 1 " 1 " " Tk. 100
6 450
" " " 450 " 1 " " Tk. 100 or, Tk. 27
On Tk. 450 interest becomes Tk. 27 in 1 year.
1
" " 450 "
"
Tk. 1 " 27 year
4
1 108
" " 450 "
"
Tk. 108 "
years or, 4 years
27
1
The required time = 4 years.
Ans : 4 years.
N.M.G. -9

57

100 interest
rate principal
(b) Generally it is considered that 30 days make 1 month, 12 months make 1 year
and 365 days make I year . The number of days in a month (Bengali or English) is
to be considered according to calendar if time is calculated from a given date to
another given date.
If a certain month consists of 31 days then 31 days are to be counted for that
month. Similary, for months consisting of 28 days or 29 days, 28 days or 29 days
are to be counted respectively.
Miscellaneous Problems
Example 10. How long would it take a certain sum of money to be double itself as
amount at 5% per annum?
Remark : (a) Time =

Solution : Let, the principal be Tk. 100.


So, amount will be double of Tk. 100, i. e. Tk. 200 and the interest will be
Tk. (200 100) or, Tk. 100.
Interest on Tk. 100 in 1 year is Tk. 5.
Tk. 100 yields an interest of Tk. 5 in 1 year
1
" "
"
"
"
" Tk. 1 " 5 "
"

"

"

"

"

"

Tk. 100 "

1 100
years or, 20 years
5

The required time is 20 years.


Ans : 20 years.
Example 11. Amount of a certain sum of principal becomes Tk. 484 in 3 years
and Tk. 540 in 5 years respectively. Find the rate of interest per annum and the
principal.
Solution : Here, in 5 years amount is Tk. 540
and
3 "
"
" Tk. 484
By subtracting, interest in 2 years = Tk. 56 (in both the cases the principal being
the same)
56

"

"

1 " Tk. 2

"

"

3 " Tk.

56 3
or, Tk. 84.
2

58

In 3 years amount = Tk. 484


"3
" interest = Tk. 84

principal = Tk. 400


So, Interest on Tk. 400 in 3 years is

"

" " 400 " 1

"

"

Tk. 84
84
Tk. 3

"

" " 1

" 1

"

"

Tk.

84
3 400

"

" " 100 " 1

"

"

Tk.

84 100
or, Tk. 7
3 400

The required rate of interest is Tk. 7 and the principal is Tk. 400.
Ans: 7 % and Tk. 400.
Example 12. As a consequence for reduction of rate of interest from 7% to 5% the
income of Ranjit reduced by Tk.70 in 5 years. What was his capital?
Solution: For Tk. 100 income reduces in 1 year by Tk. (7 5) or, Tk. 2
" " 100
"
"
" 5 " " Tk. (2 5) or. Tk. 10
Therefore, if in 5 years time,
income is reduced by Tk. 10 for the capital of Tk. 100
100

"
"
"
" Tk. 1 " "
"
" Tk. 10

"

"

"

" Tk. 70 "

"

"

" Tk.

100 70
or, Tk. 700
10

The required principal is Tk. 700.


Ans : Tk. 700.
Example 13. If at the same rate of interest the total sum of interest on Tk. 300 in 4
years and interest on Tk. 500 in 5 years be Tk. 148. What is the rate of interest
percent per annum.?
Solution : At the same rate of interest, interest on Tk. 300 in 4 years = At the same
rate of interest, interest on Tk. 100 in (3 4) years or, 12 years.
Again, at the same rate of interest, interest on Tk. 500 in 5 years = interest on
Tk. 100 in (5 5) years or, 25 years.

59

The interest for both the principals is Tk. 148 (given),


sum of interest = Tk. 148 which is the interest on Tk. 100 in (12 + 25) = 37 years.
Interest on Tk. 100 in 37 years is Tk. 148
148

"
" Tk. 100 " 1
" " Tk. 37 or, Tk. 4
The required rate of interest is Tk. 4
Ans : 4%
EXERCISE 2.2
1. How much is the interest on Tk. 650 in 6 years at 7 % per annum?
1
2. How much is the interest on Tk. 800 in 4 years 3 months at 32 % per annum?
3. How much less will be interest on Tk. 500 in 3 years if the rate of interest is
reduced from 6% to 4%?
4. How much is the amount if the principal of Tk.1800 yield interest for 6 years
1
at 62 % per annum?
5. How much will the amount if the principal of Tk. 1050 yields interest 4 years
1
at 54 % per annum?
6. How much will be the sum of money if that sum at 4% per annum for 5 years
becomes Tk. 900 as amount?
3
7. How much will be the sum of money if that sum at 34 % per annum for 100
days becomes Tk. 737.50 as amount?
8. How much will be the sum of money if the daily interest be Tk. 1 on that sum
1
at the rate of interest of 418 % per annum?
9. At what rate of interest percent per annum will the interest on Tk. 750 in 2
years be Tk.210?
10. At what rate of interest percent per annum will the interest on Tk. 1200 in 3
years be Tk. 216?
11. At what rate of interest percent per annum will the amount in 5 years be three
times of any the sum of principal?
12. How much is the sum of principal which becomes Tk. 703 as amount in 9
1
years at 53 % per annum?

60

13. What is the rate of interest percent if the amount in 10 years be four times of
any sum of the principal?
14. In how many years will Tk. 290 at 5% be Tk. 377 as amount?
15. In how many years will Tk. 500 at 6% be Tk. 800 as amount?
16. In how many years will Tk. 425 at 5% per annum be Tk. 510 as amount?
17. What is the time required for yielding interest of Tk. 18 on the principal of
1

Tk. 96 at 64 % per annum?


18. As a consequence for reduction of rate of interest from 8% to 6% Rahim's
income reduced by Tk. 77.50 in 5 years. What was his principal?
19. How much will be the sum of money which becomes Tk. 575.00 as
amount in 5 years at the rate of interest by which Tk. 450 be Tk. 504 as
amount in 4 years?
20. As a consequence for increase of rate of interest from 8% to 10% Jalil's
income increased by Tk. 128 in 4 years. What was his capital?
21. Walid borrowed Tk. 400 for 3 years and Tk. 600 for 5 years and gave a total
of Tk. 168 as interest. If the rate of interest of both the cases be the same, then
how much was the rate of interest?
22. In how many years will a certain sum of interest on Tk. 700 at 10% per
annum be found if that certain sum of interest is found from Tk. 1050 in 5
years at 8% per annum?
23. Interest on Tk. 250 in 6 years at 3% per annum is equal to the interest on a
sum of money for 4 years at 5% per annum. What is that sum of money?
24. Amount is Tk. 442 on any sum of principal in 6 years at 5% per annum. In
how many years will its amount be Tk. 510?
25. How much sum of money is to be deposited in the bank at the rate of interest
of 6% per annum so that in each day a sum of Tk. 60 as interest is found?
3

26. A sum of principal becomes Tk. 550 as amount after 5 years and interest is 8
of the principal. How much is the principal and rate of interest per annum?
2

27. The amount for a certain period of time is Tk. 450 and interest is 7 of the
4

principal. If the rate of interest be 37 % per annum, find the time.


28. The interest on Tk. 500 in 4 years at 6% is equal to the interest on a sum of
1

money in 42 years at 5%. How much is that sum of money?


29. If in 20 years amount becomes double of any sum of principal, then in how
many years will it become thrice of that principal?

61

30. Any sum of principal after 3 years is Tk. 1452 as amount and after 5 years is
Tk. 1620 as amount. Find the principal and the rate of interest.
1

31. In how many years will interest of any sum of money at 122 % be equal to 4
32.

33.
34.
35.

of that sum of money?


Any sum of capital becomes its double as amount after 6 years at a particular
rate per annum. How much will be the sum of money which becomes
Tk. 2050 as amount at the previous rate in 4 years?
A sum of principal becomes Tk. 560 as amount after 3 years and Tk. 600 as
amount after 5 years respectively. What is the rate of interest?
A sum of capital was invested for 3 years. If the rate of interest is Tk. 5 then
what is the part of interest in respect to the principal?
A sum of Tk. 750 at the interest of 8% and another sum of Tk. 1250 at 6% were
invested. What is the average percent rate of interest of the total sum of capital?

2.3 Profit and Loss


Transactions of money are made for buying and selling of commodities. Again
money is invested for manufacturing commodities. The expenditure which is done
for buying or manufacturing is the cost price and the price that is obtained by
selling is the selling price.
If the selling price is greater than the cost price then it is said that profit has been
earned. If the selling price is smaller than the cost price then it is said that loss has
been incurred. The amount by which the selling price is greater than the cost price
is the amount of profit. The amount by which the cost price is greater than the
selling price is the amount of loss.
Selling price cost price = profit or, selling price = cost price + profit.
Cost price selling price = loss or, selling price = cost price loss.
Similarly, cost price = selling price profit
and cost price = selling price + loss.
For comparison profit or loss is expressed as percentage. Profit or loss is always
calculated in terms of the cost price.
Remark : The businessmen or the proprietors of factories determine the actual
expenditure by adding rent of buildings, salary of employees, trasportation-cost of
goods and wages of labourers etc. With the cost price of goods this actual
expenditure is known as investment. This investment is taken as the cost price for
determination of profit or loss.

62

Example 1. Hanif bought a watch for Tk. 575 and sold it Tk. 600. What is his
profit or loss?
Solution : The cost price of the watch is Tk. 575 and its selling price is Tk. 600.
Here, as the selling price is greater than the cost price, so he has earned profit.
Profit = Tk. (600 575) = Tk. 25
The required amount of profit = Tk. 25.
Ans : Profit Tk. 25
Example 2. Raihan bought a parker-pen for Tk. 300 and sold it for Tk. 260. What
is his profit or loss?
Solution : The cost price of the parker-pen is Tk. 300 and its selling price is Tk.
260. Here as the selling price is greater than the cost price, so he has incurred loss.
loss = Tk. (300 260) = Tk. 40.
The required amount of loss is Tk. 40.
Ans : Loss is Tk. 40.
Example 3. 4000 kg. of rice at Tk. 7 per kg. was bought and then 2500 kg. at
Tk. 7.15 per kg. and 1500 kg. at Tk. 6.50 per kg. were sold. Thus how much will
be profit or loss? Find the percentage of profit or loss.
Solution : Cost price = Tk. (7 4000) = Tk. 28000
Selling price = Tk. (715 2500) + Tk. (650 1500)
= Tk. 17875 + Tk. 9750
= Tk. 27625

Here as the cost price is greater than the selling price, so loss has been incurred.
Loss = Tk. (28000 27625) = Tk. 375.
The required amount of loss = Tk. 375
Again in Tk. 28000 loss is Tk. 375
375
" " " 1
" " Tk. 28000
" " " 100

"

" Tk.

375 100
75
19
or,
Tk.
or,
Tk.
1
28000
56
56

19
The required loss = 1 56 %
19
Ans : Amount of loss Tk. 375 and 1 56 %

63

Example 4 Rashid buys salt at Tk. 7 per kg. and sells at Tk. 750 per kg. and thus
he makes a profit of Tk. 30. How much salt does he buy?
1
Solution : By selling one kg. of salt profit is Tk. (750 700) = Tk. 050 or, Tk. 2
Here, total amount of profit is Tk. 30
1
Profit of Tk. 2 is from 1 kg. of salt
" " " 1 " " 1 2 kg "
" " " 30 " " 1 2 30 kg " or, 60 kg of salt,
The required quantity of salt = 60 kg.
Ans : 60 kg.
Example 5. Buying one dozen of bananas for Tk. 3750 and then selling them for
Tk. 39.75, how much will be the percentage of profit?
Solution : Here, cost price = Tk. 3750 and selling price = Tk. 3975
Profit = Tk. (3975 3750) = Tk. 225
For Tk. 3750 profit is Tk. 225
225
" " 1
" " Tk.
3750
2.25 100
" " 100
" " Tk.
or, Tk. 6
3750
The required profit = 6%
Ans : 6%
Example 6. Buying 25 oranges by Tk. 100 and then selling 20 oranges for
Tk. 100, what will be the percentage of profit?
Solution : The cost price of 25 oranges is Tk. 100
100

" "
" " 1
"
" Tk. 25 or, Tk. 4
Again, 20 oranges are sold for Tk. 100
100
" 1
"
" " " Tk. 25 or, Tk. 5
Profit for one orange = Tk. (5 4) or, Tk. 1.
So profit in Tk. 4 is Tk. 1
1
"
" " " 1 " Tk. 4

64

"

"

" " 100 " Tk.

1 100
or, Tk. 25.
4

The required profit = 25%


Second Method :
Suppose, any body buys oranges of Tk. 100.
25 oranges are bought by Tk. 100.
Here, 20 oranges are sold by Tk. 100
100
" 1
"
" " " Tk. 20
100 25
" 25 "
" " " Tk.
or, Tk. 125
20
So profit in Tk. 100 is Tk. (125 100) or, Tk. 25
The required profit = 25%
Ans: 25%
Example 7. Buying some litchies at the rate of litchies 10 for Tk. 10 and the same
number at the rate of litchies 8 for Tk. 10 and then selling all of them at 9 for Tk.
10, how much will be the percentage of profit or loss?
Solution : The L. C. M of 10, 8 and 9 is 360.
Suppose, 360 litchies of each kind is bought.
10 litchies are bought by Tk. 10
10
1
"
is "
" Tk. 10
10 360
360 "
are "
" Tk.
or, Tk. 360.
10
Again, 8 litchies are bought by
Tk. 10
10

1 "
is
"
" Tk. 8
10 360

360 "
are "
" Tk.
or, Tk. 450
8
(360 + 360) = 720 litchies are bought by Tk. (360 + 450) or, Tk. 810

But 9 litchies are sold by Tk. 10


10
1 "
is " " Tk. 9
720 "
N.M.G. -10

are "

"

Tk.

10 720
or, Tk. 800
9

65

So, the cost price of 720 litchies is Tk. 810 and the selling price is Tk. 800
Loss = Tk. (810 800) or, Tk. 10
In Tk. 810 loss is Tk. 10

10

Tk. 1

" " Tk. 810


10 100
19
100
Tk. 100 " " Tk. 810
or, Tk. 81 or, Tk. 181
19

The required loss 181 %


19

Ans : 181 %
Example 8. Mr, Rafiq buys a car for Tk. 3,50,000. What should be the selling
price for a profit of 10%?
Solution : If the cost price be Tk. 100, then selling price is Tk. (100 + 10) or,
Tk.110.

If cost price be Tk. 100 then selling price is Tk. 110


110

"

"

" Tk. 1

"

"

" Tk. 100

"

"

" Tk. 350000 "

"

" Tk.

"

110 350000
or, Tk. 3,85,000
100

The required selling price = Tk. 3,85,000.


Ans : Tk. 3,85,000
Example 9. A profit of 20% is made by selling a bicycle for Tk. 7200. What is the
cost price of the bicycle?
Solution : If the cost price be Tk. 100, then at the profit of 20% the selling price
be Tk. (100 + 20) or, Tk. 120.
If the selling price is Tk. 120 then the cost price is Tk. 100
100
" "
" "
1
" " " " " Tk. 120
"

"

"

"

7200 "

"

"

The required cost price =Tk. 6000


Ans : Tk. 6000.

"

" Tk.

100 7200
or, Tk. 6000
120

66

Example 10. A loss of 10% is incurred by selling a bike for Tk. 18000. How
much is it sold to make a profit of 12%?
Solution : If the cost price be Tk. 100, then the selling price at a loss of 10% is
Tk. (100 10) or, Tk. 90. Selling price at a profit of 12% = Tk. (100 + 12) or, Tk.
112.
The selling price instead of Tk. 90 will be Tk. 112
112
"
"
"
"
" " 1 " " Tk. 90
"

"

"

"

" " 18000 " " Tk.

112 18000
or, Tk. 22400.
90

The required selling price = Tk. 22400.


Ans : Tk. 22400.
Example 11. A cow is sold at a loss of 12%. If the cow can be sold by Tk. 1200
more, then there has been a profit of 8%. What is the cost price of the cow?
Solution : If the cost price of the cow be Tk. 100, then the selling price at a loss of
12% is Tk. (100 12) or Tk. 88. The selling price at a profit of 8% = Tk (100 + 8)
or, Tk. 108.

Excess of the selling price for making profit = Tk. (108 88) or, Tk. 20.
If excess of selling price be Tk. 20, then the cost price Tk. 100
100
"
" "
" " Tk. 1
" " "
" Tk. 20

"

"

"

"

" Tk. 1200 "

"

"

100 1200
20
or, Tk. 6000.

"

Tk.

The required cost price = Tk. 6000.


Ans : Tk. 6000.
Example 12. Naser buys 8 dozen of eggs at Tk. 30 a dozen and 12 dozen of eggs
at Tk. 25 a dozen. At what rate is eggs per dozen sold to make a profit of Tk. 3 a
dozen in average?
Solution :
Cost price of 8 dozen of eggs at Tk. 30 a dozen = Tk. (30 8) or, Tk. 240
" " " 12 " " " " " 25 " " = Tk. (25 12) or, Tk. 300.
By adding, cost price of 20 dozens of eggs = Tk. (240 + 300) or, Tk. 540

67

If a profit of Tk. 3 a dozen is made, total amount of profit for 20 dozen =


Tk. (20 3) or, Tk. 60
Therefore, 20 dozen eggs are sold for Tk. (540 + 60) or Tk. 600
600
"
1
"
" " " " Tk. 20 or, Tk. 30
The required selling price = Tk. 30 a dozen.
Ans : Tk. 30.
Example 13. 180 tons of rod are manufactured in a steel mill, the raw materials
per ton bilet is bought for Tk. 12000. The monthly incidental expenditure of that
mill is Tk. 90000. How much is the selling price of per ton rod to make a profit of
10%?
Solution :
To manufacture 180 tons of rod, the cost price of 180 tons of bilet
= Tk. (180 12000) = Tk. 2160000.
The investment for manufacturing 180 tons of rod = Tk. (2160000 + 90000)
= Tk. 2250000.
"
"
"
"
1
" " " = Tk. (2250000 180)
= Tk. 12500.
To make a profit of 10% rod of Tk. 100 is to be sold by Tk. (100 + 10)
or, Tk. 110.
Rod of Tk. 100 is to be sold by Tk. 110
110
" " " 1 " " " " " " 100

"

"

" 12500 " " "

"

" "

110 12500
or, Tk. 13750.
100

(... the cost price of per ton rod is Tk. 12500)


The required selling price of per ton rod is Tk. 13750.

Ans : Tk. 13750.

68

EXCERCISE 2.3
1.
2.
3.
4.
5.
6.
7.
8.
9.

10.
11.
12.
13.
14.
15.

How much is the sum of profit or loss if Rashid bought a parkcr pen for
Tk. 325 and sold it for Tk. 300?
Habib buys 50 kg. of rice at Tk. 15 per kg. and sells the whole quantity at
Tk. 15.25 per kg. As a result how much is the profit or loss?
Taher bought 40 kg. of rice for Tk. 300 and sold them at Tk. 775 per kg. How
much will be his profit or loss?
A box of apples was sold for Tk. 750 at a loss of Tk. 90. If that box is sold for
Tk. 850, how much will be his profit or loss?
Lozens are bought at 50 paisa each and sold them at 60 paisa each and for this
deal a total profit of Tk. 2.50 was made. How much lozens are bought?
One person buys some quantities of tea-leaves at Tk. 50 per kg. By selling the
whole quantity of tea-leaves at Tk. 45 per kg. he incurrs a loss of Tk. 300.
How much were the quantity in kg. of tea-leaves he bought?
80 metres of cloth were bought for Tk. 2000. 60 metres at Tk. 24 per metre
and the rest at Tk. 27 per metre were sold. How much was profit or loss?
100 books on Arithmetic were bought for Tk. 4500 and sold them at
Tk. 4525 each. How much will be profit or loss?
A person bought 140 kg. of pulse at Tk.32 per kg. for his family's
consumption. After 2 months, he finds that 28 kg. of pulse have been
consumed. Now at what rate is the remaining quantity of pulse to be sold, so
that he gets back the whole amount of cost price?
One person exchanged 22 kg. of pulse at Tk. 25 per kg. with 15 kg. of pulse at
Tk. 35 per kg. How much was his profit of loss on the deal?
One person bought rice and found that the cost price of 20 kg. is equal to the
selling price of 25 kg. What was his percentage profit or loss?
If an item is bought for Tk. 175 and sold for Tk. 189, then what is the
percentage profit or loss?
If 20 metres of cloth were sold for the price by which 30 metres of cloth were
bought, what was the percentage of profit or loss?
One person sells rice at Tk. 20 per kg. to make a profit of 25%. What is the
cost price of per kg. rice?
One person bought 1000 mangoes at Tk. 250 a hundred and sold half of them
at Tk. 300 a hundred. Then half of the rest at Tk. 250 a hundred and the
remaining at Tk. 200 a hundred were sold. What was his percentage profit or
loss?

69

16. Myrobalans are bought at the rate of 10 per Taka and sold at the rate of 8 per
Taka. What will be the percentage profit?
17. If myrobalans are bought at the rate of 15 per Taka, how many is the number
of myrobalans are to be sold per Taka to make a profit of 25% ?
18. If a thing is sold for Tk. 252, there is a loss of 16%. If it is sold for Tk. 312,
then what will be the percentage profit or loss ?
19. A loss of 15% is incurred by selling a watch for Tk. 612. How much is sum of
money by which it is sold to make a profit of 10%?
20. There has been a loss of 25% by selling 8 bananas for Tk. 5. What was the
cost price of a dozen of bananas?
3

21. The selling price of a house is 4 of its cost price. Find the percentage in profit
22.
23.
24.
25.
26.
27.
28.

29.
30.

or loss.
Potatoes are bought for Tk. 250 per kg. and sold at a loss of 10%. How much
is the selling price of one kg. of potatoes?
If 12 things are bought for Tk.10 and sold 8 for Tk. 10. What is the
percentage in profit or loss ?
If a thing is sold for Tk. 378, there is a loss of certain amount and if it is sold
for Tk. 480, then it makes a profit of three times of that certain amount of
loss. What is the cost price of the thing?
A pen is sold at a loss of 20%. If the selling price would have been Tk. 6750
more, then there will be a profit of 10%. What is the cost price of the pen?
5% of total number of fruits of a fruit-seller has been rotten and also 5% of
them has been damaged during transportation. What is the percentage profit
by which he sells the rest so that he profit 20% as a whole ?
A goat is sold at a loss of 10%; If the selling price is Tk. 45 more, there would
have been a profit of 5%. What is the cost price of the goat ?
The price of a shirt and a trouser is Tk. 52500. If the prices of a shirt and a
trouser are increased by 5% and 10% respectively, then to buy those it costs
Tk. 56875. What is the cost price of each of a shirt and a trouser?
A person buys some bananas at Tk. 15 per dozen and the same number of
bananas at Tk. 10 per dozen and then sells all of them at Tk. 14 per dozen.
What is the percentage profit or loss?
The cost price of a horse and a cow is Tk. 10000. The horse is sold at a profit
of 20% and the cow is sold at a loss of 15% and thus a total profit of 6% is
made. What is the price of the horse and the cow separately?

70

31. Some bananas at 10 for Tk. 20 and the same number at 15 for Tk. 20 were
bought and then sold all of them at 12 for Tk. 20. What will be the percentage
in profit or loss ?
32. Some myrobalans at the rate of 15 per Taka and the same number at the rate
of 20 per Taka were bought and then sold all of them at the rate of 18 per
Taka. What will be the percentage in profit or loss ?
33. Rahim buys 15 dozen of bananas at Tk. 21 per dozen and 20 dozen at Tk. 14
per dozen. What is the selling price of each dozen of banana if on the average
he makes a profit of Tk. 5 per dozen?
34. Babul buys 20 dozen of eggs at Tk. 24 per dozen and 12 dozen at Tk. 32 per
dozen. What is the selling price of each dozen of eggs if on the average he
makes a profit of Tk. 6 per dozen?
35. Kamal buys 100 eggs at Tk. 3 per egg and 400 eggs at Tk. 325 per egg. If he
wants to make 5% profit, then what is the selling price of an egg?
36. 'I'he manufacturer sells goods to a wholesaler at a profit of 20% the
wholesaler sells to the retailer at a profit of 20% and the retailer sells to the
customer at a profit of 20%. If the customer's price of that goods is Tk. 21.60
then what is the manufacturing cost of goods?
37. While selling a thing a commission of 10% on the written price in the list of
things was allowed to make a profit of 20%. What was the percentage in
terms of the cost price of the excess sum of money marked in the price list?
38. A businessman has to spend monthly Tk. 2000 as house rent. Tk. 5000 as
salary of employees and Tk. 3000 as other expenses. He buys rice of Tk. 5
lacs and then sells in every month. If he wants to make 10% profit per month,
what is the selling price of rice of Tk. 5 lacs?
39. In a factory 50,000 bags of cement are manufactured. The incidental charges
in a month of that factory is Tk. 80,000 and Tk. 75,00,000 in a month as
expenditure for raw materials. If 20% profit is to be made, what is the price of
each bag of cement?
Multiple Choice Questions: [Mark () on the correct answer]
1.
What percent is 30 of 50?
(a) 30%
(b) 50%
(c) 55%
(d) 60%

71

2.

Which one of the following is the conversion of 28t35 in


percentage?
(a) 63%
(b) 65%
(c) 72%
(d) 80%

3.

Observe the following informations:


(i) The 100% means the whole part.
(ii) The conversion of

3
into percent is 60%
5

(iii) It is used to write 15% for 15 percent.


Which one of the following is correct in view of the above
informations?
(a) i and ii
(b) ii and iii
(c) i and iii
(d) i, ii and iii
4.

Which one of the following numbers is 65% of 39?


(a) 100
(b) 65
(c) 60
(d) 55
Answer the questions (56) in view of the following informations:
Mr. Rahim deposited Tk. 15,000 at the profit rate 8% per annum.

5.

How much Taka would be yearly profit?


(a) 1500
(b) 1200
(c) 1000
(d) 800

6.

How much Taka would become with profit after 6 years?


(a) 7200
(b) 16200
(c) 21200
(d) 22200

7.

Buying15 oranges at Tk. 100 and then selling 12 oranges for


Tk.100, what will be the percentage of profit or loss?
(b) 20% loss
(b) 20% profit
(c) 25% loss
(d) 25% profit
Answer questions (8-9) in view of the following informations:
Mr. Aziz paid Tk. 1080 for the electricity bill for the month of

72

8.

9.

May. A VAT at the rate 15% on the amount of the electricity bill
is fixed.
How much taka is the VAT?
(a) 15
(b) 85
(c) 126
(d) 162
How much taka is the price of the unit consumed by Mr. Aziz?
(a) 1065
(b) 995
(c) 918
(d) 854

CREATIVE QUESTIONS
1.

2.

3.

The population of Gopaldy Union increased to 64800 at the rate of


8%. 75% of the present population of that area are voters.55% of
the total voters are female. 75% of the total voters were present at
union parisad election. Amongst the two participating candidate
one was elected obtaining 60% votes out of the voters presented.
(a) What was the population of union before increased?
(b) What percent is the male voters of the total voters at present
of that union? Calculate the numbers of male and female
voters.
(c) In what difference of casted votes the elected candidate
defeated the other candidate?
The retail price of a good is Tk. 30030. The producer gained 4%,
whole seller gained 5% and retailer gained 10% by selling the
good.
(a) If the retailer sale giving 15% commission on the listed price,
then find the list price of the good.
(b) Find the buying price of whole seller.
(c) How much more is the retail price than the production cost?
A person invested Tk. 750 for profit rate 8% and Tk. 1250 for the
profit rate 6%.
(a) Find the profit for the principal Tk. 1250.
(b) Find the average rate of profit.
(c) In how many years the principal will be doubled?

N.M.G. -11

Chapter III

Measurement and Unit


3.1 Conception regarding measurement and the whole quantity in units and
their uses.
In our every day life it is required to measure the transactions in buying and
selling and in different fields and also to find total population, total number of
plants, total number of animals and birds. These are determined by counting or by
measurement. Measurement is also used for determining area and volume.
Total population, total number of plants and total number of animals and birds are
determined by counting. To measure length, to find weight and to find volume of
liquid materials, measurement used. There are different kinds of measurements
which are used in different fields. Quantity of weight is used for things. But
quantities of oranges, bananas, litchies, eggs, lemons etc. are expressed numbers
by counting.
Either in counting or for measurement unit is essential. For counting the first
natural number 1 is unit. Adding with 1 the second number is found. Adding I
with 2nd unmber 3rd number is found, proceeding as in the above way all the
natural numbers are found.
In order to measure the length any line is divided into some equal parts of which
each part will be 1 unit. It will be one unit of length, similarly to measure the
quantity of weight a body is divided into some pieces of equal quantites of weights
of which the weight of one piece will be one unit . This will be one unit of weight.
Again the unit of measurement for the volume of liquid materials can similarly be
found for measurement of area a piece of paper is divided into some small equal
pieces of square sizes. Each small piece of square size is taken as one unit. It will
be one square unit.
A Hali (a group of 4) or a dozen is used as unit for oranges, eggs, lemons etc. 4
oranges are called one hali of oranges. 12 oranges are called one dozen of oranges.
Here a dozen is taken as one unit. So at the time of selling or buying, it is said to
be one dozen, two dozens etc. Again for litchies hundred is known as unit, one
hundred litchies called 'hundred', 20 fishes are called 'one kuri'.
There are different systems for measurement. The unit in each system is different.
The different units are also different in respect of their values, but for counting
population and number of plants unit is the same. In all the fields an idea regarding
the whole quantity with the help of units either in counting or in measurement can

74

be obtained. The number of people that live in a village is known by counting. The
quantity of rice in a store or in a godown is known by measuring weight. How
much water are there in a reservoir is known by measuring the volume of liquid
materials. Even for various problems relating to time, there are different units
relating to time. 1 is taken as unit for counting in all the countries. But for
measurement, there are different units in different countries.
3.2. Measurement in Metric System
As there are different units for measurement for use in different countries, so there
is much difficulty in carrying out international trade and transactions. Therefore,
for measurement in international trade and transactions metric system is used. The
speciality of this measurment is that calculation is made in base ten. Measurement
in this system can easily be expressed by decimal fractions. This system was first
introduced in France in the Eighteenth century.
The unit for measuring length is a metre. Millionth of longitudinal length from the
north pole of the earth to the equator is considered to be one metre. Subsequently,
this measurement of length was not considered to be convenient, so the length of a
piece of rod made of platinum kept in the museum of Paris had been accepted as
one metre. This length is taken as a unit for linear measurement.
If the length is small, then it is taken as centimetre and if it is large, then it is expressed in
kilometre. The length of the small line-segments are written in centimetres and the length
of the roads are written in kilometres. The metric system has been name from the unit of
length, metre.
The unit of measurement of weight is a gramme. This is also unit of metric system. Less
quantity of weight of a body is expressed in gramme. More quantity of weight of a body
is expressed in kilogramme (kg). The unit for measuring the volume of liquid materials is
a litre. This is also unit of metric system. Litre and kilolitre are used respectively for less
quantity and more quantity of volumes of liquid materials.
The metric system has been introduced in Bangladesh from 1st July, 1982. Now in each
measurement of length, weight and volume this system is fully in use.
Units for Measurement of Length in Metric System
10 millimetre (m.m.) = 1 centimetre (c.m.)
10 centimetre
= 1 decimetre (decim.)
10 decimetre
= 1 metre (m.)
10 metre
= 1 decametre (deca m.)
10 decametre
= 1 hectometre h.m.)
10 hectometre
= 1 kilometre (k.m.)

75
Unit of measuring length: Metre....
The relationship between the metric unit of length measurement and the principal unit of
metre is given below :

Kilometre

Hectometre

Decametre

Metre

Decimetre

Centimetre

Millimetre

1000 m.

100 m.

10 m.

1 metre

1
10 m.

1
100 m.

1
1000 m.

or, 01m.

or, 001m.

or, 0001m.

Remark : (a) In metric system if any length is converted from lower to upper or
from upper to lower units by writing the digits side by side the decimal point is to
be shifted to the left or to the right according to the requirements.
For example, 5km. 4h.m. 7 decam. 6m. 9decim. 2 cm. 3m.m.
= 547692.3 cm = 5476923 m. = 5476923 decam.
= 5476923 h.m. = 5476923 k.m.
(b) The word representing multiple from Greek language and the word
representing portion from Latin language have been used before the name of unit.
In Greek language deca means 10 times, hecto means 100 times and kilo means
1000 times. In Latin language deci means tenth, centi means hundredth and milli
means thousandth.
Units for Measurement of Length in British (English) System

12 inches
3 feet
1760 yards

= 1 feet (ft)
= 1 yard (yd)
= 1 mile (m)

Relation between Metric and British Systems in the Measurement of Length

1 metre
1 km
1 inch
l yard
1 mile

= 3937 inches (approx.)


= 062 mile (approx.)
= 254 cm (approx.)
= 09144 metre (approx.)
= 1.6 kilometre (approx.)

76

Remark : (a) It is not possible to find exactly the relationship between Metric
system and British system. So the value obtained from a relationship is expressed
approximately to some places of decimal or only to some places of decimal.
(b) There is a scale for measurement of smaller lengths. It is 30 cm. or 1 feet in
length. One side of it is marked upto 30 cm. by dividing each centimetre into 10
small divisions. The other side of the scale is marked upto 12 inches by dividing
each inch into either 8 or 10 small divisions. Similarly a tape is used for the
measurement of larger length. The tape is either 30 metre or 100 feet in length.
One side of it is marked by dividing each centimetre into 10 divisions in
centimetres and metres, the other side of it is marked by dividing each inch into
either 10 or 8 divisions in inches and feet.
Units for Measurement of Weight in Metric System

10
10
10
10
10
10

milligramme (m. gm.)


centigramme
decigramme
gramme
decagramme
hectogramme

=
=
=
=
=
=

1 centigramme (c. gm.)


1 decigramme (decigm.)
1 gramme (gm.)
1 decagramme (decagm.)
1 hectogramme (h. gm.)
1 kilogramme (kg.)

Unit of measuring weight : Gramme

1 kilogramme or, kg =1000 gramme or, gm


There are two other units in metric system for measurement of weight, they are
given below. When the materials are larger in quantity then these two units are
used, specially for measurement of rod and cement or for measurement of food
grains.
100 kilogramme (kg)
= 1 quintal
1000 kilogramme or, 10 quintal = 1 metric ton.
Remark : Balance and weighting materials of fixed quantity are used for
measurement of weight in towns and villages. The weighting materials are of 1
gramme, 5 grammes, 10 grammes, 50 grammes, 100 grammes, 200 grammes, 500
grammes, 1 kg., 2 kg., 5 kg., 10 kg., 100 kg ........... etc.
But in the town marked balance is used for measurement of weight, such balance
is marked by dividing each kg. into 1000 divisions .

77

Remark : In the local system the units of weights were chhatak, seer and maund.
After the introduction of metric system this local system is not in use.
Units for Measurement of Volume of Liquid Materials in Metric System

10
10
10
10
10
10

millilitre (m.l.)
centilitre
decilitre
litre
decalitre
hectolitre

1000 millilitre

=
=
=
=
=
=

1 centilitre(c.l.)
1 decilitre (decil.)
1 litre (1.)
1 decalitre (deca1.)
1 hectolitre (hecto1.)
1 kilolitre (k.l.)

1 litre

Remark : For measurement of the volume of liquid materials measuring container


of litre is used. These measurirg container are angular shaped made of alumunium
1 1
or tin sheets which are of 4 , 2 , 1,2, 3, 4, 5,------ etc. litres.
Again these may be vertical container made of transparent glass marked by 25, 50,
100, 200, 300, 400, 500, 600, 700, 800,900, 1000 millilitres.
Remark : If the units in the metric system of one kind of measurement is known,
then the rest of them can easily be remembered. If the units of measurement of
length is known, then the other two are found by putting gramme or, litre
respectively in place of metre.
Measurement of land :

The area of a rectangular region = measurement of length measurement of


breadth
The area of a square region = (length of side)2
1
The area of a triangular region = 2 length of base length of height
Remark : In the measurement of length and breadth, they should be of the same
unit. If length is 1 metre and breadth is 1 metre, then the area will be 1 square
metre.

78

Units for Measurement of Area in the Metric System

100 Square centimetre (sq. cm.)


100 Square decimetre
100 Square metre
100 Ayor

=1
=1
=1
=1

Square decimetre (sq.dccim.)


Square metre (sq.m.)
Ayor (sq. decametre)
Hector, or 1sq. hectometre

Principal unit for measurement of land : Square metre


Remark : The meaning of 2 square metre is not the same as that of 2 metre
square. 2 metre square means a square region of which the length of each side is 2
metres and whose area is 4 square metres. 2 square metre means such a region
whose area is 2 square metres.
Units for Measurement of Area in the British System

144 square inches


9 square feet
4840 square yards
100 decimals

=
=
=
=

1 square feet
1 square yard
1 Acre
1 Acre

Units for Measurement of Area in Local System

1 square hand
20 gandas
16 chhataks
20 kathas

=
=
=
=

1 ganda
1 chhatak
1 katha
1 bigha

Relationship between Metric and British System in Measurement of Area

1 square centimetre
1 square metre
1 hector
1 square inch
1 square feet
1 square yard
1 square mile

=
=
=
=
=
=
=

016 square inch (approx.)


1076 square feet (approx.)
247 acre (approx.)
645 square centimetre (approx.)
929 square centimetre (approx.)
084 square metre (approx.)
640 acre.

79

Relationship between Metric, British and Local systems in Measurement of


Area

1 square hand
= 324 square inches
4 square hand or, 4 ganda
= 9 square feet = 0836 square metres (approx.)
1 katha = 720 square feet
= 80 square yards = 6689 square metres (approx.)
1 bigha = 1600 square yards = 1337.8 square metres (approx.)
1 acre = 3 bigha 8 chhataks = 4046.24 square metres (approx.)
1 decimal = 4356 square feet (approx) =1000 square kori (100 kori = 66 feet)
1 square mile
= 1936 bigha
1 square metre
= 4.78 ganda (approx.) = 0239 bigha (approx.)
1 ayor
= 239 bigha (approx.)

Volume
..Cubic units of a solid body is volume...

volume of a rectangular solid body = measurement of length measurement of


breadth measurement of height.
Remark : Expressing the measurements of length, breadth and the height in the
same unit, measurement of volume is found in cubic unit. The volume of the body
whose length 1 cm, breadth 1 cm and height 1 cm is 1 cubic centimetre.
Units for Measurement of Volume in the Metric System
1000 cubic centimetre (c.c)
1000 cubic decimetre
1 cubic metre
10 cubic stayor

=
=
=
=

1 cubic decimetre (c. decimetre)


1 cubic metre (c.m.)
1 stayor
1 decastayor

Relationship between Metric and British Units in Measurement of Volume

1 stayor
=
353 cubic feet ( approx.)
1 deca stayor =
1308 cubic yards (approx.)
1 cubic feet
=
2867 litre (approx.)
Remark : Weight of 1 cubic centimetre of pure water at the temperature of 4
selcius is 1 gramme.
1000 cubic centimetres = 1 litre Weight.
Weight of 1 litre of water = 1 kilogramme.

80

Example 1. A seller of bananas has 13 dozen of bananas. How many are bananas
in his possession?
Solution :
1 dozen of bananas = 12 bananas
13 "
"
"
= (13 12) or, 156 bananas
Ans : 156 bananas
Example 2. A runner runs for 24 turns in a round track measuring 400 metres.
How long does he run?
Solution : If he runs for 1 turn, the distance is 400 metres
" " " " 24 " "
"
" (400 24) metres
or, 9600 metres or, 9 km. 6 h.m.
The required distance = 9 km.6 h.m.
Ans : 9 km. 6 h.m.
Example 3. If Ranjit runs for 16000 metres, then how many kilometre did he run?
Solution : 1000 metres
= 1 kilometre
1
1
"
= 1000 "
1 16000
16000 "
= 1000
" or, 16 km.
Ranjit runs = 16 kilometres.
Ans : 16 kilometres.
Example 4. Kalu Mia produces 500 kg. 700 gm. of potatoes in one piece of his
land. How much will he produce potatoes in his 12 pieces of land of which the
area of one piece is equal to that of the previous piece?
Solution : In 1 piece of land he produces 500 kg. 700 gm potatoes
" 12 "

"

"

"

"

(500 kg. 700 gm) 12


= 6008 kg. 400 gm.
= 6 metric tons 8 kg 400 gm

The required production 6 metric tons 8 kg 400 gm.


Ans : 6 metric tons 8 kg 400 gm.
Example 5. 28 metric tons of paddy was produced in 16 acres of Liton's land.
How much is the quantity of paddy produced in one acre of his land?
N.M.G. -12: In 16 acres of land he produced 28 metric tons of paddy
Solution

81

28
" " "
16 "
or, 1 metric ton 750 kg. of paddy.
The required production per acre = 1 metric ton 750 kg.
Ans : l metric ton 750 kg.
Example 6. 20,000 metric tons of rod per month are manufactured in a steel mill.
How much is the daily production in that mill?
Solution : In 30 days production is 20000 metric tons of rod
20000
" 1 "
"
" 30 "
" " "
2
or, 666 metric tons 666 kg 6663 gm of rod.
2
The required quantity = 666 metric tons 666 kg. 6663 gm.
2
Ans : 666 metric tons 666 kg. 6663 gm.
Example 7. A businessman sells 20 kg. 400 grammes of pulse in a certain day.
How much is the quantity of pulse which he sells in a month on the basis of that
certain day?
Solution : He sells in 1 day 20 kg. 400 gm.
" " " 30 days (20 kg. 400 gm.) 30 or, 612 kg.
" 1

"

"

"

"

"

The required quantity 612 kg.


Ans : 612 kg.
Example 8. A motor car runs 80 km. by 10 litres of diesel. How much is the
quantity of diesel required to run 1 km?
Solution: It runs 80 km. by 10 litres of diesel
10
" " 1 " " 80 litres or, 125 millilitres of diesel.
The required quantity of diesel =125 millilitres.
Ans: 125 millilitres.
Example 9. A rectangular region is 80 metres in length and 60 metres in
breadth. What is the area of the region?
Solution : We know area of a recgular region = length breadth

82

Here, length = 80 metres and breadth = 60 metres


The required area = (80 60) square metres = 4800 square metres.
Ans: 4800 square metres.
Example 10. A rectangular region is 40 metres in length and 30 metres 30 cm. in
breadth. What is the area of the region?
Solution :
Length = 40 metres = (40 100) cm. = 4000 cm.
breadth = 30 metre 30 cm. = (30 100) cm. + 30 cm. = 3030 cm.
The required area = (4000 3030) square cm.
= 12120000 square cm.
= 1212 square metre.
= 12 ayor 12 square metre.
Ans : 12 ayor 12 square metre.
Example 11. A rectangular garden is 40 metre in length and 30 metre in breadth. l
metre wide path passes arround the garden inside. What is the area of the path?
Solution : The length of the garden is 40 metres
and breadth of
"
is 30 "
40 metre
The area of the garden = (40 30) square metres
1
1
or 1200 square metres. 1
38 metre
1
As the path is 1 metre wide, so excluding the
30
path, length of inner portion of the rectangular
metre
garden = {40 (1 2) } metres = 38 metres
1
1
28 metre
and its breadth = {30 (1 2)} metres = 28 metres
1
1
Area of inner portion of the rectangular
garden = (38 28) square metres = 1064 square metres
Area of the path = (1200 1064) square metres =136 square metres.
Ans : 136 square metres.
Example 12. The measurement of the base of a triangle is 4 metres and its height
3 metres. What is the area of the triangular region?

83

1
Solution : The area of the triangular region = 2 (base height)
1
= 2 (4 3) square metres
= 6 square metres.
Ans : 6 square metres.
Example 13. If the area of a rectangular garden is 840 square metres and its length
is 40 metre, what is its breadth?
Solution : Area = 840 square metres and length = 40 metres
Breadth = area length = (840 40) metres = 21 metres
Ans: 21 metres.
Example 14. The length of a box is 2 metres, breadth 1 metre 50 cm. and height 1
metre. What is the volume of the box?
Solution : Length
= 2 metres = 200 centimetres
Breadth = 1 metre 50 centimetres
= 1 metre + 50 centimetres = 150 centimetres
and height = 1 metre = 100 centimetres
Volume of the box = (length breadth height)
= (200 150 100) cubic cm.
= 3000000 cubic cm. = 3 cubic metres.
Second method :
Length = 2 metres,
50

breadth = 1 metre 50 centimetre = 1 + 100 metres

1
3

= 1 + 2 metres = 2 metres

and height = 1 metre.


3

Volume of the box = 2 2 1 cubic metres

= 3 cubic metres.
Ans : 3 cubic metres.
Example 15: The length of a reservoir is 3 metres, breath is 2 metres and height is
4 metres. How much in litre and in kilogramme of pure water will it contain ?

84

Solution : Length
= 3 metres = 300 centimetres
Breadth
= 2 metres = 200 centimetres
and Height
= 4 metres = 400 centimetres
Volume of the reservoir = (300 200 400) cubic centimetres.
= 24000000 cubic centimetres

= 24000 litres [Q 1000 cubic centimetres = 1 litre]


The weight of 1 litre pure water is 1 kilogramme.
The weight of 24000 litre pure water is 24000 kilogramme.
Ans : 24000 litre and 24000 kilogramme.

EXERCISE 3
1.

The population of a town is 50000. In each day 20 persons die and there is
birth of 30 babies. What will be the population in that town after one year?
2. The population of a village is 7000. In each day 3 persons go to the town in
search of employment and 2 persons come back to the village. What will be
the change in population after 1 month?
3. Rahim exchanges 5 dozen of hen's eggs with 7 dozen duck's eggs
belonging to Karim. Rahim had 9 dozen of eggs before the exchange was
made. How many is the number of eggs in possession of Rahim now?
4. The price of each lemon is Tk. 1. How much sum of money did Ratan get by
selling 20 halis of lemons?
5. If the price of walking fish (Koi) is Tk. 50 a kuri, then what is the price of one
walking fish (koi fish) ?
6. A seller of bananas has 35 dozen of bananas. How many is the number of
bananas in his possession?
7. The distance between two places is 10 km. Express it in metres.
8. The circumference of a wheel of a vehicle is 625 metres. How many will be
the number of turns of the wheel if the vehicle moves for a distance of 40
kilometers?
9. In what way is the track to be made so that 16 turns is required for a distance
of 10000 metres in a competition of race?
10. What will be the measurement in metres if 24 turns are completed in a
circular track of 125 metres?
11 Starting from the same place Moula covers a distance of 16 kilometres in 15

85

12.
13.
14.
15.
16.
17.

minutes and Mostafa covers a distance of 1 km 500 metres in 15 minutes.


What will be the distance between them after 15 minutes?
A bus runs 325 km. in 6 hours. What is the speed of the bus in kilometres per
hour?
Raihan produces 400 kg. of paddy from one acre of land. If he gets 700
grammes of rice out of 1 kg of paddy, then how much quantity of rice did he
get?
If 400 metric tons of potatoes are produced in 15 acres of land; then how
much quantity of potatoes will be produced in 1 acre of land?
If 20 kg 850 gm 350 milligramme of mustard are produced in a piece of land,
then how much quantity of mustard will be produced in such 7 pieces of land?
In each day 5000 bags of cement are manufactured in a cement factory. If the
weight of each bag of cement is 25 gm 30 centigramme, then what is the daily
production of cement?
If the yearly production of rod in a steel mill is 300000 metric tons, what is
the daily production of rod?
3

18. A businessman sells in a certain day 18 kg 300 gm of rice and 54 kg. of salt.
How much quantities of rice and salt does he sell in a month on the basis of
the previous quantities?
19. A businessman has 500 metric tons of rice in his godown. He brings daily 2
metric tons 500 kg. of rice to his shop from the godown. How many days will
be required to bring all of rice from the godown?
20. If a bus runs 125 km. by 80 litres of diesel, then how much quantity of diesel
will be required for running 1 km.?
21. If a motor car runs 128 km. by 9 litres of petrol, then how much quantity of
petrol will be necessary to run 1 kilometre?
22. A bus runs 16 km. by 4 litres of diesel. How much quantity of diesel will be
necessary to run 64 kilometres?
23. The length of a rectangular garden is 8 metres and its breadth is 4 metres.
What is its area in square centimetres?
24. The area of a rectangular region is 900 square metres and its length is 36
metres. What is the breadth of the region?
25. The length of a rectangular garden is 32 metres and its breadth is 24 metres. A
path of 2 metres wide passes around the garden inside. What is the area of the
path?

86

26. The length of a pond is 60 metres and its breadth is 40 metres. If the wide of
the bank of the pond is 3 metres, them find the area of the bank of the pond.
27. The area of a rectangular region is 10 acres and its length is 4 times of its
breadth. What is the length in metres of the region?
1

28. The length of a rectangular room is 12 times of its breadth. If its area is 216
square metres, then what is its perimeter?
29. If the base of a triangular region is 20 metres and its height is 15 metres 50
centimeters, then find its area.
30. The length of a rectangular region is 48 metres and its breadth is 32 metres 80
centimetres. A path of 3 metres wide passes around the region outside. What
is the area of the path?
31. The length of a square region is 200 metres and a path of 4 metres wide passes
around it outside. What is the area of the path?
32. The area of a triangular region is 216 square metres. If its base is 18 metres,
then find its height.
33. A reservoir contains 8000 litres of water. If its depth is 256 metres and
breadth is 125 metres, then what is its length?
34. Gold is 193 times heavier than water. The length of a rectangular golden bar
is 88 centimetres, breadth 64 centimetres and height 25 centimetres. What is
the weight of the golden bar?
35. The length of a small box is 15 centimetres 24 millimetres, its breadth is 7
centimetre 62 millimetres and height is 5 centimetres 8 millimetres. What is
the volume in cubic centimetres of the box?
36. The length of a room is 20 metres, breadth 15 metres 50 centimetres and
height 4 metres. What is the volume of the room ?
37. Iron is 75 times heavier than water. The length of a piece of iron sheet is 3
metres, wide 2 metres and the thickness 1 centimetre. What is the weight of
the iron sheet?
38. The length of a reservoir is 5 metres, its breadth is 4 metres and its height is 3
metres. If it is fully filled in by water, how much quantity of water in litres
will contain the reservoir? How much is the weight of that water?
Multiple Choice Questions [Marks () on the correct answer]
1.
Which one of the following is the basic unit of length measurement?
(a) metre
(b) centimetre
(c) kilometre
(d) millimetre

87

2.
3.

4.

5.
6.
7.

The volume of a solid is(a) area


(b) square unit
(c) cubic unit
(d) cubic metre
There is a road around the boundary outside of a rectangular field.
The field is 50 metres in length, 40 metres in breadth and the road
is 2 metres in width. What is the length of the field including the
road?
(a) 54
(b) 50
(c) 46
(d) 44
Observe the following informations:
(i) 1 kilometre = 100 metre
(ii) 100 kilogram = 1 quintal
(iii) mass of 1 liter water = 1 kg.
Which one of the following is correct in view of the above
informations?
(a) i
(b) i and iii
(c) ii and iii
(d) i, ii and iii
The length and breadth of a copper sheet are 4 metre and 3 metre
respectively.
Answer the questions (5-7) in view of the above informations:
What is the area in square metre of the sheet?
(a) 6
(b) 7
(c) 12
(d) 14
What is the perimeter in metres of the sheet?
(a) 7
(b) 12
(c) 14
(d) 24
The length of the diagonal of the sheet is(a) 7 metre
(b) 5 metre
(c) 4 metre
(d) 3 metre.

CREATIVE QUESTIONS
1.

A house is 15 metres in length, 12 metres in breadth. There is a


verandah of width 2 metres around the house.
a) Find the length and breadth of the house including the
verandah.
(b) Find the perimeter of the house and the area of the verandah.
(c) How many tiles would be require to set on a square shape
floor of perimeter equal to the perimeter of that house?

88

2.

3.

A piece of iron-sheet is 3 metres in length, 2 metres in width and


0.01 meter in thickness. Wings of fan would be made by melting
the iron sheet. The length of each wing of each fan is 4 metre,
width 1 metre and thickness is 0.001 metre. There are three wings
in each fan. The price of one fan is Tk. 10.25. (The mass of 5
cubic cm water is 5 gm and that of iron is 0.037 kg).
(a) How many times is iron heavier than water?
(b) Find the mass of the iron sheet.
(c) How many fan and wing of fan can be made using the ironsheet? Find the price of the fans.
The length is twice of the width of a rectangular plot and
perimeter is 120 metres. There is a pond with bank of equal
breadth in side the plot. The breadth of bank is 2 metres. The cost
of digging the pond is Tk. 25 per cubic metre and the depth of the
pond is 5 metre.
(a) Find the width of the plot.
(b) Find the perimeter and area of the pond.
(c) How much would be expend to dig the pond and how much
kilogramme of water does the pond contain in the pond?

N.M.G. -13

Chapter IV

Statistics
4.1. Preliminary Concept Regarding Statistics
Statistics is a informative science, some statisticians define it as a numerical
informations, some others have defined it as the science for research with
numbers. According to the first meaning statistics are numerical states i. e.
informations of any fact or matter. For example, statistics of characteristics of
birth and death regarding population, price of goods, wages etc. are denoted by
numerical information's. Numerical index of such a definite characteristic or
subject is called statistics of that fact. The informations expressed by numbers is
called data.
Statistics means the collection, classification, presentation, analysis and
interpretation of information's systematically. With the help of it some light on any
sphere of enquiry can be thrown by analysing a data.
Statistics is a branch of applied mathematics which is applied in collecting and
analysing numerical information's.
4.2. Purpose of Statistics: The main purpose of statistics is the research on
numerical information's for implementation of a plan or making decision, policy
and formation of procedure, method for any uncertain matter. Statistics presents a
large number of information's in brief and in easily understandable way and also
help in comparison the information among many characteristics. It helps to take
decisions in taking managerial plan of different social, economical and
commercial institutions. In statistics death and birth, import and export,
production, wages, census, agricultural census and management of population etc.
are discussed. It is used for states relationship income and expenditure, man power
and bank insurance and industry in the commercial and economical sectors etc.
The task of statistics is the explanation to make decisions for the future by
induction and application of necessary formulae obtained on the basis of past
experiences and information's regarding any matter.
4.3. Characteristics of Statistics
Information expressed by one number only can not be called statistics. But this can
be named for some numerical information's. Data of statistics are expressed in
numbers.
The data are collected by statistical investigations. In collecting data measurement,
observation and enumeration are necessary. Measurement and enumeration will
their proper units are to be clearly determined so that there is no confusion in data.

90

The unit of enumeration is as the base of collection of informations. The units


should be necessary of the same kind and measurement. If there is no similarity,
comparisons of information can not be judged and interpreted.
4.4. Data of Statistics
The data are collected in two ways: by direct observation or, from data collected
by other institutions for their use or used data of other institutions.
The ages or incomes of the inhabitants of certain cities, the maximum and the
minimum temparatures of some places or the quantity of rain fall of any place in
day or the daily income of a commercial institution etc. are those data when
expressed in numbers. Data collected at random generally called non-arranged
data. When the maximum and the minimum temparatures of each day or the
quantity of rain fall of each day in a place were collected for some days, such
collections are called non-arranged data.
For proper presentation of data collected in such a way are classified conveniently.
Data obtained by classification are called arranged data.
To make classes of the same value a convenient number is taken as the lower limit
of first class. Similarly, a convenient number is selected for determination of the
upper limit. The number thus determined may not belong to informations. The
difference between the number of upper limit and the number of lower limit in
each class is called the class interval of that class. In classification the class
intervals of all the classes are equal to one another. The class interval is obtained
by dividing that number which is equal to the number of classes as found from the
difference between the upper limit of the highest class and the lower limit of first
class.
4.5. Non-arranged Data and Arranged Data
Concept regarding non-arranged and arranged data is given by the example below:
Obtained the marks in Mathematics of 50 students in the annual examination
(Marks are given according to roll numbers)
7
18 37
53
24 39 41 23
64
67
68
50
93
43 11 27
68
72 19 12 21
19
32
75
52
84
15 11 23
19
52 29 92 79
45
81
63
36
21
33 53 8
41
14 26 26 33
49
40
19.
In the above way obtained marks are non arranged data.
If the numbers from 1 to 100 are divided into 10 classes, then the classes will be as
follows-

91

l - 10, 11 - 20, 21 - 30, 31- 40, 41 - 50, 51 - 60, 61- 70, 71- 80, 81- 90, 91- 100.
On the basis of the marks obtained by these 50 students, the number in each class
of students who have got the marks included in that class can be known. The
number of students belonging to a particular class is called frequency.
Table - 1
Classification of marks

Number of students

1 - 10

11 - 20

10

21 - 30

31 - 40

41 - 50

51 - 60

61 - 70

71 - 80

81 - 90

91 - 100

2
Total = 50

In such a way the table formed by showing the obtained number of students in
each class along with the classification of marks is the arranged data.
4.6 . Rules to convert non-arranged data into arranged data
The obtained marks of 50 students in Mathematics in the annual examination are
given according to their roll-numbers above. These are to be converted into
arranged data. In this example, the lowest mark is 7 and the highest mark is 93. So
all the marks will be divided into 18 classes if in each class there consists of 5
numbers beginning from 4. At first classes of numbers are written at the left side
The given marks are considered one by one. The first mark is 7. It will remain in
the class of 4 - 8. So a tally sign '|' is placed against that class. The 2nd mark is 18,
so it will remain in the class of 14 - 18. Now a tally sign '|' is placed against that
class. Proceeding this way, if four tally signs are placed in any class, the fifth tally
sign is not to be placed separately after the fourth one rather is placed by a
crossing of 4 tally signs. Then keeping a small space the next tally sign will be
placed after every four tally signs. The fifth one will be placed in the way such as

92

. It is easy to count if the fifth tally-sign is placed in such a way. The numbers
of tally signs placed against a class is the numbers of data included in a class is the
frequency of that class.
Table-2

Class of marks

Tally-sign

Frequency

4-8

9 - 13

14 - 18

19 - 23

24 - 28

29 - 33

34 - 38

39 - 43

44 - 48

49 - 53

54 - 58

59 - 63

64 - 68

69 - 73

74 - 78

79 - 83

84 - 88

89 - 93

2
Total = 50

4.7. Mean of nonarranged marks


Sum of marks
2027
Mean of obtained marks = number of students = 50 = 4054
This is the mean of non-arranged marks

93

4.8. Mean of classified marks


In order to find the mean of classified marks first the mid value of each class is to
be found out. The frequency of each class indicates the repeated number of the
mid value of that class. Finding out the mid value of each class, it is to be
multiplid by the frequency of that corresponding class. Dividing the sum of these
products by the total number of students, the mean of these classified marks will
be found.
Table - 3

Class of marks

Mid value of
class

Frequency

Mid value of class


frequency

4-8

12

9 - 13

11

33

14 - 18

16

48

19 - 23

21

168

24 - 28

26

104

29 - 33

31

124

34 - 38

36

72

39 - 43

41

205

44 - 48

46

46

49 - 53

51

306

54 - 58

56

59 - 63

61

61

64 - 68

66

264

69 - 73

71

71

74 - 78

76

76

79 - 83

81

162

84 - 88

86

86

89 - 93

91

182

Total = 50

2020

94

2020
So, mean of classified marks = 50 = 404
Mean of classified data =

Sum of the products of mid value of class and frequencies


Sum of class frequencies

Remark : Some informations have been lost for classifying the given 50 marks in
the manner as done above. It is known only from the classified data in which class
do a definite number of students get marks. It is not known how much are the
marks exactly. In classification the frequency for a particular number among the
included numbers of a class is not also known. On the other hand from the
classified data the class at which the highest number of students has obtained
marks is known.
Let us find the mean of data given in Table -1

Table-4
Class-division

Mid-value of
class

Frequency

Mid-value of class
frequency

1 - 10

5.5

11

11 - 20

15.5

10

155

21 - 30

25.5

229.5

31 - 40

35.5

248.5

41 - 50

45.5

273

51 - 60

55.5

222

61 - 70

65.5

327.5

71 - 80

75.5

226.5

81 - 90

85.5

171

91 - 100

95.5

191

Total = 50

2055

2055
Therefore, Mean of classified marks as stated above = 50 = 411.
Remark : The data of Table - 3 and Table - 4 has been formed from non-arranged
marks obtained by 50 students. The actual mean of the marks obtained by 50
students is 40. 54. But from Table - 3 the obtained mean is 404 and from Table - 4

95

the obtained mean is 41.1. As much larges the divisions of classes are the mean of
classified marks will be such closer to the actual mean; that is, as much smaller the
class interval the obtained mean in classification will be such nearer to the actual
mean.
4.9. Determination of mean of classified data by easy method
The class in which mean may belong is to be assumed. It is assumed that the mean
belongs to the class of 51- 60 in Table - 4. This class is situated almost at the
middle of the table . A separate column is to be constructed in the table stated
above and at the right side of this class 0 is to be written in the new column. Then
from the middle of that class in the right side -1,-2, -3, ............ are to be written
successively for the lower classes and 1, 2, 3, ........... are to be written successively
for the upper classes in the new column. These numbers are called deviation
numbers. Then the frequency of each class is to be multiplied by the related
deviation-number. Then the sum of such products are to be determined. This sum
is to be divided by the sum of frequencies and its result is multiplied by class
interval. This product is to be added with the mid value of the assumed class. This
will be the mean.
Table - 5
Class-division Mid-value of
class

Frequency

Deviation
number

Frequency
deviation
number

1 - 10

5. 5

-5

- 10

11 - 20

15. 5

10

-4

- 40

21 - 30

25. 5

-3

- 27

31 - 40

35. 5

-2

- 14

41 - 50

45. 5

-1

-6

51 - 60

55. 5

61 - 70

65. 5

71 - 80

75. 5

81 - 90

85. 5

91 - 100

95. 5

Total = 50

Sum = 72

96

Here, the class interval =10


Therefore, mean = the mid-value of assumed class +
Sum of (frequency deviation number)
Class interval
Total frequencies
72
= 55. 5 + 50 10
= 55 . 5 14. 4 = 41.1
The mean of classified data
= mid-value of assumed class +

(frequency deviation number)


Class interval.
Total frequencies

Remarks : (a) This is equal to the obtained mean from Table - 4


(b) Whatever may be the class at which mean is assumed to be situated, the mean
will always be the same.
(c) The sum is expressed by the symbol, ''
Example 1 .
The marks in Mathematics in the annual examination obtained by 50 students of
any class are given below :
68, 87, 81, 72, 65, 69, 73, 84, 73, 48, 56, 72, 62, 93, 73, 83, 75, 56, 58, 87, 97, 81,
51, 61, 53, 72, 62, 79, 88, 47, 78, 76, 42, 79, 70, 58, 71, 67, 80, 78 , 85, 66, 76, 45,
66, 91, 77, 64, 71, 74.
(a) What is the mean of marks?
(b) Find the mean by classifying data.
Solution : (a) The sum of given marks = 3540
and the number of marks = 50
3540
The required mean = 50 = 708

Ans : 708.
(b) The highest mark of the given marks is 97 and the lowest mark is 42. So
beginning from 40 classes are formed taking class interval of 5 marks. The marks
are classified by tally-signs.
N.M.G. -14

97

Class division

Tally

Frequency

Deviation
number

40 - 44

-6

Frequency
deviation number
-6

45 - 49

-5

- 15

50 - 54

-4

-8

55 - 59

-3

- 12

60 - 64

-2

-8

65 - 69

-1

-6

70 - 74

10

75 - 79

80 - 84

10

85 - 89

12

90 - 94

95 - 99

Total = 50

Sum = - 12

Suppose, the mean is in the class of 70 - 74. The mid value of this class is 72.
- 12
The required mean = 72 + 50 5 = 72 - 1.2 = 70.8.
Ans: 70.8.
Remark : Here the mean of the principal data is equal to the mean of classified
data. But in a rare occasion this equality found. If the class interval is changed,
there will be no equality between two means.
4. 10. Algebraic Formulae
(a) Let, n numbers of expressions or numbers (data) be given and those are x1, x2,
............, xn (in short xi, i = 1, 2, 3, ..............., n). The mean of these terms is
expressed by x . For indication of sum, the sign '' is used.
i, e. x1 + x2 + .................. + xn = xi , (i = 1, 2, ..........., n)
x

x = n i , (i = 1, 2 ........, n) (formula - 1)

This formula is directly used for finding the mean.

98

(b) Let, n terms such as x1, x2, ..............., xn be given. Suppose, each term of them
occurs f1, f2, ......., fn times respectively.
(c) If those are classified, the class from which it begins is called first class, the
next one is second class, the class next to the 2nd is the 3rd class. In such a way
classes can be called 1st, 2nd , 3rd, ..........., nth class (if there are n terms).
In statistics frequency is generally expressed by f. The frequency of i - th class is
denoted by fi. The mid value of i - th class is denoted by xi and x expresses the
mean of all the data.
fixi

x =
, (i = 1, 2 ........., n) (formula - 2)
fi
(d) Algebraic formula for finding the mean by easy method :
(1) Let, the actual mean of n terms such as x1, x2, x3 ,.............., xn be x and its
assumed mean be a.
1

Suppose, x11 = xi - a, x 2 = x2 - a, ............, x n = xn - a


fixi

x = a + n , (i = 1, 2 .............. n) (formula- 3)
(2) The actual mean of classified data is x . Mid values of 1st, 2nd , 3rd, .............,
n-th classes are respectively x1, x2, x3, ............, xn . Let, the mid value of r -th class
be the assumed mean. Therefore, xr is the assumed mean.
Let, d be the class-interval.
x1ifi

x = xr +
d (i = 1, 2,------, n)
fi
Example 2.
Marks obtained by 60 students are given below. Find the mean of their marks.
Marks

Number of Students

52

10

57

15

60

14

62

12

65

99

Solution :

Marks (xi)

Number of students (fi)

fixi

52

10

520

57

15

855

60

14

840

62

12

744

65

585

Total = 60

3544

3544
The required mean = 60 = 5907 (approximately) ( Applying formula - 2)
Second method:
Let the assumed mean be 60.

Marks
xi

Number of
Students fi

x i

fi xi

52

10

80

57

15

45

60

14

62

12

24

65

45

Total = 60

56

56
The required mean = 60 + 60 = 60 093 = 5907 (approximately) (Applying
formula - 3)
Ans : 5907 (Approx.)
4.11. Uses of Arithmetic Mean
The arithmetic mean is used in our daily life in the fields of our income,
expenditure, production, rate of pass in the examination etc.
This mean is widely used in social, economical and commercial fields. The use of
this is the highest of all.

100

EXERCISE 4.1

1. The daily wages (in Taka) of 40 workers are given below:


42, 49, 38, 50, 51, 44, 56, 53, 45, 43, 46, 42, 53, 54, 50, 46, 43, 36, 48, 56, 62,
49, 39, 47, 46, 54, 64, 40, 41, 74, 49, 50, 49, 57, 52, 49, 53, 48, 52 and 56.
(a) Find the mean of daily wages (in Taka) of the workers.
(b) Construct a table by taking a convenient class- interval and from the table find
the mean of daily wages (in Taka).
2. Marks in Statistics obtained by 25 students in the annual examination are given
below :
78, 72, 85, 78, 84, 69, 75, 88, 67, 80, 74, 77, 79, 69, 74, 83, 73, 65, 75, 63, 69,
75, 86, 66, 71.
(a) Find the mean of the above marks by dividing them into 5 convenient classes.
(b) Find the difference between the above mean and the mean obtained directly.
3. The weekly income of 120 families are given below :
Weekly income (in Taka)
251 500
501 750
751 1000
1001 1250
1251 1500

Number of families
22
25
40
23
10

Find the mean of weekly income of the families.


4. Marks obtained by 50 students in an examination are given below :
40, 46, 48, 65, 34, 20, 15, 32, 38, 45, 48, 60, 82, 10, 30, 36, 44, 48, 60, 80, 20,
32, 38, 46, 48, 62, 20, 25, 29, 25, 34, 35, 34, 36, 44, 42, 44, 46, 44, 46,48, 46,
50, 68, 50, 70, 60, 75, 60 and 75.
Construct a table of classes with a class interval of 5 and then find the mean of
them. Also calculate their mean directly.

101

5. A table is given below. Find its mean.


Obtained marks
Number of students
6 10
5
11 15
17
16 20
30
21 25
38
26 30
35
31 35
10
36 40
7
41 45
3
6. Find the mean from the informations given below :
Weight (kg)
20 30 40 50 60 70 80 90
Number of persons
5 14 27 16 13 3 1 1
7. Find the mean from the following table :
House rent (in Tk. - hundred)
Number of families
10 19
10
20 29
17
30 39
30
40 49
50
50 59
29
60 69
9
70 79
5
8. Find the mean from the following table :
Daily sales
2210 2215 2220 2225 2230 2235 2240 2245 2250
(in Taka)
Number of
2
3
5
7
6
5
5
4
3
Shops
4.12. Concept Regarding Measures of Central Tendency
If informations are presented by classification, then it is seen that the frequencies
of central classes are larger; i. e. numbers in a row of class show tendency to be
accumulated to a mid-value and most of the numbers of this row are deviated from
the mid-value by a regular and balanced way. The tendency to be accumulated to
the same value is called the central tendency.
It can be said easily that tendency for values of most of the data to be accumulated
to the middle position roughly is called the central tendency.

102

If it is wanted to know from a student about his monthly expenditure, generally he


will give reply by a whole number. In reality his expenditur in each month is not
the same. But his monthly expenditure remains close to that number. Thus the
expenditure is presented by median value or central value, this tendency is konwn
as central tendency.
The purpose of measures of central tendency is to ascertain the middle place.
Mean, median and mode are the measures of central tendency. But these have the
common use. In addition to these there are other measures of central tendency. But
common uses are not very large. But in some specific cases these may be very
useful.
We have discussed earlier elaborately about mean. So those are not repeated.
4.13. Median
If the given data are arranged according to their values successively (from lower to
higher or from higher to lower) the value which divides the data into two equal
parts is called the median of the data.
The median is then value of the middle term when the data are arranged
successively, that is the term which remains at the middle.
If the number of data be odd, then value of the middle term is the median. Again if
the number of data be even, then the median will be the mean of the quantites of
two middle terms. The median of 7 numbers such as 4, 5, 7, 8, 10, 12 and 13 is 8.
Here the fourth term is the middle term.
The median of 8 numbers such as 4, 5, 7, 8, 10, 13, 16, and 19 is

8 + 10
or, 9. Here
2

the mean of the 4th and 5th terms. If n numbers of data are given, then the median
n
n+1
n
will be 2 th term, if n is odd or, the mean of 2 th and 2 + 1 th terms, if n is

even.
Example 1. Find the median of numbers such as 5, 7, 8, 19, 15, 17, 9, 12, 2, 20,
13, 4 and 18.
Solution : The numbers are arranged according to their values successively from
lower to higher as below :
2, 4, 5, 7, 8, 9, 12, 13, 15, 17, 18, 19 and 20.
Here, n = 13 (an odd number)
13 + 1
median = the value of 2 th term = the value of 7th term = 12.

Ans : 12.

103

Example 2. Find the median of the numbers. 12, 9, 15, 5, 20, 8, 25, 17, 21, 23, 13
and 11.
Solution : The numbers are arranged according to their values successively from
lower to higher as below :
5, 8, 9, 11, 12 , 13 , 15 , 17 , 20 , 21 , 23 and 25. Here, n = 12 (an even number)
Sum of values of the two middle terms
median =
2
12
12

Sum of values of 2 th and 2 + 1 th terms

=
2
Sum of values of 6th and 7th terms 13 + 15 28
=
= 2
= 2 = 14.
2

Ans: 14.
Example 3. The marks distribution of 50 students are given below. Find the
median of their marks.
Obtained marks
Number of students
40
8
50
13
56
10
60
8
68
7
70
3
75
1
Solution : The table for finding out the median :
Obtained marks
Number of students
Cumulative frequency
( numbers)
(frequency)
40
8
8
50
13
21
56
10
31
60
8
39
68
7
46
70
3
49
75
1
50
n = 50
Here , n = 50 (an even number)
/

"

104

50
50

Sum of the values of 2 th and 2 + 1 th terms

median =
2
=

Sum of the values of 25 th and 26th terms 56 + 56


=
= 56.
2
2

Ans : 56
Remark : Here, value of each from 22nd to 31st term is 56.
4.14. Determination of the Median of Classified Data
n
The value of 2 th term of the classified data is the median. The lower limit of that
class which contains the median is L and the cumulative frequency of the previous
class is Fc. The frequency of the class which contains the median is fm and its class
interval is d.

So, the formula for determination of median is as follows :


d
n

median = L + 2 Fc f

m
Example 4. Find the median from the informations given below :

N.M.G. -15

Daily wages

Number of workers

30 35

36 41

10

42 47

18

48 53

25

54 59

60 65

105

Solution :

Daily wages (Class)

Number of workers
(frequency)

Cumulative frequency

30 - 35

36 - 41

10

13

42 - 47

18

31

48 - 53

25

56

54 - 59

64

60 - 65

70

n = 70
n 70
Here, n = 70 2 = 2 = 35. So, value of 35 th term is the median.
35 th term is situated in the class of (48 - 53). So, the median is situated in the
class of (48 - 53).
Here, L = 48 , Fc = 31, fm= 25 and d = 6.
d
n

median = L + 2 - Fc f

m
6
46
24
= 48 + (35 - 31) 25 = 48 + 25 = 48 + 25 = 48 + 096 = 4896.
Ans : 4896
4.15. Uses of Median

Median is used widely in case of measures of attributive characteristics. Very


useful measurements with the help of median are found from those information
which can not be expressed in numbers. Uses of the median is mentionable in
cases of abnormally high and low values of informations.
4. 16. Mode
The number of the data which appears for the highest number of times is called the
mode. When 5, 7, 9, 10, 15, 9, 7, 9, 11 and 6 are arranged systemetically from
lower to higher it becomes 5, 6, 7, 7, 9, 9, 9, 10, 11 and 15. Here 9 appears for the
highest number of times that is 3 times. So the mode of the given data (numbers)
is 9.

106

Example 5. Marks in Mathematics obtained by 40 students in the annual


examination are given below. Find the mode of the data.
42, 31, 45, 27, 60, 61, 39, 41, 35, 58, 29, 53, 48, 39, 52, 38, 40, 47, 28, 51, 49, 78,
90, 52, 48, 36, 52, 39, 71, 64, 32, 49, 56, 33, 48, 33, 25, 37, 48 and 29.
Solution : The data are arranged systemetically from lower to higher as below :
25, 27, 28, 29, 29, 31, 32, 33, 33, 35, 36, 37, 38, 39, 39, 39, 40, 41, 42, 45, 47, 48,
48, 48, 48, 49, 49, 51, 52, 52, 52, 53, 56, 58, 60, 61, 64, 71, 78, 90.
Here, 29 appears for 2 times
33
" " 2 "
39
" " 3 "
48
" " 4 "
49
" " 2 "
52
" " 3 " and each of the rest numbers appears once.
Here, 48 appears for the highest number of times, so the required mode is 48.
Ans : 48.
4.17. Determination of the mode of classified data
The lower limit of mode-class is L, the difference of frequency between the mode
- class and its previous (lower) class is f1, the difference of frequency between the
mode-class and its next (higher) class is f2 and the class interval of mode-class is
d.
The formula for determination of the mode :
f1

mode = L + f + f d
1 2
Example 6. Find the mode from the data given below :
Daily savings (in taka)
Number of workers
25 30
7
31 36
21
37 42
47
43 48
62
49 54
37
55 60
16
61 66
5

107

Solution : In the given table the highest number of workers is in the class of
(43 - 48). So, the mode is in this class.
Here, L = 43, f1 = 62 47 =15, f2 = 62 37 = 25 and d = 6.

Now applying the formula, we get


f1
mode = L + f + f d
1 2
15
= 43 + 15 + 25 6
90
= 43 + 40 = 43 + 2 25 = 45.25
The required mode =Tk.4525
Ans : Tk. 4525.
Remark : (a) If any one of the numbers of the data does not appear more than
once, there is no mode in such a case.
(b) There may be mode more than one in the given data.
(c) If in the classified data the 1st class is the mode-class, then the mode is to be
calculated by taking 0 as the frequency of the previous class. Again if the last class
is the mode-class, then the mode is also to be calculated by taking 0, as the
frequency of the next class.
Example 7. Find the mode of the numbers 2, 7, 5, 4, 6 and 10.
Solution : Here, there is no mode, because no date appears more than once.
Example 8. Find the mode from the data given below.

Class

frequency

11 - 20

25

21 - 30

20

31 - 40

41 - 50

Solution : In the given table the highest number of frequency is in the class of
(11 - 20). So the mode is in this class.

108

Here, L = 11, f1 = 25 - 0 = 25, f2 = 25 - 20 = 5, d = 10.


Now applying the formula, we get,
f1
Mode = L + f + f d
1 2
25
25
= 11 + 25 + 5 10 = 11 + 30 10
25
= 11 + 3 = 11 + 833... = 1933
Ans : 1933
Example 9. Find the mode from the data given below.

Class
6 10
11 15
16 20
21 25

frequency
4
10
15
20

Solution : In the given table the highest number of frequency is in the class of
(21 - 25). So the mode is in this class.
Here, L = 21, f1 = 20 - 15 = 5, f2 = 20 - 0 = 20 and d = 5.
Now applying the formula, we get
f1
Mode = L + f + f d
1 2

5
= 21 + 5 + 20 5
25
= 21 + 25 = 21 + 1 = 22.
Ans : 22
4.18. Uses of Mode
We use mode ordinarily in our every day life. Mode is used in Biology, weather
forecast, etc. Wages, rent, income, demand, supply, etc. are generally determined
by the value of mode.
Its use is remarkable in case of attributive informations.

109

1.
2.

3.

4.

5.

6.

EXERCISE 4.2
Find the mean, median and mode of the following numbers:
30, 12, 22, 17, 27, 25, 20, 24, 19, 2, 23, 32, 26, 29, 35, 21, 11, 28 and 19.
The marks obtained by 40 students in Mathematics in the annual
examination are given below:
53, 48, 65, 35, 55, 38, 50, 67, 25, 58, 62, 21, 76, 46, 57, 67, 55, 70, 44, 72, 60,
42, 64, 73, 38, 41, 55, 41, 34, 51, 40, 38, 28, 44, 36, 40, 42, 35, 22 and 30.
Find the median and mode of the marks. Find the median and mode by
classifying them with class interval of 5.
The daily sales (in taka) of 25 grocer's shops are given below:
75, 63, 69, 86, 71, 66, 75, 65, 73, 80, 83, 74, 69, 79, 77, 69, 74, 85, 72, 78, 84,
69, 75, 88 and 67.
Find the mean, median and mode of the distribution of daily sales.
The daily income (in taka) of 40 masons are given below :
155, 173, 166, 143, 168, 160, 156, 146, 162, 158, 159, 148, 150, 147, 132, 136,
156, 140, 155, 145, 135, 151, 141, 169, 140, 125, 122, 140, 137, 175, 145, 150,
164, 142, 156, 152, 146, 148, 157 and 167.
Find the mean, median and mode of their daily incomes.
Find the mean, median and mode from the table given below :
Weekly savings (in Taka)
Number of workers
71 80
12
81 90
18
91 100
35
101 110
42
111 120
50
121 130
45
131 140
20
141 150
8
Find the mean, median and mode from the table given below :
Daily wages (in Taka)
Number of workers
51 55
7
56 60
25
61 65
76
66 70
32
71 75
17
76 80
12
81 85
3

110

7.

Find the mean, median and mode from the data given below :
Class
Frequency
31 40
4
41 50
6
51 60
8
61 70
12
71 80
9
81 90
7
91 100
4
8. Find the mean, median and mode from the data given below :
Age (in years)
Number of students
56
25
78
27
9 10
28
11 12
31
13 14
29
15 16
28
17 18
22
9. The marks in Mathematics obtained by students according to their roll numbers
in a class of a school are given below:
81, 80, 79, 78, 74, 74, 73, 71, 70, 69, 68, 68, 67, 66, 64, 64, 79, 64, 64, 64, 63,
62, 62, 61, 60, 60, 59, 57, 50, 49, 44 and 40.
What is the mean and median of their marks?
What is the mark which is obtained by the highest number of students?
10. The table of daily wages of 100 workers working in a workshop is given
below.
Daily wages (in Taka)
Number of workers
51 55
6
56 60
20
61 65
30
66 70
15
71 75
11
76 80
8
81 85
6
86 90
4

111

What is the mean and median of daily wages of workers? How much is the
amount of daily wages which is earned by the highest number of workers?
11. The percentages of population according to different age-groups of population
as found in the census of 1991 are given below. Find the mean, median and
mode of their ages.
Age - groups
Population (%)
04
165
59
165
10 14
121
15 19
84
20 24
83
25 29
85
30 34
62
35 39
56
40 44
43
45 49
34
50 54
29
55 59
19
60 64
21
65 69
11
70 74
11
75 79
04
80 84
07
Total = 100
12. In the census of 1981 the population of Bangladesh was 87119965 and of them
males were 44919191 and females were 42200774. In the census of 1991 the
population of Bangladesh become 106314992 and of them males were
54728350 and females were 51586642.
Find the percentage increase in population, percentage increase in males and
percentage increase in females in the census of 1991. Find the average of males
and also females of population as found in two censuses.

112

Multiple Choice Questions [Marks () on the correct answer]


1.
(i) Statistics is an informative science.
(ii) Statistics is a branch of Applied Mathematics.
(iii) Statistics is the information expressed by one number only.
Which one of the following is correct in view of the above
information?
(a) i and ii
(b) i and iii
(c) ii and iii
(d) i, ii and iii
2. By which one of the following does mean class interval mean?
(a) Sum of first and last data among the data.
(b) Sum of greatest data and smallest data of each of the class.
(c) Difference of first and last data.
(d) Difference between the greatest number and smallest number
of each class.
3. Which one of the following is the indicator of number of data in a
class?
(a) frequency of the class
(b) Mid value of class
(c) class interval
(d) Cumulative frequency.
4. Which one of the following is the average of 8,10,11,13 and 12?
(a) 1.8
(b) 1.08
(c) 10.5
(d) 10.8
5. Which one of the following is the median of the numbers 15, 17,14,
and 10?
(a) 13.5
(b) 14
(c) 14.5
(d) 15.5
6. Which one of the following is the mode of 5,11,13,6,13 and 11?
(a) 13 and 5
(b) 11 and 6
(c) 5 and 6
(d) 11 and 13
7. Which one of the following is the formula to find the median?
[Where L=lower limit of median class, n = total number of data,
fm = frequency of median class,
Fc = the cumulative frequency of the preceding class of median
N.M.G. -16

113

class, d= class interval.]


n
2

d
fm

(a) L + + Fc
n
2

(c) L+ Fc

n
2

d
FC

n
2

d
Fc

(b) L+ f m

d
fm

(d) L + + f m

Answer the questions (8-10) in view of the following informations:


Classified marks in mathematics obtained by 40 students of class eight
of Chalita Baria R.D. High school are given below:
class

36-41

42-47

48-53

54-59

60-65

Marks

14

8.

Which one of the following is the class interval of the above


classified data?
(a) 4
(b) 5
(c) 6
(d) 7
9. Which one of the following is the median of the third class interval?
(a) 47.5
(b) 50.5
(d) 51.5
(d) 53.5
10. Which one of the following is the lower limit of the given mode
class?
(a) 60
(b) 54
(c) 48
(d) 42

CREATIVE QUESTIONS
1.

Marks in science obtained by 30 students of class eight in annual


examination of Goga Kaliany Secondary School are given below:
42, 45, 60, 61, 58, 53, 48, 52, 51, 49, 73, 52, 57, 71, 64, 49, 56, 48,
67, 63, 70, 59, 54, 46, 43, 56, 69, 43, 68, 52.
(a) Find the number of class by taking class interval as 5.
(b) Classify the unclassify data taking class interval 5 and find the
cumulative frequency.
(c) Find the average of the classified data obtained in (b) using
algebraic formula.

114

2.

The table of daily wages of 50 labours of the village Isapur are


given below:
Daily income
Number of labour

3.

4.

41-50 51-60 61-70 71-80


6

13

10

81-90

91-100

(a) Construct a table of cumulative frequency.


(b) Find the median from the table.
(c) Find the mode and also find the difference between obtained
from (b) median and mode.
The marks obtained in evaluation Examination of 30 students of
class eight of Sonaton Kati Junior High School are given below:
62, 74, 80, 79, 81, 78, 74, 70, 73, 71, 69, 68, 67, 68, 64, 66, 64, 79,
64, 63, 64, 62, 60, 61, 59, 60, 57, 49, 50, 40.
(a) Find the mean directly.
(b) Find the median of these marks and also the mark which is
obtained by most of the students.
(c) Find the mean constructing a table taking class interval as 5.
The weight of 25 students of class eight of Buruj Bagan Secondary
School are given in kg. below.
56, 46, 62, 46, 58, 48, 41, 54, 51, 47, 51, 40, 55, 63, 57, 47, 53, 54,
51, 43, 52, 53, 46, 53, 56.
(a) Construct the frequency distribution using 40-44, 45-49, 50-54,
55-59, type of class interval.
(b) Find the median of the frequency distribution.
(c) Find the mean and mode of the weights of the students.

JUNIOR SECONDARY MATHEMATICS

ALGEBRA

Chapter I

Algebraic Formulae and their


Application
1.1 Any general law expressed by algebraic symbols is known as algebraic
formula or formula in short. The first four formulae and the corollary related to
them have been elaborately discussed in class VII. Those are discussed below in
short and many examples are given for acquiring knowledge regarding
applications.
1.2. First Four Formulae
Formula 1 : (a + b)2 = a2 + 2ab + b2.
In words, the square of the sum of two quantities is equal to the sum of the squares
of those two quantities and twice their product.
Formula 2 : (a b)2 = a2 2ab + b2.
In words, the square of the difference of two quantities is equal to the result
obtained by twice their product from the sum of the squares of those two
quantities.
Formula 3 : a2 b2 = (a + b) (a b)
In words, difference of squares of two quantities is equal to the product of the sum
and the difference of those two quantities.
Formula 4 : (x + a) (x + b) = x2 + (a + b) x + ab.
In words, if the first term of two binomial expressions is the same, then their
product will be equal to the square of the first term, product of the first term with
the sum of second terms with their usual signs and product of two second terms
with their usual signs.
Corollary 1: a2 + b2 = (a + b)2 2ab.
Corollary 2: a2 + b2 = (a b)2 + 2ab.
Corollary 3 : (a + b)2 = (a b)2 + 4ab.
Corollary 4: (a b)2 = (a + b)2 4ab.
Corollary 5 : 2 (a2 + b2) = (a + b)2 + (a b)2:
Corollary 6: 4ab = (a + b)2 (a b)2.
a + b2 a b2
or, a b = 2 2

117

Square of trinomials : (a + b + c)2 = a2 + b2 + c2 + 2ab + 2bc + 2ca.


Example 1. Find the square of 7a + 9b.
Solution : (7a + 9b)2 = (7a)2 + 2 (7a) (9b) + (9b)2 = 49a2 + 126ab + 81b2
Answer : 49a2 + 126ab + 81b2.
Example 2. Find the square of 4x3 5y2.
Solution : (4x3 5y2)2 = (4x3)2 2 (4x3) (5y2) + (5y2)2 = 16x6 40x3y2 + 25y4.
Answer : 16x6 40x3 y2 + 25y4
Example 3. If a + b = 5 and ab = 6, then find the value of a2 + b2.
Solution: a 2 + b 2 = ( a + b ) 2 2 a b = 5 2 2 . 6 = 2 5 1 2 = 1 3
Answer: 13.
Example 4. If a b = 7 and ab = 44, then find the value of a2 + b2.
Solution : a2 + b2 = (a b)2 + 2ab = 72 + 2.44 = 49 + 88 = 137
Answer: 137.
Example 5. If x y = a and xy = b, then what is the value of (x + y)2 ?
Solution : (x + y)2 = (x y)2 + 4xy = a2 + 4b.
Answer: a2 + 4b.
1
1
Example 6. If x + x = 2, then find the value of x2 + x2 .

1
1 2
1
1 2

Solution : x 2 + x2 = (x) 2 + x = x + x 2x x = 22 2 = 4 2 = 2

Answer: 2.
Example 7. If a = 21 and b = 9, then find the value of 16a2 72ab + 81b2 .
Solution : 16a2 72ab + 81 b2 = (4a)2 2(4a) (9b) + (9b)2
= (4a 9b)2 = (4.21 9.9)2 = (84 81)2 = 32 = 9.
Answer: 9
Example 8. Find the square of 2a 3b 4c.
Solution : (2a 3b 4c)2 = {2a (3b + 4c)}2
= (2a )2 2 (2a) (3b + 4c) + (3b + 4c)2
= 4a2 12ab 16ac + 9b2 + 24bc + 16c2
= 4a 2 + 9b 2 + 16c 2 12ab + 24bc 16ac.
Answer : 4a2 + 9b2 + 16c2 12ab + 24bc 16ac.

118

Example 9. Find the square of 4x + 5y 7z.


Solution : We know that, (a + b + c)2 = a2 + b2 + c2 + 2ab + 2bc + 2ca
Assuming, a = 4x, b = 5y and c = 7z, we get
(4x + 5y 7z) 2
= (4x) 2 + (5y) 2 + (7z) 2 + 2 (4x) (5y) + 2 (5y) (7z) + 2 (7z) (4x).
= 16x 2 + 25y 2 + 49z 2 + 40xy 70yz 56zx.
Answer : 16x 2 + 25y 2 + 49z 2 + 40xy 70yz 56zx.
Example 10. Simplify :
(3a + 5b + 7c)2 + (7c 4a + 5b)2 2(3a + 5b + 7c) (7c 4a + 5b).
Solution : Let, x = (3a + 5b + 7c) and y = (7c 4a + 5b)
Given expression = x2 + y2 2xy = (x y)2
= {(3a + 5b + 7c) (7c 4a + 5b)}2 [substituting the value of x and y]
= (3a + 5b + 7c 7c + 4a 5b)2
= (7a)2 = 49a2
Answer : 49a2
Example 11. Resolve into factors : (8x2 + 9y3 1lz4)2 (2x2 9y3 + 11z4)2
Solution : We know, a2 b2 = (a + b) (a b).
Given expression
= {(8x2 + 9y3 11z4) + (2x2 9y3 + llz4)} {(8x2 + 9y3 1 lz4) (2x2 9y3 + 11z4)}
= (8x2 + 9y3 11z4 + 2x2 9y3 + llz4) (8x2 + 9y3 llz4 2x2 + 9y3 llz4)
= 10x 2 (6x 2 + 18y 3 22z 4 ) = 20x 2 (3x 2 + 9y 3 llz 4 ).
Answer : 20x2 (3x2 + 9y3 llz4).
Example 12. Find the square of 9999.
Solution : 9999 = 10000 1
(9999) 2 = (10000 1) 2 = (10000) 2 2.10000.1 + (1) 2
= 100000000 20000 + 1 = 99980001.
Answer: 99980001.
Example 13. Express (4x 3y) (6x + 5y) as the difference of two squares.
a + b2 a b2
Solution : We know, ab = 2 2

4x 3y + 6x + 5y2 4x 3y 6x 5y2

(4x 3y) (6x + 5y) =


2
2

119

10x + 2y2 2x 8y2


2
2

= (5x + y)2 ( x 4y)2


= (5x + y)2 {( 1) (x + 4y)}2 = (5x + y)2 (x + 4y)2
Answer: (5x + y)2 (x + 4y)2
Example 14. Find the product of (x 12) and (x + 13) with the help of formula.
Solution : We know, (x + a) (x + b) = x2 + (a + b) x + ab.
(x 12) (x + 13) = x 2 + (12 + 1 3 ) x + (12)(13) = x 2 + x 156.

Answer : x2 + x 156.
Example 15. Find the product of (5x2 16) and (5x2 11).
Solution : We know, (x + a) (x + b) = x2 + (a + b) x + ab.
(5x2 16) (5x2 11) = (5x2)2 + ( 16 11) (5x2) + ( 16) ( 11)

= 25x 4 27.5x 2 + 176 = 25x 4 135x 2 + 176.


Answer : 25x4 135x2 + 176.
Example 16. Find the value of 9467 9467 2533 2533.
Solution : 9467 9467 2533 2533 = (9467)2 (2533)2

= (9467 + 2533) (9467 2533) = 12000 6934 = 83208000.


Answer: 83208000.
EXERCISE 1.1
1. Find the square of following expressions with the help of the formula:
(i) 5a + 7b ; (ii) 7x 9y ; (iii) 4ab + 5bc ; (iv) 6x2y 5xy2 ; (v) 4x3 + 3y4;
(vi) x2 3 ; (vii) x3 + xy ; (viii) 11a 12b; (ix) ax by; (x) x y;
(xi) a + b; (xii) xyz abc ; (xiii) a2x3 b2y4 ; (xiv) a b + c ;
(xv) x2 y2 z2 ; (xvi) x2yz y2zx ; (xvii) xy + yz zx ; (xviii) 301 ;
(xix) 606; (xx) 597; (xxi) 999; (xxii) 1002; (xxiii) 5a 6b 7c;
(xxiv) 7a2 + 8b2 5c2 ; (xxv) a b c d.
2. Simplify:
i) (2a + 3b)2 2 (2a + 3b) ( 3b a) + (3b a)2
ii) (3x2 7y2)2 + 2(3x2 7y2) (3x2 + 7y2) + (3x2 + 7y2)2.
N.M.G. -17

120

iii) (5x 9)2 2(5x 9) (9 + 5x) + (9 + 5x)2.


iv) (2a 4b + 7c)2 + (2a + 4b + 7c)2 2(2a 4b + 7c) (2a + 4b + 7c).
v) (3x2 y2 4z2)2 + 2(3x2 y2 4z2) (6z2 + y2 3x2) + (6z2 + y2 3x2)2.
vi) (5x2 3x 2)2 2(5x2 3x 2) (2 + 5x2 3x) + (2 + 5x2 3x)2.
vii) 4569 4569 + 2 4569 5431 + 5431 5431
3. If a = 4, b = 6 and c = 3, then find the value of 4a2b2 16ab2c + 16b2c2.
4. If x = 4, y = 8 and z = 5, then find the value of
25 (x + y)2 20 (x + y) (y + z) + 4 (y + z)2.
5. If m = 5, n = 7, then find the value of
16 (m2 + n2)2 + 56 (m2 + n2) (3m2 2n2) + 49(3m2 2n2)2.
1

6.

If x x = 3, then find the value of x2 + x2 .

7.

If a + a = 4, then find the value of a4 + a4.

8.

1
1
If a a = m, then show that, a4 + a4 = m4 + 4m2 + 2.

9.

2
1
1
2
If a a = 4, then show that, a + a = 18.

10. If a + b = 8 and ab = 15, then find the value of (a b)2 and a2 + b2.
11. If x y = 7 and xy = 60, then find the value of (x + y)2 and x2 + y2.
12. If x + y = 12 and xy = 27, then find the value of (x y)2 and x2 + y2.
1

13. If x x = 5, then find the value of (x + x )2.

14. If a + b = 13 and a b = 3, then find the value of (2a2 + 2b2) and ab.
15. If x + y = 19 and x y = 11, then find the value of 2x2 + 2y2 and 4xy.
16. Multiply with the help of the formula :
(i) (x + y)(x y);
(ii) (5x + 7y) (5x 7y);
(iii) (7a 11) (7a + 11);
(iv) (a + b c) (a b + c);
(v) (5a + 2b 3c) (5a + 2b + 3c);
(vi) (ax by + cz) (ax + by cz);
2
2
2
2
4
4
(vii) (a + b ) (a b ) (a + b );
(viii) (x + 10) (x 14);
(ix) (3a 10) (3a 5);
(x) (6x + 17) (6x 13);
(xi) (ax + by + 5) (ax + by + 3).
17. Express as the square of the difference of two expressions.
(i) (5a + 2b) (7a + 6b);
(ii) (5x + 13) (5x 13);
(iii) (3x + 5y) (7x 5y);
(iv) (6a + 9b) (7b 8a).

121

1.3 Formula of Cubes and Their Corollaries


Formula 5: (a + b)3 = a3 + 3a2b + 3ab2 + b3
= a3 + b3 + 3ab (a + b)
Proof : (a + b)3 = (a + b) (a + b)2 = (a + b) (a2 + 2ab + b2)
= a(a2 + 2ab + b2) + b(a2 + 2ab + b2)
= a3 + 2a2b + ab2 + a2b + 2ab2 + b3
= a3 + 3a2b + 3ab2 + b3
= a3 + 3ab(a + b) + b 3
= a 3 + b 3 + 3ab (a + b).
Corollary 1. a3 + b3 = (a + b)3 3ab (a + b)
Formula 6 : (a b) 3 = a 3 3a 2 b + 3ab 2 +b
=a 3 b3 3ab (a b)
Proof: (a b) 3 = (a b)(a b) 2 .
= (a b)(a 2 2ab + b 2 )
= a(a 2 2ab + b 2 ) b(a 2 2ab + b 2 )
= a3 2a2b + ab2 a2b + 2ab2 b3
= a3 3a2b + 3ab2 b3
= a3 3ab (a b) b3
= a3 b3 3ab (a b).
Corollary 2. a 3 b 3 = (a b) 3 + 3ab (a b).
Example 1. Find the cube of (2a + 3b)
Solution : (2a + 3b)3 = (2a)3 + 3 (2a)2 (3b) + 3 (2a) (3b)2 + (3b)3
= 8a3 + 3.4a2.3b + 3.2a.9b2 + 27b3
= 8a3 + 36a2b + 54ab2 + 27b3.
Answer : 8a3 + 36a2b + 54ab2 + 27b3.
Example 2. Find the cube of (5a 7b)
Solution : (5a 7b)3 = (5a)3 3 (5a)2 (7b) + 3.(5a) (7b)2 (7b)3
= 125a3 3.25a2. 7b + 3.5a. 49b2 343b3
= 125a3 525a2 b + 735ab2 343b3.
Answer : 125a3 525a2b + 735ab2 343b3

122

Example 3. Find the cube of (5x 4y)


Solution : (5x 4y)3 = (5x)3 3(5x)2 (4y) + 3. (5x) (4y)2 (4y)3
= 125x3 3.25x2.4y + 3. 5x. 16y2 64y3
= 125x3 300x2y + 240xy2 64y3
Answer: 125x3 300x2y + 240xy2 64y3
Example 4. Find the cube of (a + b c.)
Solution : (a + b c)3
= {(a + b) c}3
= (a + b)3 3(a + b)2. c + 3(a + b).c2 c3
= (a3 + 3a2b + 3ab2 + b3) 3 (a2 + 2ab + b2).c + 3(a + b). c2 c3
= a3 + 3a2b + 3ab2 + b3 3a2c 6abc 3b2c + 3ac2 + 3bc2 c3
= a3 + b3 c3 + 3a2b + 3ab2 3a2c + 3ac2 3b2c +3bc2 6abc.
Answer: a 3 + b 3 c 3 + 3a 2 b + 3ab 2 3a 2 c + 3ac 2 3b 2 c + 3bc 2 6abc.
Example 5. Simplify :
(3x + 5y)3 + 3(3x +5y)2 (2x 5y) + 3(3x+5y) (2x 5y)2 + (2x 5y)3.
Solution : Let, 3x + 5y = a and 2x 5y = b.
Given expression = a3 + 3a2b + 3ab2 + b3 = (a + b)3
= {(3x + 5y) + (2x 5y)} 3
= (3x + 5y + 2x 5y) 3
= (5x)3 = 125x3.
Answer : 125x3
Example 6. Simplify : (4x 8y)3 (3x 9y)3 3 (x + y) (4x 8y) ( 3 x 9 y )
Solution : Let, (4x 8y) = a and (3x 9y) = b.
a b = (4x 8y) (3x 9y) = 4x 8y 3x + 9y = x + y.
Now given expression = a3 b3 3 (a b) a.b = a3 b3 3 ab (a b)
= (a b)3 = (x + y)3 [putting the values of (a b)]
= x3 + 3x2y + 3xy2 + y3.
Answer : x3 + 3x2y + 3xy2 + y3.
Example 7. If a + b = 8 and ab = 15, then find the value of a3 + b3.
Solution : a 3 + b 3 = (a + b) 3 3ab (a + b) = 8 3 3. 15.8
= 512 360 = 152.
Answer: 152.

123

Example 8. If x y = 1 and xy = 30, then find the value of x3 y3.


Solution : x3 y3 = (x y)3 + 3xy (x y) = 13 + 3.30.1 = 1 + 90 = 91.
Answer: 91.
Example 9. If x + y = 3, then what is the value of (x3 + y3 + 9xy)
Solution : x3 + y3 + 9xy = x 3 + y3 + 3.3.xy

= x3 + y3 + 3(x + y) xy; [ Q 3 = x + y]
= x3 + y3 + 3xy (x + y) = (x + y) 3 = 33 = 27.
Answer: 27.
Example 10. If x = 2, then find the value of (27x 3 135x 2 + 225x 125)
Solution : Given expression
= (3x)3 3.9x2.5 + 3.3x.52 (5)3 = (3x 5)3
= (3.2 5)3 = (6 5)3 = (l)3 = l
Answer : 1
Example 11. If x = 1, then find the value of 27x3 + 54x2 + 36x + 3
Solution : Given expression = (3x)3 + 3.9x2.2 + 3.3x.22 + (2)3 5
= (3x + 2) 3 5 = (3.1 + 2) 3 5
= 5 3 5 = 125 5 = 120
Answer: 120
1
1
Example 12. If a a = 4, then find the value of a3 a3
1
13
Solution : a3 a3 = a3 a

3
1
1 1

= a a + 3a . a a a = 43 + 3.4 = 64 + 12 = 76

Answer: 76.
1.3.1. Some more Formulae related to Cubes
Formula 7 : a3 + b3 = (a + b) (a2 ab + b2)
Proof : a 3 + b 3 = (a + b) 3 3ab(a + b)
= (a + b) {(a + b) 2 3ab}
= (a + b) (a2 + 2ab + b2 3ab)
= (a + b) (a2 ab + b2)
Conversely, (a + b) (a2 ab + b2) = a3 + b3.

124

Now. (a + b) (a2 ab + b2) = a(a2 ab + b2) + b(a2 ab + b2)


= a3 a2b + ab2 + a2b ab2 + b3
= a3 + b3
The formula can be proved in this way also.
Formula 8 : a3 b3 = (a b) (a2 + ab + b2)
Proof :
a3 b3 = (a b)3 + 3ab (a b)
= (a b) {(a b)2 + 3ab}
= (a b) (a2 2ab + b2 + 3ab).
= (a b) (a2 + ab + b2).
Conversely, (a b) (a2 + ab + b2) = a (a2 + ab + b2) b(a2 + ab + b2)
= a3 + a2b + ab2 a2b ab2 b3
= a3 b3
Example 13. Resolve into factors : 8x3 + 27y3.
Solution : 8x3 + 27y3 = (2x)3 + (3y)3
= (2x + 3y) {(2x)2 (2x) (3y) + (3y)2}
= (2x + 3y) (4x2 6xy + 9y2)
Answer : (2x + 3y) (4x2 6xy + 9y2).
Example 14. Resolve into factors : 64a3 125b3
Solution : 64a3 125b3 = (4a)3 (5b)3
= (4a 5b) {(4a)2 + (4a) (5b) + (5b)2}
= (4a 5b) (16a2 + 20ab + 25b2).
Answer : (4a 5b) (16a2 + 20ab + 25b2).
Example 15. Multiply (49a2 28ab + 16b2) by (7a + 4b)
Solution : (7a + 4b) (49a2 28ab + 16b2)
= (7a + 4b) {(7a)2 (7a) (4b) + (4b)2}
= (7a)3 + (4b)3 = 343a3 + 64b3
Answer: 343a3 + 64b3.
Example 16. Multiply (9x2 + 6xyz + 4y2z2) by (3x 2yz)
Solution : (3x 2yz) (9x2 + 6xyz + 4y2z2)
= (3x 2yz) {(3x)2 + 3x (2yz) + (2yz)2}
= (3x)3 (2yz)3 = 27x3 8y3z3.
Answer : 27x3 8y3z3.

125

EXERCISE 1.2
1. Find the cube of the following expressions :
(i) 3x + 4y; (ii) x3 + y2 ; (iii) a2b + c2d: (iv) ab + bc; (v) 8x2 + 11y3;
(vi) 7m + 5n; (vii) x + y + z; (viii) x3 + y3; (ix) 2x 5y; (x) p2 q2;
(xi) 11a 12b; (xii) x3 + 2; (xiii) x6 8; (xiv) 2x 3y z; (xv) x2 y2 + z2;
(xvi) a2b b3c; (xvii) xy 2yz; (xviii) a2b2 c2d2; (xix) x3 2y3;
(xx) 7x2 9y2.
2. Simplify:
i. (3a + 5b)3 + 3(3a + 5b)2 (3a 5b) + 3(3a + 5b) (3a 5b)2 + (3a 5b)3;
ii. (x + 2y)3 3(x + 2y)2 (x 2y) + 3(x + 2y) (x 2y)2 (x 2y)3.
iii. (3a 8b)3 (2a 7b)3 3(3a 8b) (2a 7b) (a b);
iv. (7x 6)3 (5x 6)3 6x(7x 6)(5x 6);
v. (x + y z)3 + (x y + z)3 + 6x(x2 y2 z2 + 2yz);
vi. (x y)3 + (x + y)3 + 6x(x2 y2).
3. If a + b = 10 and ab = 21, then find the value of a3 + b3.
4. If 2x + 3y = 13 and xy = 6, then find the value of 8x3 + 2zy3.
5. If x y = 10 and xy = 30, then find the value of x3 y3.
6. If 5a 7b = 14 and ab = 21, then find the value of 125a3 343b3.
7. If x + y = 2, show that, x3 + y3 + 6xy = 8.
8. If a = 3 and b = 2, then find the value of 8a3 + 36a2b + 54ab2 + 27b3.
9. If x = 5, then find the value of x3 12x2 + 48x 64.
10. If a = 7, then find the value of a3 + 6a2 + 12a + 1.
11. If x 2y = 5, then find the value of x3 8y3 30xy.
12. If 3x 2y = 8, then find the value of 27x3 8y3 144xy.
13. If a2 + b2 = c2, then show that, a6 + b6 + 3a2b2c2 = c6.
1
1
14. If a + a = 3, then show that, a3 + a3 = 18.
1
1
15. If x x = 5, then find the value of x3 (x )3.
16. Find the product with the help of formula :
(i) (a2 + b2) (a4 a2b2 + b4) ;
(ii) (x2 + 2) (x4 2x2 + 4) ;
(iii) (2a + 3b) (4a2 6ab + 9b2) ;
(iv) (7a + 4b) (49a2 28ab + 16b2)

126

(v) (4a 3b) (16a2 + 12ab + 9b2);

(vi) (8x 3y) (64x2 + 24xy + 9y2);

(vii) (ax by) (a2x2 + abxy + b2y2);

(viii) (x2 + a) (x4 ax2 + a2);

(ix) (x + a) (x2 ax + a2) (x a) (x2 + ax + a2);


(x) (2a 1) (4a2 + 2a + 1) (8a3 + 1);
(xi) (5a + 3b) (25a2 15ab + 9b2) (125a3 27b3) .
17. Resolve into factors :
(i) a3 + 8: (ii) 8x3 + 216y3; (iii) 8x3 + 343; (iv) 27a3b3 + 64b3c3;
(v) 8a4 + 27ab3; (vi) 64b3 125; (vii) 27a3 8; (viii) 24x3 81y3;
(ix) 56x3 189y3; (x). 729a3 64b3c6.
1.4. Resolution into Factors of quadratic expression
Discussion regarding factors has been made in class VII. It is discussed briefly
here again. When any expression is expressed as the product of two or more
simple expressions, then it is said to have been resolved into factors and each of
such simple expressions is called the factor of the first expression. For example,
when the expression x2 + 5x is resolved into factors we get,
x2 + 5x = x(x + 5), where x and x +5 are the factors.

Rules of resolving expressions into factors are stated below :


(a) Arranging conveniently :
ax cy + cx ay is arranged as, ax + cx ay cy.
Now, ax + cx ay cy = x(a + c) y (a + c) = (a + c) (x y)
(b) Expressing an expression in the form of square :
x2 + 8xy + 16y2 = x2 + 2.x.4y + (4y)2 = (x + 4y)2 = (x + 4y) (x + 4y)
(c) Expressing an expression as the difference of two squares and then
applying the formula a2 b2.

Adding y2 once with and again subtracting y2 from x2 +2xy 2y 1 it becomes


x2 + 2xy + y2 y2 2y 1.
Now, x2 + 2xy + y2 y2 2y 1 = (x2 + 2xy + y2) (y2 + 2y + 1) = (x + y)2 (y+1)2

= (x + y + y + 1)(x + y y 1) = (x + 2y + 1)(x 1).


(d) Applying the formula, x2 + (a + b) x + ab = (x + a) (x + b):
x2 + 5x + 6 = x2 + (3 + 2) x + 3.2 = (x + 3)( x + 2).

127

(e) Expressing the expression in the form of cubes :


x3 + 6x2 +12x + 8 = x3 + 3.x2.2 + 3.x.22 + 23
= (x + 2)3 = (x + 2) (x + 2) (x + 2).
(f) Applying the formulae : a3 + b3 = (a + b)( a2 ab + b2) and
a3 b3 = (a b)(a2 + ab + b2).
8x3 + 27 = (2x)3 + (3)3 = (2x + 3) {(2x)2 (2x)(3) + (3)2}
= (2x + 3)(4x2 6x + 9)
Again, x3 64 = (x)3 (4)3 =(x 4){(x)2 + (x) (4) + (4)2}
= (x 4) (x2 + 4x + 16)

1.4.1 Factors of the Expression in the Form of x2 + px + q.


If the expression of left hand side of the formula, x2 + (a + b) x + ab = (x + a) (x + b)
is compared with the expression, x2 + px + q, then it is found that in both the
expressions three terms occur, the first one being x2 whose coefficient is 1 (one),
the second or middle term contains x whose coefficients are (a + b) and p
respectively and the third term is free from x, where there are ab and q
respectively. x2 + (a + b) x + ab consists of two factors. Therefore, x2 + px + q has
also two factors. Let, two factors of x2 + px + q are (x + a) and (x + b)
Hence, x2 + px + q = (x + a) (x + b) = x2 + (a + b ) x + ab.
Then, p = a + b and q = ab.
Now in order to find the factors of x2 + px + q, q is to be expressed in two such
factors so that their sum (algebraic) is equal to p.
If it is required to resolve x2 + 9x + 20 into factors, then the number 20 is to be
expressed into two such factors whose sum is 9 and the product is 20.
The possible pairs of factors of 20 are (1, 20), (2, 10) and (4, 5). Of them, the sum
of the pair (4, 5) is (4 + 5) = 9 and the product is 4. 5 = 20.
x2 + 9x + 20 = (x + 4) (x + 5). This method is called middle term distribution or
middle term break -up.
Remark : If p and q are both positive, then in order to resolve into factors of the
expression in the form of x2 + px + q, x2 px + q, x2 + px q and x2 px q,
both the factors of q will be of the same sign i. e. both the factors will be positive
or negative since q is positive in the first and second expressions. In this case, if p
is positive, then both the factors of q are positive and if p is negative, then both the
factors of q are negative. In the third and fourth expression, q, is negative i.e.(q)
and hence two factors of q will be of opposite sign and if p is positive, the positive
N.M.G. -18

128

number of two factors will be greater than the absolute value of the negative
number and if p is negative, the absolute value of negative number of two factors
will be greater than the positive number.
Example 1. Resolve into factors : x2 + 18x + 72.
Solution : We have to find two such positive numbers whose product is 72 and
their sum is 18. The possible pairs of factors of 72 are (1, 72), (2,36), (3,24) (4, 18),
(6, 12) and (8, 9). Of them, the sum of numbers of the pair (6, 12) is 6 + 12 = 18.
x2 + 18x + 72 = x2 + 6x + 12x + 72
= x (x + 6) + 12 (x + 6) = (x + 6) (x + 12)
Answer : (x + 6) (x +12).
Example 2. Resolve into factors : x2 23x + 132.
Solution : We have to find two such numbers whose product is 132 and their sum
is 23. Since the sum of two numbers is negative but the product is positive,
therefore both the numbers will be negative.
The possible pairs of factors of 132 are ( 1, 132), ( 2, 66), ( 3, 44),
( 4, 33), ( 6, 22) and ( 11, 12). Of them, the sum of numbers of the pair
(11, 12) is ( 11 12) = 23.
x2 23x + 132
= x2 11x 12x + 132 = x(x 11) 12(x 11)
= (x 11)(x 12).
Answer : (x 11)(x 12).
Example 3. Resolve into factors : x2 + 15x 54.
Solution : We have to find two such numbers whose product is 54 and their sum
is 15. Since the sum of two numbers is positive but their product is negative, hence
of two numbers, the number whose absolute value is greater than that of the other
is positive and that number is negative whose absolute value is smaller than the
other. The possible pairs of factors of ( 54) are ( 1, 54), ( 2, 27), (3, 18),( 6, 9).
Of them, the sum of the numbers of the pair ( 3, 18) is ( 3 + 18) = 15.
x2 + 15x 54 = x2 + 18x 3x 54 = x(x + 18) 3(x +18)
= (x +18) (x 3)
Answer : (x +18)(x 3).

129

Example 4. Resolve into factors : x2 9x 36.


Solution : We have to find two such numbers whose product is ( 36) and their
sum is ( 9). Since the sum of two numbers is negative and their product is
negative, hence of the two numbers, that number is negative whose absolute value
is greater than the other and that number is positive whose absolute value is
smaller than that of the other. The possible pair of factors of ( 36) are (1, 36);
(2, 18), (3, 12), (4, 9) and ( 6, 6). Of them, the sum of numbers of the pair
(3, 12) is ( 12 + 3) = 9.
x2 9x 36 = x2 12x + 3x 36 = x(x 12) + 3(x 12) = (x 12) (x +3).
Answer: (x 12) (x + 3).
Example 5. Resolve into factors : (x2 + 6x)2 + 5(x2 + 6x) 84.
Solution : Let, x2 + 6x = a.
Given expression = a2 + 5a 84.
= a2 + 12a 7a 84 [Since 12( 7) = 84 and 7 + 12 = 5.]
= a (a + 12) 7(a + 12) = (a + 12) (a 7)
= (x2 + 6x + 12) (x2 + 6x 7), [Putting the value of a.]
= (x2 + 6x + 12)(x2 + 7x x 7), [Since (1). 7 = 7 and (7 1) = 6]
= (x2 + 6x + 12) {x(x + 7) l(x + 7)} = (x2 + 6x + 12)(x + 7)(x 1).
Answer : (x2 + 6x + 12) (x + 7)(x 1).
Remark : x2 + 6x + 12 can not be resolve into factors for any real value of x.
1.4.2. Factors of the Expression in the Form of ax2 + bx + c.
Let, (rx + p )(sx + q) be the factors of the expression ax2 + bx + c.
ax2 + bx + c = (rx + p) (sx + q) = rsx2 + (rq + sp) x + pq.
Then, a = rs, b = rq + ps and c = pq.
Hence, ac = rspq = rq. sp and b = rq + sp
Now to find the factors of ax2 + bx + c, the product of the coefficient of x2 and the
term free from x is to be expressed in two such factors such that their sum is equal
to b, the coefficient of x.
To factorise 2x2 + 9x + 10, (2.10) = 20 is to be expressed into two such factors
whose sum is 9 and their product is 20. The pairs of factors of 20 are (1, 20), (2, 10)
and (4,5). Of them, the sum of the pair (4, 5) is (4+5) = 9 and their product is 4.5 = 20.
2x2 + 9x + 10 = 2x2 + 4x + 5x + 10 = 2x (x + 2) + 5 (x + 2) = (x + 2) (2x + 5).

130

Remark : To factorise ax2 + bx + c, the rules which are followed for different
values of p, q having positive and negative signs of x2 + px + q are also followed
for different values of a, b, c having positive and negative signs. Here b for p and
the product of a and c for q are to be considered.
Example 6. Factorise 8x2 + 18x + 9.
Solution : Here, 8.9 = 72. Now, 6.12 = 72 and 6 + 12 = 18.

Therefore, 8x2 + 18x + 9 = 8x2 + 6x + 12x + 9 = 2x(4x + 3) + 3 (4x + 3)


= (4x + 3) (2x + 3)
Answer : (4x + 3) (2x +3).
Example : 7. Factorise 4x2 23x2 + 33.
Solution : Here, 4.33 = 132.
For the sum of ( 23), we get the pair (11, 12)
where ( 11) + ( 12) = 23 and ( 11) ( 12) = 132.
Therefore, 4x2 23x + 33 = 4x2 1lx 12x + 33 = x(4x 11) 3(4x 11)
= (4x 11) (x 3).
Answer : (4x ll) (x 3).
Example 8. Factorise 9x2 9x 4.
Solution : Here, 9.( 4) = 36. Now, 3.( 12) = 36 and ( 12) + 3 = 9.
9x2 9x 4 = 9x2 + 3x 12x 4 = 3x (3x + 1) 4 (3x + 1).
= (3x + l)(3x 4).
Answer : (3x + 1) (3x 4).
Example 9. Factorise 27x2+15x 2.
Solution : Here, 27. ( 2) = 54. Now, ( 3). 18 = 54 and ( 3 ) + 18 = 15.
27x2 + 15x 2 = 27x2 3x + 18x 2 = 3x(9x 1) + 2(9x 1)
= (9x l) (3x + 2)
Answer : (9x l)(3x + 2).
1.4. 3 H. C .F. and L. C. M. of Algebraic Expressions

The clear concept for finding H. C. F. and L. C. M. of two or three algebraic


expressions having not more than three terms (including arithmetical coefficients)
has already been given in class VII. A brief discussion is made here again.

131

Common Factor : If two terms have the same factor, this is called a common
factor. a is the common factor of a2b, ab and a2c; again (x + y) is the common
factor of (x + y)3, (x + y)2 and x2 y2.
Highest Common Factor (H. C. F.)
The product of common prime factors of two or more expressions is called the
Highest Common Factor or simply H. C. F. of those two or more expressions.
The Highest Common Factor of three expressions, a3b4c3, a5b3c4 and a4b3c5 is
a3b3c3. Again the Highest Common Factbr of three expressions, (x2 + y2)2, (x2 + y2)3
and (x4 y4) is x2 + y2.
Common Multiple
If any expression is completely divisible by two or more expressions, then the
dividend is called the common multiple of those two or more divisors.
The expression, a2b2c is the common multiple of ab, bc, ab2, a2c and b2c.
The expression, (x + y)2 (x y) is the common multiple of three expressions,
(x + y), (x + y)2 and x2 y2.
Lowest Common Multiple (L. C. M.)
Among different multiples of two or more expressions the common multiple
which consists of lowest number of prime factors is called Lowest Common
Multiple or L.C.M. in short.
The expression a2b2c is the L.C. M. of three expressions, a2bc; ab2 and abc.
The expression, (x + y)3 (x y) is the L.C.M. of three expressions,
(x + y)2, ( x + y)3 and (x2 y2).
Example 1. Find the H. C. F. of 6a3b2c, 12a2b2c4 and 15a4b4c2.
Solution : H.C.F. of 6, 12 and 15 = 3 and the common factors with highest
common power among a3b2c, a2b2c4 and a4b4c2 are a2, b2 and c respectively.
Required H. C. F. = 3a2b2c.
Ans : 3a2b2c.
Example 2. Find the H. C. F. of x3 + x2y, x2y + xy2, x3 + y3 and (x + y)3.
Solution : Here,
1st expression = x3 + x2y = x2(x + y)
2nd expression = x2y + xy2 = xy (x + y)
3rd expression = x3 + y3 = (x + y) (x2 xy + y2) and
4th expression = (x + y)3 = (x + y) (x + y) (x + y).

132

Here the common factor of the expressions is (x + y) and this factor with the
highest common power among the expressions is (x + y).
Required H. C. F = ( x + y)
Answer : ( x +y ).
Example 3. What is the L.C.M. of 4x2y2z and 6xy2?
Solution : L.C.M. of 4 and 6 = 12
The highest factor between the given expressions are x2, y2 and z respectively.
The required L. C. M. = 12x2y2z.
Answer :12x2y2z.
Example 4. Find the L. C. M. of 3(a4 + a3b) and a2b + 2ab2 + b3
Solution : Here, 1st expression = 3(a4 + a3b) = 3a3(a + b) and
2nd expression = a2b + 2ab2 + b3 = b (a2 + 2ab + b2) = b (a + b)2
Here the highest powers of the factors a, b and (a + b) are a3, b and (a + b)2
respectively.
Again , the L. C.M. of the coefficients which are natural numbers = 3.
Required L. C. M. = 3a3b(a + b)2.
Answer : 3a3b(a + b)2.
Example 5. Find the L. C. M of 24a2bc, 18ab3c2 and 54a4b2c2.
Solution : L. C. M. of 24, 18 and 54 = 216
The factors, a, b and c with their highest powers among three given expressions
are a4, b3 and c2 respectively.
Required L. C. M. = 216a4b3c2
Answer : 216a4b3c2.
Example 6. Find the L. C. M. of 4(x2 + ax)2, 6(x3 a2x) and 14x3 (x3 a3).
Solution : L. C. M. of 4, 6 and 14 = 84.
Now, part of 1st expression = (x2 + ax)2 = x2(x + a)2
2nd expression = (x3 a2x) = x(x2 a2) = x(x + a)(x a)
and 3rd expression = x3(x3 a3) = x3(x a)(x2 + ax + a2).
Here the factors, x, (x + a), (x a) and (x2 + ax + a2) with their highest powers are
x3, (x + a)2, (x a) and (x2 + ax + a2) respectively.
Required L. C. M. = 84x3(x + a)2 (x a) (x2 + ax + a2)
= 84x3(x + a)2 (x3 a3).

Answer : 84x3(x + a)2 (x3 a3).

133

Example 7. Find the L.C. M. of x3 + y3, x3 y3 and x4 + x2y2 + y4.


Solution: 1st expression = x3 + y3 = (x + y) (x2 xy + y2)
2nd expression = x3 y3 = (x y) (x2 + xy + y2)
3rd expression = x4 + x2y2 + y4
= (x2)2 + 2x2y2 + (y2)2 x2y2 = (x2 + y2)2 (xy)2
= (x2 + xy + y2) (x2 xy + y2)
Required L. C. M. = (x + y) (x y) (x2 + xy + y2) (x2 xy + y2)
= (x3 + y3) (x3 y3) = x6 y6.
Answer : x6 y6.
EXERCISE 1.3
Resolve into factors :
(1) 4x2 y2;
(2) x2 144y2;
(3) 3x 75x3;
(4) ax4 + 64a;
(5) 4a4 + 81 ;
(6) x2 y2 + 2y 1;
(7) x4 6x2 + 1;
(8) x4 + x2 + 1;
(9) a2 2ab + 2b 1;
(10) a4 + a2b2 + b4;
(11) 64x3 8y3;
(12) (x y)3 + z3;
(13) a6 + b6;
(14) a6 b6;
(15) x2 + x 72;
(16) x2 + 18x + 56;
(17) x2 8x 105;
(18) 15 8x + x2;
(19) x2 51x + 650;
(20) 35 2x x2;
(21) x2 + 14x + 40;
(22) a2 + 7ab + 12b2;
(23) x2 + 7x 120;
(24) x2 + 11x 42;
(25) x2 + 3xy 40y2; (26) p2 + 2pq 80q2;
(27) (x2 x)2 + 3 (x2 x) 40;
(28) (a2 + 7a)2 8(a2 + 7a) 180;
(29) (x2 + 6x)2 32 (x2 + 6x) 320 ;
(30) 6x2 x 15;
(31) x2 + ax (3a 2)(4a 2); (32) x2 + (3a + 4b)x + (2a2 + 5ab + 3b2);
(33) (x + 5)(x + 13) 9; (34) 3x2 + 11x 4; (35) 8x2 + 2x 15;
(36) 3x2 16x 12; (37) 2x29x 35; (38) 2a2 + 7ab 15b2;
(39) 9a4 + 2a2b2 32b4; (40) 2(x + y)2 3(x + y) 2;
(41) 15x2 11xy 12y2; (42) 2x2 5xy + 2y2;
(43) 10p2 + 11pq 6q2; (44) 2x2 + 5x 42; (45) ax2 + (a2 + 1) x + a.

Find the H. C. F. of the following :


(46) 15a3b2c4, 25a2b4c3 and 20 a4b3c2;
(47) 36a2b2c4d5, 54a5c2d4 and 90a4b3c2;
(48) 18a3b4c5, 42a4c3d4, 60b3c4d5 and 78a2b4d3;

134

(49)
(50)
(51)
(52)
(53)
(54)
(55)
(56)
(57)

32x4y5z6, 40x5y4z3, 56x2y7z4 and 72x6y2z3;


x3 + x2y, x2y + xy2 and x3 + y3;
x2 3x, x2 9 and x2 4x + 3;
x2 3x + 2, x3 5x2 + 6x and x3 x2 2x;
x2 y2, x3 y3 and (x y)3;
(x + 2)2, (x2 + 2x) and (x2 + 5x + 6);
18 (x + y)3, 24 (x + y)2 and 32 (x2 y2);
a2b (a3 b3), a2b2(a4 + a2b2 + b4) and a3b2 + a2b3 + ab4;
x3 3x2 10x, x3 + 6x2 + 8x and x4 5x3 14x2.

Find the L. C. M. of the following :


(58) 4xy, 9x2z, 3xzy2 and 12x2z3;
(59) 3a2d3, 9d2b2, 12c3d2, 24a3b2 and 36c3d2;
(60) 3x3y2, 4xy3z, 5x4y2z2 and 12xy4z2;
(61) 24b3c2, 15c2a. 30a3b and 40a2bc;
(62) 5a2b3c, 10ab2c3 and 15ab3c;
(63) a3 + b3, (a + b)3, (a2 b2)2 and (a2 ab + b2)2;
(64) x2 + 3x + 2, x2 1 and x2 + x 2;
(65) a3 1, a3 + 1 and a4 + a2 + 1;
(66) (x + 2)2, (x2 + 2x) and (x2 + 5x + 6);
(67) a2b (a3 b3), a2b2 (a4 + a2b2 + b4) and (a3 + b3);
(68) 6x2 x 1, 3x2 + 7x + 2 and 2x2 + 3x 2.
Multiple Choice Questions [Mark () on the correct answer]
1. If a b = 3 and ab = 4, then which one of the following is the value
of a2+b2?
(a) 25
(b) 17
(c) 11
(d) 1
3
3
2
2
2. (i) x y = (xy) (x +xy+y )
2

a+b a b
(ii) ab =

4 4

(iii) x3 + y3 = x3+y3+3xy (x+y)

135

Which one of the following is correct according to the above


informations?
(a) i and ii
(b) ii and iii
(c) i and iii
(d) i, ii and iii
3. Which one of the following is the L.C.M. of x3+x2y, x2y+xy2?
(a) xy
(b) x +y
(c) xy (x+y)
(d) x2y (x + y)
Answer the questions (4 6) on the basis of the following
informations:
If x + y = 8 and x y = 2 then,
4. How much is the value of 2x2 +2y2?
(a) 32
(b) 34
(c) 64
(d) 68
5. Which one of the following is the value of 4xy?
(a) 68
(b) 64
(c) 62
(d) 60
6. Which one of the following is the value of x2 y2?
(a) 16
(b) 10
(c) 8
(d) 4
7. Which one of the following is the product of (x+8) and (x7)?
(b) x2 15x +56
(a) x2 + x56
(d) x2 x +56
(c) x2 +15x56
8. Which one of the following is the H.C.F. of 15 a3b2c4,
25a2b4c3 and 20 a4b3c2?
(ii) 150 a4b4c4
(i) 5a2b2c2
(iv) 5a4b4c4
(iii) 150a2b2c2
Answer the questions (9 11) on the basis of the following informations
If x

1
= 5 then,
x
2

9.

Which one of the following is the value of x + ?


x

(a) 29
(c) 21
N.M.G. -19

(b) 25
(d) 5

136

10. Which one of the following is the sum of x2 and


(a) 27
(c) 23

(b) 25
(d) 10

11. Which one of the following is the value of x 3


(a) 15
(c) 125

1
?
x2

1
?
x3

(b) 110
(d) 140

CREATIVE QUESTIONS
1.

x2+x2, 2x3+12x2+24x+16, 2x2+3x2 and 2x38x are four algebraic


expressions.
(a) Factorise the first expression.
(b) Factorise the 3rd and 4th expressions.
(c) Find the H.C.F. and L.C.M. of the above expressions.

2.

a2 +
(a)
(b)
(c)

3.

b2 and a6 + b6 + 3a2b2c2 are two algebraic expressions.


If a = c and b = c, then find the value of 2nd expression.
If a +b = 5 and a b =1, then find the value of ab and a2 + b2.
If a2 + b2 = c2 and c = 2, then show that the value of 2nd
expression is 64.

1
1
1
, x 2 + 2 and x 4 + 4 are three algebraic expressions.
x
x
x
1
1
(a) If x + =3, then find the value of x 2 + 2
x
x
1
1
(b) If x + = p, then prove that, x 4 + 4 = p4 4p2+2
x
x
1
1
1
(c) If x 4 + 4 = 2, then find the value of x + and x 4 4 .
x
x
x
x+

Chapter II

Algebraic Fraction
2.1 Algebraic Fraction

In class VII the concept of algebraic fraction, reduction of the algebraic fractions
in its lowest terms, making the fractions of common denominator, addition and
subtraction of algebraic fractions have already been given. But here some of them
with examples relating to algebraic fractions have been discussed again.
2.1.1. Algebraic Fraction

p
If p and q are two algebraic expression, then q is an algebraic fraction. Here in the
p
fraction q , p and q are called numerator and denominator of the fraction
respectively, where q o.
x a + b x2 + y2
y , a , x + y etc. are algebraic fractions.
2.1.2. To express the Fraction in its Lowest Terms

If there are common factors in the numerator and denominator of any fraction and
the numerator and denominator are divided by the highest common factor of
numerator and denominator, then the new fraction formed by the quotients of
numerator and denominator is the original fraction in its lowest term.
x4y x2y3 x2y(x2 y2) x2y(x + y)(x y) y(x + y)
= x2
x5 x4y = x4(x y) =
x4(x y)
Here the numerator and denominator have been divided by x2(x y) which is the
highest common factor of numerator and denominator.
2.1.3. To Express with the Common Denominators
To express two or more fractions with the common denominators, L. C. M. of
denominators is to be found out and then this L. C. M. is to be made the
denominators of each of the fractions. At the time of expressing fractions with the
common denominators and to keep the values of the fractions unchanged, the L. C.
M. of the denominators is to be divided by the denominator of each of the
fractions and the corresponding numerator is to be multiplied by the quotient thus
obtained. This product will be the numerator of the corresponding fraction. For

138

x y

example, the L. C. M. of the denominators, a, b, c of a , b and c is abc. If abc is


x y

divided by a, b and c, the quotients are bc, ca and ab respectively. Hence, if a , b


z

and c are expressed as the fractions with the common denominators, then these are
zab
xbc yac
abc , abc and abc respectively.

2.1.4. Addition of Fractions

To add two or more fractions it is required to express the fractions with the
common denominators and then numerators are to be added. The sum of the given
fractions will be a new fraction of which the numerator is the sum of numerators
of the transformed fractions and denominator is the common denominator.
x y z xbc yac zab xbc + yac + zab
For example, a + b + c = abc + abc + abc =
abc
2.1. 5. Subtraction of the Fractions
To determine the difference of two fractions it is required to express two fractions
with the common denominator and then two numerators are to be subtracted. The
difference of two fractions will be a new fraction of which numerator will be the
difference of two numerators and denominator will be the common denominator.
x
y
xc
ya
xc ya
For example, ab bc = abc abc = abc
Remark : In case of addition and subtraction of the algebraic fractions, the
fractions are to be expressed in its lowest terms if necessary.
xpq yrs zuv x y z xbc + yac + zab
For example, apq + brs + cuv = a + b + c =
abc
3 2 4
24a b x
Example 1. Express 56a5bx6 in its lowest terms.
Solution : The common factors with the highest common power of the
expressions, 24a3b2x4 and 56a5bx6 are respectively 23, a3, b and x4.
Hence, the H. C. F. of two expressions is 8a3bx4.
3b.8a3bx4
3b
The given fraction = 7a2x2.8a3bx4 = 7a2x2 [dividing numerator and
denominator by 8a3bx4]

139

3b
Required fraction in its lowest terms = 7a2x2
3b
Answer : 2 2
7a x
x(x2 + 2xy + y2) (x2 xy + y2)
Example 2. Express
in its lowest terms.
(x3 + y3) (x2 y2)
Solution : Resolving numerator and denominator of the fractions into factors, we
get,
x(x2 + 2xy + y2) (x2 xy +y2)
(x3 + y3) (x2 y2)
x(x + y) (x + y) (x2 xy + y2)
= (x + y) (x2 xy + y2) (x + y) (x y)
x(x + y)2 (x2 xy + y2)
= (x + y)2 (x y) (x2 xy + y2)
Now dividing numerator and denominator by (x + y)2 (x2 xy + y2) which is H. C.
F. of numerator and denominator, we get
x
x(x2 + 2xy + y2) (x2 xy + y2)
=
3
3
2
2
xy
(x + y ) (x y )
x
The required fraction in its lowest terms = x y
x

Answer : x y
a
b
c
Example 3. Express the fractions a2 9b2 , a2 + 6ab + 9b2 , a3 27b3 with the
common denominators.
Solution : Here, the denominators are first resolved into factors. Then the L. C. M
of denominations are to be found.
a2 9b2 = (a)2 (3b)2 = (a + 3b) (a 3b);
a2 + 6ab + 9b2 = (a)2 + 2.a.3b + (3b)2 = (a + 3b)2;
a3 27b3 = (a)3 (3b)3 = (a 3b) (a2 + 3ab + 9b2).
Hence, the L. C. M. of denominators = (a + 3b)2 (a 3b) (a2 + 3ab + 9b2)
= (a + 3b)2 (a3 27b3)

140

a
a(a + 3b)2 (a3 27b3)
Therefore, a2 9b2 = (a2 9b2) (a + 3b)2 (a3 27b3)
a(a + 3b) (a + 3b) (a 3b) (a2 + 3ab + 9b2) a(a + 3b) (a2 + 3ab + 9b2)
= (a 3b) (a + 3b) (a + 3b)2 (a3 27b3) = (a + 3b)2 (a3 27b3)
b
b (a + 3b)2 (a3 27b3)
b (a3 27b3)
Again, a2+ 6ab + 9b2 = (a + 3b)2 (a + 3b)2 (a3 27b3) = (a + 3b)2 (a3 27b3)
c
c(a+ 3b)2 (a3 27b3)
c(a + 3b)2
and a3 27b3 = (a3 27b3) (a + 3b)2 (a3 27b3) = (a + 3b)2 (a3 27b3)
a(a + 3b) (a2 + 3ab + 9b2)
b(a3 27b3)
c(a + 3b)2
Answer : (a + 3b)2 (a3 27b3) , (a + 3b)2 (a3 27b3) , (a + 3b)2 (a3 27b3)
1
a
b2
Example 4. Add : a b + a2 + ab + b2 + a3 b3
1
1
a
a
Solution : Here, a b = a b , a2 + ab + b2 = a2 + ab + b2
b2
b2
and a3 b3 = (a b) (a2 + ab + b2)
Therefore, L . C. M. of denominators = (a b) (a2 + ab + b2) = a3 b3.
1
a
b2
Hence, a b + a2 + ab + b2 + a3 b3
a2 + ab + b2 a(a b)
b2
+ 3
= a3 b3
3 + 3
a b3
a b
a2 + ab + b2 + a2 ab + b2 2a2 + 2b2 2(a2 + b2)
=
= a3 b3 = a3 b3
a3 b3
2(a2 + b2)
Answer : a3 b3
2x
x
3x
Example 5. Add : x22x3 + x2 + x12 + x2 + 5x + 4
2x
2x
2x
Solution : Here, x22x3 = x2 3x + x 3 = (x 3) (x + l)
x
x
x
= 2
=
2
x + x12 x + 4x 3x12
(x + 4)(x 3)

141

3x
3x
3x
and x2 + 5x + 4 = x2 + 4x + x + 4 = (x + 4)(x + l)
Now L. C. M. of denominators = (x + 1) (x + 4) (x 3).
2x
x
3x
Hence, x22x3 + x2 + x 12 + x2 + 5x + 4
2x
x
3x
=
+
+
(x 3) (x + l) (x + 4)(x 3) (x + 4) (x + l)
.**

'

2x(x + 4) + x(x + 1) + 3x(x 3) 2x2 + 8x + x2 + x + 3x2 9x


=
=
(x 3) (x + 1) (x + 4)
(x 3) (x + 1) (x + 4)
2
6x
=
(x 3) (x + l) (x + 4)
6x2
Answer : (x 3) (x + l) (x + 4)
x
y
xy
Example 6. Add : x3 + y3 + 3
3 + 4
x + x2y2 + y4
x y
x
x
Solution : Here, x3 + y3 = (x + y)(x2 xy + y2)
y
y
3
3 =
2
x y
(x y) (x + xy + y2)
xy
xy
xy
and x4 + x2y2 + y4 = 4
2
2 2
2 2
4
2 2 =
(x + y ) (xy)2
x + 2x y + y x y
xy
= 2
2
(x + xy + y ) (x2 xy + y2)
Therefore, L. C. M. of denominators
= (x + y) (x y) (x2 + xy + y2) (x2 xy + y2) = (x3 + y3) (x3 y3) = x6 y6
x
y
xy
Hence, x3 + y3 + x3 y3 + x4 + x2y2 + y4
x (x3 y3) y(x3 + y3) xy (x + y) (x y)
=
+
+
x6 y 6
x6y6
x6 y 6
x4 xy3 + yx3 + y4 + x3y xy3 x4 + 2x3y 2xy3 + y4
=
=
x6 y 6
x6 y 6
x4 + 2x3y 2xy3 + y4
Answer:
x6 y 6

142

a
b
c
Example 7. Add : (a b)(c a) + (a b) (b c) + (b c)(c a)
Solution : Here, L.C.M. of denominators = (a b) (b c) (c a)
a
b
c
Hence, (a b) (c a) + (a b) (b c) + (b c) (c a)
a(b c) + b (c a) + c(a b) ab ac + bc ab + ac bc
=
= (a b) (b c) (c a)
(a b)(b c) (c a)
0
= (a b) (b c) (c a) = 0
Ans : 0
2
3
2(5 2x)
Example 8. Add : (x l) (x2) + (x l) (x 3) + (x l)(x 2)(x 3)
Solution : Here, L.C.M. of denominators = (x 1) (x 2) (x 3).
2
3
2(5 2x)
Hence, (x l)(x 2) + (x l)(x 3) + (x l)(x 2)(x 3)
2(x 3) + 3(x 2) + 2(5 2x) 2x 6 + 3x 6 + 10 4x
=
= (x l)(x 2)(x 3)
(x l)(x 2) (x 3)
x2
1
= (x l)(x 2) (x 3) = (x l)(x 3)
[dividing the numerator and denominator by (x 2)]
1
Ans : (x l)(x 3)
a
b
Example 9. Subtract : 6x2yz5 8x3y3z2
Solution : Here, L.C.M. of denominators = 24x3y3z5
a
a.24x3y3z5
4axy2
Now, 6x2yz5 = 6x2yz5.24x3y3z5 = 24x3y3z5
b
b.24x3y3z5
3bz3
and 8x3y3z2 = 8x3y3z2.24x3y3z5 = 24x3y3z5
a
b
4axy2
3bz3
4axy2 3bz3
Hence, 6x2yz5 8x3y3z2 = 24x3y3z5 24x3y3z5 = 24x3y3z5
4axy2 3bz3
Ans: 24x3y3z5

143

a+b
a
Example 10. Subtract: (a b)2 a2 b2
Solution : Here, L.C.M. of denominator = (a b)2 (a + b).
a+b
a
(a + b)2 (a b)a

=
Hence, (a b)2 a2 b2
(a b)2 (a + b)
a2 + 2ab + b2 a2 + ab
3ab + b2
b(3a + b)
=
=
2
2
(a b) (a + b) (a b)2 (a + b)
(a b) (a + b)
b(3a + b)
Ans: (a b)2 (a + b)
1
b
a
Example 11. Simplify : a + b + a2 b2 a2 + b2
Solution : Here L.C.M. of denominators = (a2 + b2) (a2 b2) = a4 b4
1
b
a
Hence, a + b + a2 b2 a2 + b2
(a b) (a2 + b2) + b (a2 + b2) a(a2 b2)
=
a4 b4
a3 + ab2 ba2 b3 + a2b + b3 a3 + ab2
2ab2
=
= a4 b4
a4 b4
Second method : The sum of first two expressions is,
1
b
ab+b
a
+ 2
2 =
2
2 = 2
(a + b) a b
a b
a b2
The third expression is subtracted from this sum,
1
b
a
a
a
a + b + a2 b2 a2 + b2 = a2 b2 a2 + b2
=

a + b a + b
1
a 2b
2ab
1
= aa2 b2 a2 + b2 = a (a2 b2) (a2 + b2) = a4 b4 = a4 b4

2ab2
Ans : a4 b4
1
1
1
1
Example 12. Simplify : x 1 x + 2 + x 2 x + 1
1 1
1
1
Solution : Given expression = x 1 x + 1 + x 2 x + 2

N.M.G. -20

144

x + 1 (x 1) x + 2 (x 2)
+
x2 1
x2 4
2
4
2 (x2 4) + 4(x2 1)
= x2 1 + x2 4 = (x2 1) (x2 4)
2x2 8 + 4x2 4
6x2 12
6(x2 2)
= (x2 1) (x2 4) = (x2 1) (x2 4) = x4 5x2 + 4
6(x2 2)
Ans : x4 5x2 + 4
Remark: Simplification can be done within the shortest time of some expressions
are arranged conveniently and then added or subtracted, afterwards this sum or
difference are added to or subtracted from any other expression or added or
subtracted from the sum or difference of some other expressions.
=

EXERCISE 2.1
1.

2.

Express the following fractions in its lowest terms:


18x3y2z5
49a4b7c9
45a4b3x7
121p3q9r10
(i) 24x5yz7
(ii) 42a5b9c5 (iii) 36ab6x9 (iv) 176q7p3r7
x3y2 + x2 y3
6a(a + b)2
(a b)(a2 ab + b2)
(v) x2y4 + xy5
(vi) 9a2(a2 b2) (vii) (a3 + b3) (a + b)
x2 6x + 5
2x2 x 6
a2 6a + 9
(ix)x2 + 4x 45 (x) 3x2 2x 8
(viii) (a2 9)
a2 b2 2bc c2
2a2 + ab b2
14x4 9x2 + 1
(xi) 21x4 + 4x2 1 (xii) a2 + 2ab + b2 c2 (xiii) a3 + a2b a b
Express the following fractions with the common denominators:
b
c2
ab
ac
bc
a2
(ii) 4cd2 , 6bd3 , 10ab5
(i) 12bc , 20ca , 28ab
b
c
ab bc ca
a
(iv) ab , bc , ca
(iii) a b , a + b , a(a + b)
ab
a
a
b
c
a+b
(vi) a b , a2 + ab + b2 , a3 b3
(v) (a b)2 , a3 + b3 , a2 b2
3
4
2
(vii) x2 x 2 , x2 + x 6 , x2 + 6x + 9
ab
bc
ca
(viii) (a b) , (b c) , (c a)

145

3.

Add :
z
x
y
a
b
x+y xy
(ii) xy + yz + zx
(iii) a b + a + b
(i) x + y
1
1
ab
2x + 3y 2x 3y
(v) (a + b)2 + a2 b2 + (a + b)3
(iv) 2x 3y + 2x + 3y
1
1
1
(vi) x2 3x + 2 + x2 5x + 6 + x2 4x + 3
1
1
1
(vii) x2 x + 1 + x2 + x + 1 + x6 1
1
1
1
(viii) x2 5x + 6 + x2 4 + x2 x 6

4.

Subtract:
x
x2
(i) x 4 x2 16
(a b)2
1
(iii) a + b a3 + b3
1
x2
(v) x 2 x2 + 2x +4

5.

1
1
(ii) a(a + b) a(a b)
x2 + 9y2 x 3y
(iv) x2 9y2 x + 3y
1
1
(vi) 1 x + x2 1 + x +x2

Simplify:
ab bc ca
ab bc ca
(i) ab + bc ac (ii) a + b + b + c c + a
a
c
b
(iii) (a b)(b c) + (c a)(a b) + (b c)(a c)
a+b ab
2ab
(iv) a b a + b + b2 a2

1
1
2x
(v) x + 2y + x 2y x2 4y2

x 2y x + 2y
8xy
1
2
1
2
(vi) x + 2y x 2y x2 + 4y2 (vii) a b 2a + b + a + b 2a b
2
x
1
3
(viii) x 1 x2 + 1 x + 1 x2 1
a2
6a
1
(ix) a 2 a2 + 2a + 4 + a3 + 8

146

1
1
2x
(x) 1 x + x2 1 + x + x2 1 + x2 + x4
1
4
32
1
(xi) x 2 x + 2 x2 + 4 + x4 + 16
b a
b c
c a
(xii) (a b)(b c) + (a c)(b c) + (a c)(b c)
1
a
1
(xiii) a b c + a b + c a2 + b2 c2 2ab
1
6y
1
(xiv) 2x 3y 2x + 3y + 4x2 9y2
2.2. Multiplication of Fractions
The product of fractions of more than one is such a fraction whose numerator is
equal to the product of numerators of the given fractions and denominator is equal
a

to the product of denominators of the given fractions. b and d are two fractions
a c ac
and their product is b d = bd .
a c e g a c e g aceg
The product of the fractions, b , d , f , h = b d f h = bdfh
Remark : The fraction obtained from the product is to be expressed in its lower
terms. In the process of finding the product, the common factors of numerators
and denominators can also be cancelled out. If an expression is a, then the
numerator of the expression is 'a' and its denominator is 1.
b a b ab
For example, a c = 1 c = c
ab
a3b2
Example 1. Multiply c2d3 by c2d2
ab
a3b2 ab.a3b2
(a.a3)(b.b2)
Solution : Required product = c2d3 c2d2 = c2d3.c2d2 = (c2.c2)(d3 .d2)
a4b3
= c4d5
a4b3
Answer : 4 5
cd

147

8a3bc3
15e3fg
Example 2. Multiply : 25d3e2f and 16a2b2pq
8a3bc3
15e3fg
8 15 a3 b c3 e3
f
g
Solution : 25d3e2f 16a2b2pq =
a2 b2 d3 e2 f pq
25 16
3
a.c3eg
3ac3eg
= 10 b.d3pq = 10bd3pq
3ac3eg
Answer : 10bd3pq
x (x y)
y (x + y)
Example 3. Multiply x2 + 2xy +y2 by x3 y3
x (x y)
y (x + y)
Solution : Required product = x2 + 2xy + y2 x3 y3
xy (x y) (x + y)
xy
= (x + y)2 (x y) (x2 + xy + y2) = (x + y) (x2 + xy + y2)
xy
Answer : (x + y) (x2 + xy + y2)
a2 x2 a2 y2
1
Example 4. a + y ax + x2 a x = how much?
(a + x) (a x) (a + y) (a y)
1

(a + y)
x (a + x)
(a x)
ay
= x

Solution : Given expression =

ay
Answer : x
EXERCISE 2.2

Multiply :
6a2b2 49c5d4
4a7b8
1. 35c3d3 , 36a5b7 and 21c4d5

20x2y2 28z4
3y7z
2. 21z2 , 9x3y4 and 10x

x2 y2
z2
3. yz , zx and xy

x + 1 x2 + x 2
x2
4. x 1 , x2 + x and x2 + 5x + 6

148

a4 b4
ab
a+b
1 x2 1 b2
x

5. a2 2ab +b2 , a3 + b3 and a3 b3 6. 1 + b , x + x2 and 1 + 1 x

x2 6x 16 x2 11x + 28
x+3
7. x2 4x 21 , x2 12x + 32 and x + 2
2x2 7x + 3 3x2 + 11x 4
2x2 + x 15
8. 2x2 + 7x 4 , 3x2 + 8x 3 and 2x2 11x + 15
ab
a3 b3
a3 + b3
9. a b a3 a2b + ab2 a2b + ab2 + b3
2x(1b2)
1
x2 y2
y
10.

2
y
(x + y) x + bx 2x 2y
a3 + b3 a + b a2 + ab
11. a2 b2 a b (a + b)2
Simplify :
a bb c
12. b + ca + d

2x 1 1
1
13. 1 + x + 1 x2 x x2

x
z
x
x 1
1

14. 1 x + y x + y + z + x + y z 15. 1 + x + 1 x 1 + x2 1 + x + x2

x
8y
x
16. 3x y + 3x + y 9 + x2 y2

2.3. Division of Fractions


The meaning of dividing a fraction by another one is the multiplication of the first
a
c
fraction by the reciprocal of the second fraction. If b is divided by d , the quotient
a d ad
d
c
1
is b c = bc . Here, c is the reciprocal of d . The reciprocal of 'a' will be a .
c
c
c 1
c
If d is divided by a, then quotient is d a = d a = ad .
a2b3
a3b2
Example 1. Divide c2d by cd3
a3b2
cd3
Solution : The reciprocal of the expression cd3 is a3b2

149

a2b3 a3b2 a2b3 cd3 a2b3cd3 bd2


Therefore, 2
cd
cd3 = c2d a3b2 = a3b2c2d = ac
bd2
Answer: ac
9a4b2c3
6a3bc5
Example 2. 10x3y4z 5x2y5z2 = how much?
9a4b2c3
6a3bc5
9a4b2c3 5x2y5z2 3ab yz 3abyz
Solution : 10x3y4z 5x2y5z2 = 10x3y4z 6a3bc5 =
= 4c2x
2x 2c2
3abyz
Answer : 4c2x .
a
a+b
Example 3. Divide a b by a3 b3
a
a+b
a
a3 b3
a
(a b) (a2 + ab + b2)
Solution : a b a3 b3 = a b a + b = a b
a+b
2
2
a (a + ab + b )
=
a+b
2
2
a (a + ab + b )
Answer:
a+b
a3 b3
a4 + a2b2 + b4
Example 4. Divide a + b by
a3 + b3
a3 b3 a4 + a2b2 + b4 (a b) (a2 + ab + b2) (a + b) (a2 ab + b2)
Solution : a + b
=
(a2 + b2)2 (ab)2
a3 + b3
(a + b)
(a b)(a + b) (a2 + ab + b2) (a2 ab + b2)
= (a b)
(a + b) (a2 + b2 + ab) (a2 + b2 ab)
Answer. (a b).
b a
b
a
Example 5. Simplify : a + b + a b a b a + b

a (a b) + b(a + b) a (a + b) b(a b)
Solution : Given expression =

a2 b2
a2 b2
a2 ab + ab + b2 a2 + ab ab + b2 a2 + b2 a2 + b2 a2 + b2 a2 b2
=

= a2 b2 a2 b2 = a2 b2 a2 + b2 = 1
a2 b2
a2 b2
Answer : 1.
=

150

x2 + x 2 x2 3x 10 x2 4x 5
Example 6. Simplify : x2+7x+12 x2 + x 12 x2 4x + 3
x2 + x 2 x2 3x 10 x2 4x 5
Solution : Given expression = x2+7x+12 x2 + x 12 x2 4x + 3
x2 + x 2
x2 + x 12 x2 4x 5
= x2 + 7x + 12 2

x 3x 10 x2 4x + 3
(x + 2)(x 1)
(x + 4)(x 3)
(x 5)(x + l)
(x + 1)
= (x + 3) (x + 4) (x 5) (x + 2) (x 3) (x 1) = (x + 3)
(x + 1)
Answer : (x + 3)
(x + y)2 3xy (x + y)
x 3 + y3
Example 7. Simplify : (x y)2 + 3xy (x3 y3)
(x y)
x3 + y3
(x + y)2 3xy (x + y)
Solution : Given expression = (x y)2 + 3xy (x3 y3)
(x y)
(x + y) (x2 xy + y2) (x y) (x2 + xy + y2) (x + y)
=

(x y) = (x +y)2 = x2 +2xy + y2
(x2 + xy + y2)
(x2 xy + y2)
Answer : x2 + 2xy + y2.
Remark : If there are +, , , and brackets, the rule of BODMAS is followed
in case of simplification. where B = Bracket, O = Of, D = Division, M =
Multiplication, A = Addition, S = Subtraction.
EXERCISE 2.3
Divide the first expression by the second expression:
2a2 9b2
4a2b2 12ab3
7a2b2c2 21a4b4c4
5a a + b
1. 3b , 16ac
2. 3c2 , 5c5
3. 12xyz , 4x3y3z3 4. b , 5b
a2 + ab ab + b2
x2 49 x + 7
a3 b3 a2 + ab +b2
5. 4a2 , 6b2
6. x2 25 , x + 5
7. a + b , a2 b2
x2 4
x2 5x 14
x2 x 30 x2 + 6x 72
8. x2 + 3x 18 , x2 36
9. x2 36 , x2 + x 56
Simplify :
b
1
1
2a + b


10. a + b 1 1 a + b
11. 1 + x 1 x2

151
3

y x
y
x
12. x + y +
x + y

y
x y

2ab a b

13. 1 a2 + b2 a b 3ab

a + b
a+b
b
b2 1
a

2a
14. a + b a b 1 a2 b2 15. a + b a b + a2 b2 a + b + a2 b2

x3 + y3 + 3xy (x + y)
(x y)2 + 4xy
16.
3
(x + y)2 4xy
x y3 3xy(x y)
2
a
a2 + ab + b2
a b
a

17. b + a 1 b2 + b + 1 b(a2 ab + b2)

x2 + 2x 15
x2 25
x2
18. x2 + x 12 x2 x 20 x2 5x + 6
3x2 4x 4 2x2 + 5x 3 x2 + x 6
19. 3x2 x 2 2x2 + x 1 x2 1
2

a b
b a + b a b a + b a b
b
a
20. a b a + b a b a + b + a b + a + b a b a + b 1 + a2

a3 + a2b + ab2 + b3
a+b
a2 + b2 a2 b2
21.
a2 ab 2b2 4a2 a 2b
2a (a b)
Multiple Choice Questions [Marks () on the correct answer]
1.

If

x y z
, ,
are expressed in fractions with common denominator
p q r

then the result would be-

2.

(a)

xqr ypr zpq


,
,
pqr pqr pqr

(b)

xpq yqr zrp


,
,
pqr pqr pqr

(c)

xrp ypq zqr


,
,
pqr pqr pqr

(d)

xqr ypq zpr


,
,
pqr pqr pqr

a3b2
ab
Which one of the following indicates the product of c2d3 and c2d2 ?
(a)

a 3b 3
c4d 5

(b)

a 4b 3
c4d 5

(c)

a 3b 4
c5d 4

(d)

a 4b 2
c5d 5

N.M.G. -21

152

3.

4.

By which one of the following the algebraic fractions can be


expressed in general?
x
y

(a) x y

(b)

(c) 2 3

(d) 2 3

How much should be subtracted from the sum of

1
1
and
,
x y
x+ y

so that the result would be 2?


(a) 2

5.

2
x y2

(b) 2

(c)

2
2
x y2

(i)

The sum of

(d)

2x
x y2
2

2x
2
x y2
2

2
1
1
and
is equal to 2
x+ y
x y
x 1

(ii) The H.C.F. of the denominators of the fractions


x
y
z
is 1.
,
,
( x y ) ( y z ) ( y z ) ( z x) ( z x ) ( x y )

(iii) The product of more than one fractions means the ratio of
product of numerators and product of denominators.
Which one of the following is correct on the basis of the above
informations.
(a) i and ii
(b) i and iii
(c) ii and iii
(d) i, ii and iii
Observe the following four algebraic expressions:
x
y
x
y
,
,
,
x+ y x y x y x+ y

6.

Answer the questions (6 8) on the basis of the above informations.


Which one of the following is the L.C.M. of the above expressions?
(a) x+y
(b) x y
2
2
(d) x2y2
(c) x +y

153

7.

8.

Which one of the following is the sum of the 1st and 2nd
expressions?
(a)

x2 + y2
x2 y2

(c)

2x 2
x2 y2

(b)

2(x 2 + y 2 )
x2 y2

(d)

2y2
x2 y2

Which one of the following is the ratio of the sum of first two
expressions and subtraction of last two expressions?
(a) -1
(b) 0
(c) 1
(d) 2

CREATIVE QUESTIONS
1.

Observe the following algebraic expressions:


a b
+ 1 ,
b a

a2 a 1
a
2 + 1 ,
2

b a + b a b2
b

(i) Find the ratio of 1st and 2nd expressions.


(ii) How much should be added with the product of 1st and 3rd
expressions so that the result would be
(iii) Convert the
denominator.
2.

expressions

into

a
?
a+b

fractions

with

common

Observe the following algebraic expressions:


x
y
xy
, 3
, 4
3
3
2 2
x +y
x y
x + x y + y4
3

(a) Find the quotient of 1st expression and 3rd expression.


(b) Find the subtraction of the 3rd expression from the sum of 1st
and 2nd expressions.
(c) Convert the expressions into fractions with common denominator.

154

3.

x2 4
x 2 5 x 14 x 2 + 6 x 72
,
, 2
are three algebraic
x 2 + 3 x 18
x 2 36
x + x 56

expressions.
(a) Express the 1st expression in simple form.
(b) Divide the 1st expression by the 2nd expression.
(c) If the result of (b) is divided by the 3rd expression, then how
x2 2
much the quotient would be more than
?
( x 3) ( x + 12)

Chapter III

Linear Equations And Their Applications


3.1 Linear Equation
The concept of linear equation and the solution of given linear equations of easy
form containing one unknown quantity have been solved in class VII. We have
also learnt the linear equation on the basis of real problems and their solution.
Here it is discussed briefly again.
If two expressions are connected by the sign of equality, then such relation is
called an equation. There are two expressions in the equation of which one at the
left and the other at the right side of the sign of equality. The expression in the left
side is called left hand side and the expression in the right side is called right
hand side.
8x + 2 = 3x + 12 is a linear equation.
Here x is an unknow quantity. 8x + 2 is left hand side and 3x + 12 is right hand
side. 5x + 4 = 14 is a linear equation. 3 5 + 4 = 11 + 8 is not an equation. Here
there is no unknown quantity though there is sign of equality.
The value of the unknown quantity for which both the sides of the equation
become equal is called the root of the equation or solution of the equation.
There will be only one root for one unknown quantity whose power is one. The
process of finding the roots of the equation is known as solving the equation.
If we put x = 2 in the equation 8x + 2 = 3x + 12, then value of both left hand side
and right hand side become 18. Hence 2, the value of x is the root of the equation.
In 5x + 3, x is an unknown quantity and it is called a variable. Here any value can
be assingned to x. To find the roots of the equation, the following rules are
followed :
(a) Interchange of sides of terms
If any term of right hand side is transfered to left hand side, then sign before that
term is to be changed. Similarly, if a term of left hand side is transfered to right
hand side, then sign before that term also be changed, a + b = c will be a = c b
and a = b + c will be a b = c.
(b) Cancellation law of addition
The root of a linear equation is not changed if equal terms are added to or
subtracted from the expressions of both the sides.
If a = b, then a + c = b + c, a c = b c.

156

(c) Cancellation law of multiplication


The root of a linear equation is not changed if any term not zero is divide or
multiply by the expression of both the sides.
If ac = bc and c 0, then a = b;
a b
If c = c and c 0, then a = b.
(d) Cross multiplication
a c
If b = d , then ad = bc, where b 0 and d 0
Example 1. Solve : 7(3 2x) + 5(x 1) = 34.
Solution : 7(3 2x) + 5(x 1) = 34 or, 21 14x + 5x 5 = 34.

or, 14x + 5x = 34 21 + 5. [Interchanging the sides of the terms]


18
or, 9x = 18 or, x = 9 , [dividing both sides by 9]
or, x = 2.
Required solution, x = 2.
Answer : x = 2.
Example 2. Solve : 5(x + 7) + 9(2x + 23) = 3(x + 6) 8x.
Solution: 5(x + 7) + 9(2x + 23) = 3(x + 6) 8x.

or, 5x + 35 + 18x + 207 = 3x + 18 8x.


or, 5x +18x 3x + 8x = 18 35 207.
224
or, 28x = 224 or, x = 28 [dividing both sides by 28]
or, x = 8.
Required solution, x = 8.
Answer : x = 8.
Remark : In the process of solution, unknown quantities are kept in the left hand
side and the numbers or known quantities are kept in the right hand side.
3.2. Solution of the Equations Reducible to Linear Form
To find the solution of the equation involving fractions the equation is to be
reduced to a linear equation free from fractions. If the denominators of the
fractions can be removed then the equation involving fractions becomes free from
fractions. In order to remove the denominators, the L. C. M. of the denominators is

157

to be determined and then each term of both sides is to be multiplied by this L. C. M.


and thus the fractions become free from denominators.
5x 2 x + 4 6x + 3 x + 2
Example 3. (a) Solve : 8 + 6 = 5
4
Solution : Here, L. C. M. of denominators, 8, 6, 5 and 4 is 120.
5x 2 x + 4 6x + 3 x + 2
Now, 8 + 6 = 5
4
5x 2 x + 4
6x + 3 x + 2
or, 8 + 6 120 = 5 4 120.

5x 2
x+4
6x + 3
x+2

120
+

120
=

120

8
6
5
4 120.
or, 75x 30 + 20x + 80 = 144x + 72 30x 60.
or, 75x + 20x 144x + 30x = 72 60 + 30 80,[Interchanging the sides of terms]
or, 19x = 38 or, x = 2, [dividing both sides by 19]
Required solution, x = 2.
Answer : x = 2.
3x
x
Example 3. (b) Solve : 4 + 3 = 6 + 4.
3x
x
Solution: 4 + 3 = 6 + 4.
3x x
or, 4 6 = 4 3, [interchanging the sides of terms]
9x 2x
or, 12 = 1, [L. C. M. of 4 and 6 = 12 and applying algebraic subtraction]
7x
or, 12 = 1. or, 7x = 12, [multiplying both sides by 12]
12
or, x = 7 [dividing both sides by 7]
12
Required solution, x = 7
Examining correctness it is found that left hand side = right hand side.
12
Answer : x = 7
or,

158

3x 5x 2 4x + 5
Example 3. (c) solve : 4 + 6 = 8
3x 5x 2 4x + 5
Solution : 4 + 6 = 8
3x
5x 2
4x + 5
or, 24 4 + 24 6
= 24 8
[L.C.M. of 4, 6 and 8 is 24 and
multiplying both sides by 24]
or, 6(3x) + 4(5x 2) = 3(4x + 5)
or, 18 x + 20x 8 = 12x + 15
or, 38x 12x = 15 + 8, [interchanging the sides of terms]
23
or, 26x = 23 or, x = 26 [dividing both sides by 26]
23
Required solution, x = 26
23
Answer : x = 26
5
2
Example 4. Solve : 4x 15 = 3x + 1
5
2
Solution : 4x 15 = 3x + 1
or, 5(3x + 1) = 2(4x 15), [by cross multiplication]
or, 15x + 5 = 8x 30
or, 15x 8x = 30 5, [interchanging the sides of terms]
35
or, 7x = 35, or, x = 7 or, x = 5,
Required solution, x = 5.
Answer : x = 5.

2x + 5 2x 3 2x 5 2x + 3
Example 5. Solve : 2x + 3 2x 5 = 2x 3 2x + 5
2x + 5 2x 3 2x 5 2x + 3
Solution : 2x + 3 2x 5 = 2x 3 2x + 5

159

or,

(2x + 5)(2x 5) (2x 3)(2x + 3) (2x 5)(2x + 5) (2x + 3)(2x 3)


=
(2x + 3)(2x 5)
(2x 3)(2x + 5)

4x2 25 4x2 + 9 4x2 25 4x2 + 9


16
16
or,
=
or,
=
(2x + 3)(2x 5)
(2x 3)(2x + 5)
(2x + 3)(2x 5) (2x 3)(2x +5)
1
1
or, (2x + 3)(2x 5) = (2x 3)(2x +5) [dividing both sides by 16]
or, (2x 3)(2x + 5) = (2x + 3)(2x 5) [by cross-multiplication]
or, 4x2 6x + l0x 15 = 4x2 + 6x l0x 15
or, 4x2 + 4x 4x2 + 4x = 15 + 15 [interchanging the sides of terms]
0
or, 8x = 0
or, x = 8 , [dividing both sides by 8]
or, x = 0. [The quotient is zero if 0 is divided by any number]
Required solution, x = 0.
Answer : x = 0.
5
4
9
Example 6. Solve : x 1 + x 2 = x 3
5
4
9
Solution : x 1 + x 2 = x 3
5
4
5
4
or, x 1 + x 2 = x 3 + x 3 [Q 9 = 5 + 4]
5
5
4
4
or, x 1 x 3 = x 3 x 2
or,

5 (x 3) 5 (x 1) 4 (x 2) 4 (x 3)
(x 1) (x 3) = (x 3) (x 2)

5x 15 5x + 5 4x 8 4x + 12
or, (x 1) (x 3) = (x 3) (x 2)
10
4
or, (x l) (x 3) = (x 3) (x 2)
1.

5
2
or, x 1 = x 2 [dividing and multiplying both sides by 2 and (x 3)
respectively]

160

or, 5(x 2) = 2(x 1) [by cross multiplication]


or, 5x + 10 = 2x 2
N.M.G. -22
or, 5x 2x = 2 10 [interchanging the sides of terms]
12
or, 7x = 12 or, x = 7
12
Answer : x = 7 .
Remark : In the above example the denominators of algebraic fraction are
different but there is a relation among the three numerators. The sum of the
numerators of left hand side is 5 + 4 = 9, which is the numerator of right hand
side. Hence the above method is easier to find solution. Different techniques are
applied to solve the equations involving fractions. A particular technique is
applied above. There are many such techniques of which some are applied below :
xa xb xc
Example 7. Solve : b + c + c + a + a + b = 3
xa xb xc
xa xb xc
Solution : b + c + c + a + a + b = 3 or, b + c + c + a + a + b 3 = 0
xa
xb
xc
or, b + c 1 + c + a 1 + a + b 1 = 0
or,

xabc xbca xcab


+
+
=0
b+c
c+a
a+b

or,

xabc xabc xabc


+
+
= 0.
b+c
c+a
a+b

1
1
1
or, (x a b c) b + c + c + a + a + b = 0

1
1
1
or, x a b c = 0 [Since b + c + c + a + a + b 0; so dividing both sides by it]

or, x = a + b + c
Required solution, x = a + b + c.
Answer : x = a + b + c.
Remark : If AB = 0, then must be either A = 0 or, B = 0. Hence, if any equation
AX = 0 and A 0, then the factor involving the unknown quantity must be equal
to zero.

161

2x
8x + 3
Example 8. Solve : x 4 + 2x + 1 = 6
2x
8x + 3
2x
8x + 3
Solution : x 4 + 2x + 1 = 6 or, x 4 + 2x + 1 = 2 + 4
2x

8x + 3

or, x 4 2 = 4 2x + 1 [interchanging the sides of terms]


or,

2x 2x + 8 8x + 4 8x 3
8
1
=
or,
=
x4
2x + 1
x 4 2x + 1

or, 8(2x + 1) = x 4

or, 16x + 8 = x 4

or, 16x x = 4 8 [ interchanging the sides of terms]


12 4
or, x = 15 = 5
4
Required solution, x = 5
4
Answer : x = 5
or, 15x = 12

Remark : If x of (x 4) is multiplied by 2, then the numerator of the first 2x is


obtained and also if 2x of (2x +1) is multiplied by 4, then numerator of second
fraction, 8x is obtained. Hence 6 of right hand side is written as 2 + 4.
2

Example 9. Solve : 2x 1 + 3x 1 = 4x + 1
2

Solution : 2x 1 + 3x 1 = 4x + 1
2
3
4
4
or, 2x 1 + 3x 1 = 4x + 1 + 4x + 1
2
4
4
3
or, 2x 1 4x + 1 = 4x + 1 3x 1
8x + 2 8x + 4
12x 4 12x 3
or, (2x 1) (4x + 1) = (4x + 1) (3x 1)

or,

7
6
=
(2x 1) (4x + 1) (4x + 1) (3x 1)

162

6
7
or, 2x 1 = 3x 1 [multiplying both sides by (4x + l)]
or, 18x 6 = 14x + 7, [by cross-multiplication]
or, 18x + 14x = 7 + 6, [interchanging the sides of terms]
13
or, 32x = 13
or, x = 32
13
Repuired solution, x = 32
13
Answer : x = 32
Remark : If 4x of (4x + 1) is multiplied by the numerator 2 of the first fraction, it
becomes 8x and if 2x of (2x 1) of first fraction is multiplied by 4, it also
becomes 8x. Again if 4x of (4x + 1) is multiplied by the numerator 3 of the second
fraction, if becomes 12x and if 3x of (3x 1) is multiplied by 4 it also becomes
12x. Here 8, the numerator of the fraction of right hand side is written as (4 + 4).

x + 1 x + 2 10x + 18
Example 10. Solve : x 1 + x 2 = 5x 6
x + 1 x + 2 10x + 18
Solution : x 1 + x 2 = 5x 6
or,

(x 1) + 2 (x 2) + 4 2(5x 6) + 30
+ x2
=
x1
5x 6

2
4
30
or, 1 + x 1 + 1 + x 2 = 2 + 5x 6
2
4
10 + 20
or, x 1 + x 2 = 2 2 + 5x 6
2
10
20
4
or, x 1 5x 6 = 5x 6 x 2 , [interchanging the sides of terns]
or,

10x 12 10x + 10 20x 40 20x + 24


(x 1) (5x 6) = (5x 6) (x 2)

2
16
or, (x 1) (5x 6) = (5x 6) (x 2)

163

1
8
or, x 1 = x 2 , [dividing and multiplying both sides by 2 and (5x 6) respectively]
or, x 2 = 8x 8 or, x 8x = 8 + 2, [interchanging the sides of terms]
6
or, 7x = 6
or, x = 7
6
:. Required solution, x = 7
6
Answer : x = 7 .
4x2 + 7 9x2 + 5 5x2 3x + 4
Example 11. Solve : 2x 1 + 3x 1 =
x1
4x2 + 7 9x2 + 5 5x2 3x + 4
Solution : 2x 1 + 3x 1 =
x1
(4x2 1) + 8 (9x2 1) + 6 5x2 3x + 4
or,
+
=
2x 1
3x 1
x1
4x2 1
8
9x2 1
6
5x2 3x + 4
or, 2x 1 + 2x 1 + 3x 1 + 3x 1 =
x1
(2x + 1) (2x 1)
8
(3x + 1) (3x 1)
6
5x2 3x + 4
or,
+ 2x 1 +
+ 3x 1 =
x1
(2x 1)
3x 1
8
6
5x2 3x + 4
or, (2x + 1) + 2x 1 + (3x + 1) + 3x 1 =
x1
8
6
(5x + 2) (x 1) + 6
or, (5x + 2) + 2x 1 + 3x 1 =
x1
8
6
6
or, (5x + 2) + 2x 1 + 3x 1 = (5x + 2) + x 1
8
6
6
or, 2x 1 + 3x 1 = x 1 [cancelling (5x + 2) from both sides]
8
6
4
2
or, 2x 1 + 3x 1 = x 1 + x 1
8
4
2
6
or, 2x 1 x 1 = x 1 3x 1

164

8x 8 8x + 4 6x 2 6x + 6
or, (2x 1) (x 1) = (x 1)(3x 1)
4
4
or, (2x 1)(x 1) = (x 1)(3x 1)
1
1
x 1
or, 2x 1 = 3x 1 , [multiplying by 4 ]

or, 3x + 1 = 2x 1
2
or, 5x = 2 or, x = 5
2
Required solution, x = 5
2
Answer : x = 5

EXERCISE 3. 1
Solve:
1. 15x 9 = 11x 25;
2. 8x + 5(x + 7) = 3 (x + 6) 9 (2x + 23);
3. 3(4x + 1) + 9 = 5(3x + 2) + 4(2x 5);
4. 2(x 1) 4(x 3) = 3(x 2);
x
3x
5x
3x
3x 4 5x 3
5. 5 + 4 = 10 + 6;
6. 6 + 3 = 5 10; 7. 2 + 3 = 4 ;
2x + 5 3x + 1 2x 3
7x 10x 3 7x + 5
8. 6 9 = 8 ;
9. 2 8 = 6 ;
6x + 7 2x 1 3x 2
x 3 2x 3 6x + 1 x 6
10. 5 10 = 15 ;
11. 7 3 = 2 4 ;
2x 3 x 3 3x 4 2x + 5
5x + 6 4x 5 2x + 3 3x 7
12. 4 5 = 8 3 ;
13. 4 + 5 = 8 + 3 ;
5
7
12
a
b
a+b
14. x + 2 + x 3 = x 1 ;
15. x a + x b = x a b ;
6
5
11
8
9
7
16. x + 1 +x + 5 = x + 3 ;
17. 2x 1 + 3x 1 = x + 1 ;
4
15
35
10
1
18
18. 2x + 1 + 5x + 4 = 7x + 6 ;
19. 2x 5 + x + 5 = 3x 5 ;
x bc x ab x ca
20. b + c + a + b + c + a = a + b + c ;

165

ax a2 bx b2 cx c2
21. b + c + c + a + b + a = a + b + c ;
x+a
x+b
x+c
22. b + c + 2a + c + a + 2b + a + b + 2c = 3;
3x bc 3x ca 3x ab
23. b + c + c + a + a + b = a + b + c ;
2x 1 3x 4 5x 12
24. x 1 + x 2 = x 3 ;
4x 7 15x + 11 12x + 1
25. 4x + 5 + 5x + 7 = 3x + 4 ;
x2 x + 1 x2 2x + 1 2x2 6x + 2
26. x 1 + x 2 =
;
x3
2x2 3x + 7 6x2 + 2x + 21 3x2 + 8x + 7
27. 2x 1 +
=
;
3x + 1
x+3
x2 x3 x1 x4
28. x 3 + x 4 = x 2 + x 5 ;
2x + 11 9x 9 4x + 13 15x 47
29. x + 5 3x 4 = x + 3 3x 10 .
3.3 Applications of Equations in Solving the Problems Relating to Our Real
Necessity.
Problems relating to our real necessity can be solved by using rules of arithmetic.
Those problems can be solved easily with the help of algebraic equations. There is
no general rule for solving all the problems. In solving the problems the required
term is denoted by x. Then the problem is arranged in the form of an equation
according to the given conditions. The process of solution can be understood from
the following examples :
Example 1. Divide Tk. 420 between Farid and Zafar in such a way so that Farid
gets Tk. 30 more than double of Zafar's share. How much each of Farid and Zafar
would get?
Solution: Let, Zafar gets Tk. x.
Therefore, Farid gets Tk. (2x + 30)
According to the conditions, x + (2x + 30) = 420 or, x + 2x + 30 = 420.
390
or, 3x = 420 30 or, 3x = 390 or, x = 3 = 130.

166

Therefore, Zafar gets Tk. 130 and Farid gets Tk. (2x+30) = (2130+30) = Tk. 290.
Answer : Farid gets Tk. 290 and Zafar gets Tk. 130.
Example 2. The present age of Razat is 9 times that of Anik. After 9 years, the age
of Razat will be 3 times that of Anik. What are the present ages of Razat and
Anik?
Solution : Let, the present age of Anik is x years.
Therefore, present age of Razat is 9x years.
After 9 years, the age of Anik will be (x + 9) years and the age of Razat will be
(9x + 9) years.
Hence, according to the conditions,
9x + 9 = 3(x + 9) or, 9x + 9 = 3x + 27.
18
or, 9x 3x = 27 9 or, 6x = 18 or, x = 6 = 3
Therefore, the present age of Razat is 9x years = 9 3 years = 27 years
and present age of Anik is x years = 3 years.
Answer : 27 years and 3 years.
Example 3. Hanif bought some mangoes as 15 mangoes at Tk. 100.00 sold 12
mangoes at Tk. 100.00 and thus he made a profit of Tk. 400.00. How many
mangoes did he buy?
Solytion : Let, Hanif bought x mangoes.

20x
x

Therefore, the cost price of x mangoes is Tk. 15 100 = Tk. 3

x 100
25x
= Tk.
3
12

Again, the selling price of x mangoes is Tk.

25x 20x
Hence, according to the conditions, 3 3 = 400 [sales cost = profit]
25x 20x
5x
=
400
or,
3
3 = 400
400
or, x = 5 3 = 80 3 = 240
or,

Required number of mangoes = 240.


Answer: 240.

167

Example 4. If Ranjit is in possession of 200 coins in fifty paisa and ten paisa and
that amounts to Tk. 44, then what is the number of coins of each kind?
Solution : Let, the number of coins in fifty paisa = x pieces.
Then, the number of coins in ten paisa = (200 x) pieces.
x
x pieces of coins of fifty paisa amounted to Tk. 2 and (200 x) pieces of coins
200 x
of ten paisa amounted to Tk. 10
x 200 x
5x + 200 x
Hence, according to the conditions, 2 + 10 = 44 or,
= 44
10
or, 4x + 200 = 440 or, 4x = 440 200 or, 4x = 240 or, x = 60.
Therefore, the number of coins in fifty paisa pieces, x = 60 and the number of
coins of ten paisa pieces (200 x ) = 200 60 = 140 pieces.
Answer : 60 and 140.
Example 5. Income of Ranjan is equal to that of Ranjit. Ranjan saves one fifth of
his total income. But in one year Ranjit spends Tk. 4000 more than the amount
which Ranjan spends. After four years it is found that Ranjit is indebted Tk.
11000. How much is the income of each of them?
Solution : Let, each of them earns Tk. x in one year.
x
4x

Then, Ranjan spends Tk. x 5 = Tk. 5 in a year.

4x

Therefore, Ranjit spends Tk. 5 + 4000 in a year and in one year he is indebted

11000
to Tk. 4 = Tk. 2750
Hence, Ranjit spends Tk. 2750 more than the amount of his income yearly.
4x

According to the conditions, 5 + 4000 = x + 2750

4x

or, 5 + 4000 x = 2750

4x + 20000 5x
or,
= 2750
5
20000 x
or,
= 2750
or, 20000 x = 13750.
5
or, x = 20000 13750
or, x = 6250.
Therefore, yearly income of each of them is Tk. 6250.00
Answer: Tk. 6250.00
N.M.G. -23

168

Example 6. A businessman of banana bought some bananas each at Tk. 1.00 and
the same number of bananas each at 50 paisa and sold all bananas each at 80 paisa.
Thus he made a profit of Tk. 100.00. How many bananas did he buy?
Solution : Let, the businessman bought x bananas. Half of the total number of x
x
bananas was bought each at Tk. 1.00 and therefore, its cost price is Tk. 2 .
1
x 1
Again the rest was bought each at 50 paisa or Tk. 2 , so its cost price is Tk. 2 2

3x
x x 1
x x
Hence, the total cost price is Tk. 2 + 2 2 = Tk. 2 + 4 = Tk. 4 .

4
As the total number of bananas are sold each at 80 paisa or, Tk. 5 , the selling
4x
4
price is Tk. 5 x = Tk. 5

4x 3x
According to the condition, 5 4 = 100. [selling price cost price = profit]

or,

16x 15x
x
=
100
or,
20
20 = 100 or, x = 2000.

Hence, the number of bananas is 2000.


Answer : 2000.
EXERCISE 3.2

1.

The sum of two numbers is 100. Twice of a number as much smaller than
100 as three times of another number is that much greater than 160. What are
those two numbers ?

2.

The sum of two numbers is 61. Twice of a number is greater than two third
of another number by 10. What are those two numbers ?

3.

10 years ago, father was five times as old as his son. After 20 years, father
will be twice as old as his son. What are the present ages of father and son?

4.

The sum of two numbers is 75. One third of greater number is as much
smaller than 30 as four times of smaller number is that much greater than 50.
Find those two numbers.

169

5.

The denominator of a fraction is greater than its numerator by 1. If 2 is


subtracted from numerator of the fraction and 2 is added to its denominator,
1

the fraction becomes equal to 6 . Find the fraction.


6.

Divide Tk. 104.00 among Chaity, Pew and Rafat in such a way so that double
of Chaity's share, three times of Pew's share and four times of Rafat's share
equal to one another.

7.

Rahim bought a certain number of dates at Tk. 100 for two and the same
number of dates at Tk. 100 for three dates and sold all of them at Tk. 200
for five dates. As a result he made a loss of Tk. 4. How many dates did he
buy?

8.

Divide Tk. 170 among 50 men and women such that each man gets
Tk.350 and each woman gets Tk. 325. Find the numbers of men and
women.

9.

The number of male students is of the 6 students in a school and the rest are
female students. If the number of male students is more than that of female
students by 120, find the number of students in the school.

10. Half of a pillar is under the earth, one third of it is within water and the rest of
2 metres is above water. What is the length of the pillar?
11. Monju bought a certain quantity of sugar at Tk. 40 per kg. and certain
quantity of sugar at Tk. 30. Then he sold the sugar at Tk. 36 per kg. and these
1

made a profit of 122 % per kg. Find the ratio that he bought both kinds of
sugar.
12. 100 coins in fifty paisa pieces and twenty five paisa pieces were found in a
box and amounted to Tk. 45. How many coins of each kind were there?
13. Tongi is 4 kms. from Uttara. From Uttara Taher was riding by ricksha at
6km/hour and kalam was walking towards Tongi at 3 Km/hour from the
same place. After reaching at Tongi Taher took rest for 10 minutes and then
started towards Uttara. How far from Tongi will he meet Kalam?

170

!4. Farid and Zafar had 80 kg. of goods with them. They had to pay Tk. 125 and
Tk. 75 respectively for carring cost of goods in addition to quantities which
they can carry with them free of cost. If all goods belonged to one of them,
he had to pay Tk. 300.00 for carring cost. How much weight of goods did
each of them carry and how much weight of goods could each of them carry
free of cost ?
Multiple Choice Questions [Mark () on the correct answer]
1. How many unknown quantities are there in a simple linear
equation?
(a) 1
(b) 2
(c) 3
(d) Infinite
2. Which one of the following is the cancellation law of addition?
(a) If a + c = b + c, then b + c = a + c,
(b) If p + q = r, then p = r q,
(c) If m + n = p + n, then m = p
(d) If ac = bc, c 0, then a = b
3.

If

a c
= , then for which law of the following is applicable for ad = bc?
b d

(a)
(b)
(c)
(d)
4.

If

Side interchange
Cancellation law of addition
Cancellation law of multiplication
Cross multiplication

x
x
+ 2 = , then which one of the following is the value of x?
6
2

(a) 3
(c) 5
5.

One number is 1

(b) 4
(d) 6
1
times of the other and the sum of the two
2

numbers is 25. Which one of the following is the large number?


(a) 20
(b) 15
(c) 12
(d) 10

171

6.

7.

The 5 times of the age of Akib is equal to the 3 times of the age of
Arif. The sum of their age is 24 years, then which one of the
following are the ages of Arif and Akib respectively?
(a) 14, 10

(b) 9, 15

(c) 15, 9

(d) 18, 6

The

5
part of a pillar is black coloured and the remaining part is
6

white coloured. If the difference of the parts of the pillar is 6 metre,


then which one of the following is the length of the pillar?

8.

(a) 9 metre

(b) 8 metre

(c) 6 metre

(d) 5 metre.

The sum of the ages of Mr. Jagadish and his two sons is 36 years.
What will be the sum of their ages after 5 years?
(a) 40

(b) 41

(c) 46

(d) 51

CREATIVE QUESTIONS
1.

Mr. Khaleque bought some mangoes as 15 mangoes at Tk. 100.00 and sold
12 mangoes at Tk. 100.00 and thus he made a profit of Tk. 400.00.
(a) Form an equation in the view of the above informations.
(b) Find the buying price and selling price of his mangoes.
(c) If Mr. Khaleque paid the shopkeeper total price by 170 notes of Tk. 5.00
and Tk. 10.00, then how many notes of which type did he give?

2.

Ranu, Rabu and Panu are three sisters. Taka 208 is distributed among them
so that the double of Ranus share is triple of Rabus share and 4 times of
Panus share are equal to one another.
(a) Form an equation in view of the above informations.
(b) Find the amount of taka of Ranu, Rabu and Panu.

(c) How much amount of taka would be added with each of their amount so
that the summation will equal to three times of the amount of Rabu and
Panu and four times of the amount of Ranu?

172

3.

Lutfor and Jagadish are two friends. The earnings of both of them
are equal, Lutfor used to save

1
of his earnings. He distributed his
5

savings between his wife and daughter such that the seven times of
daughters amount is equal to the three times of wifes amount. But
in three years Jagadish spends Tk. 12000 more than the amount
which Lutfor spends. After 5 years it is found that Jagadish is
indebted Tk. 13750.
(a) How much taka was indebted by Jagadish in one year and how
much more taka did he spend than that of Lutfor?
(b) Find out the amount of yearly earnings of each of them.
(c) Find the amount of taka of the wife and daughter of Lutfor.

Chapter IV

Simultaneous Linear Equations And Their


Applications
4.1. Simultaneous Linear Equations
x y = 4 is an equation. Here x and y are two unknown quantities. Each of the
unknown quantities, x and y are of first degree. It is a linear equation containing
these two unknown quantities. The equation will be satisfied by those two
numbers whose difference is 4. For example, x = 5, y = 1; or, x = 6, y = 2;
or, x = 7, y = 3; or, x = 8, y =4; or, x 2, y = 2; or, x = 1, y = 5; ...... etc. will
satisfy the equation. There are innumerable values of two unknown quantities of
the equation.
x + y = 6 will be satisfied by innumerable pairs of values of two quantities, x and
y. For example, x = 5, y = 1; or, x = 4, y = 2; or, x = 3, y = 3; or, x = 7, y = 1;
or, x = 8, y = 2, --------etc will satisfy the equation.
Now, if two equations x y = 4 and x + y = 6 are taken together, then the
innumerable pairs of values of x and y obtained from both the equations, the pair
of those values x = 5 and y = 1 will satisfy both the equations. Hence, two
equations x y = 4 and x + y = 6 are simultaneously satisfied for only values of
x = 5 and y = 1.
If more than one equation are satisfied simultaneously by the values of unknown
quantities, then the equations taken together are called simultaneous equations
and if the unknown quantities are of first degree then simultaneous equations are
called simultaneous linear equations.
The values of the unknown quantities which satisfy simultaneous equations
simultaneously are called the roots or the solution of the simultaneous equations.
Here two equations, x y = 4 and x + y = 6 are simultaneous equations. Its only
solution, x = 5 and y = 1 is written shortly as (x, y) = (5,1).
4.1.1. Solution of Two Simultaneous Linear Equations Involving Two
Unknown Quantities
There are three methods of finding the solution of two simultaneous linear
equations. The methods are:
(a) Method of substitution
(b) Method of elimination and
(c) Method of cross multiplication.

174

Substitution method and elimination method are discussed below.


(a) Method of Substitution
In this method, the value of any one of two unknown quantities from any one of
two equations is expressed by the other and the value thus obtained is substituted
for another equation and then the value of one unknown quantity is found by
applying the method as used in case of linear equations. The value of the other
unknown quantity is found and thus the values of two unknown quantities are
obtained.
In this way to find out the values of unknown quantities.
Example 1. Solve : x y = 5 x + y = 9
Solution : First equation, x + y = 9 ........... (1)
or, x = 9 y, [interchanging the sides of y] ............ (2)
Second equation, x y = 5 ............. (3)
or, 9 y y = 5 [ putting the value of x from (2)]
or, 9 2y = 5
or 2y = 5 9 [ interchanging the sides of 9]
or, 2y = 4
or, y = 2 [dividing both the sides by 2]
putting the value of y in equation (1),
x + 2 = 9 or, x = 9 2 [interchanging the sides of 2]
or, x = 7
Required solution, x = 7 and y = 2.
Answer : (x, y) = (7, 2).
[Verification : If we put x = 7 and y = 2 in the given equations then in the first
equation 7 + 2 = 9 = right hand side and in the second equation 7 2 = 5 = right
hand side]
Remark : The value of x can be obtained by putting the value of y either in
equation (2) or (3). Thus the same value is found.
Example 2. Solve : 2x + y = 5
4x 5y = 3.
Solution : First equation, 2x + y = 5 ............ (1 )
5y
or, 2x = 5 y or, x = 2 .............. (2)

175

Again, putting the value of x in the second equation we get,


or, 4x 5y = 3 ......... (3)
5 y
or, 4 2 5y = 3 or, 2(5 y) 5y = 3

or, 10 2y 5y = 3 or, 7y = 3 10 or, 7 y = 7.


y = l.
Putting the value of y in equation (2) we get,
51
4
x = 2 ; or, x = 2 = 2. x = 2
Required solution , x = 2 and y = 1.
Answer : (x, y) = (2, 1).

[Verification : Putting x = 2 and y = 1, the left hand side of first equation = 2.2 +1
= 4 + 1 = 5 = right hand side and the left hand side of second equation = 4. 2 5.1
= 8 5 = 3 = right hand side]
Example 3. Solve : 3x + 2y = 13
5x 3y = 9.
Solution : First equation, 3x + 2y =13 ............ (1)
or, 2y = 13 3x
13 3x
or, y = 2
............... (2)
Second equation, 5x 3y = 9
13 3x
or, 5x 3. 2 = 9 [putting the value of y from (2)]

or, 2.5x 3(13 3x) = 9.2 [multiplying both the sides by 2]


57
or, 10x 39 + 9x = 18 or, 19x = 18 + 39 or, 19x = 57 or, x = 19 = 3. x = 3.
Putting x = 3 in equation (2), we get
y=

13 3.3 13 9 4
= 2 =2 =2 y=2
2

Required solution, x = 3 and y = 2.

Answer : (x, y) = (3, 2)


N.M.G. -24

176

(b) Method of Elimination


In this method, two equations are to be multiplied by two such numbers
respectively so that the absolute values of the coefficients of any one of two
unknowns obtained from two equations after multiplication are both equal. Then
the last two equations are to be added to or subtracted from, so that only one
unknown quantity exists in the equation obtained, the value of the unknown
quantity can be determined from the equation thus obtained. Putting the value of
this unknown quantity in any one of the equations, the value of the other quantity
can be determined.
Example 4. Solve : 3x 2y = 5
5x + 3y = 21.
Solution : Multiplying both the sides of the first equation by 3 it becomes
9x 6y = 15 and multiplying both sides of the second equation by 2 it becomes
l0x + 6y = 42.
Then adding we get, 19x = 57 x = 3.
Putting the value of x in the first equation, we get 3.3 2y = 5
or, 9 2y = 5 or, 2y = 5 9
or, 2y = 4 or, y = 2.
Required solution, x = 3 and y = 2
Answer : (x, y) = (3, 2)
[Verification : Putting x = 3 and y = 2, the left hand side of the first equation
= 3.3. 2.2 = 9 4 = 5 = right hand side and the left hand side of the second
equation = 5.3 + 3.2 = 15 + 6 = 21 = right hand side]
Example 5. Solve : 5x y = 2.
7x 4y = 5
Solution : Multiplying the first equation by 4, we get,
20x 4y = 8 ........... (i)
and 7x 4y = 5 ............ (ii)
Subtracting, 13x = 13. or, x = 1.
Putting x = 1 in the first equation, we get,
5.1 y = 2 or 5 y = 2
or, y = 2 5 = 3 or, y = 3.
Required solution, x = 1 and y = 3.
Answer : (x, y) = (1, 3).

177

Example 6. Solve : 8x + 11 y = 3
6x 13y = 19.
Solution : Multiplying the first equation by 3 and the second equation by 4, we
get,
24x + 33y = 9
and 24x 52y = 76
Subtracting,
85y = 85. or, y = 1.
Now putting y = 1 in the second equation, we get,
6x 13.1 = 19 or, 6x = 19 + 13
or, 6x = 6; or, x = 1.
Required solution, x = 1 and y = 1
Answer : (x, y) = ( 1, 1).
EXERCISE 4.1
Solve by the method of substitution and verify the solutions:

1.

xy=1
x + y = 3.

4.
7.

2. x + y = 5
2x y = 4.

3. x y = 0
2x + 3y = 10.

3y = 4x + 1

5. x + 4y = 14

6. 3x + 5y = 7

4x 5y = 7.

7x 3y = 5.
x y
8. 3 + 2 = 6
x y
2 +4 =4

5x + 4y = 10.

7x 5y = 11
3x + 2y = 13.

9. 3x 5y = 9
5x 3y = 1

2 3
12. x + y = 2
3 2 5
5x 4y = 3
ax by = c
x y =6
Solve by the method of elimination and verify the solutions:
10. 7x + 8y = 9

13. x y = 3
x + y = 7.
16. 2x + 5y = 14
4x 5y = 17.

11. x + ay = b

14. 2x + 3y = 7
8x 7y = 9
17. 7x 5y = 11
3x + 2y = 13.

15. 4x + 3y = 12
5x + 4y = 15.
18. 8x 9y = 20
7x 10y = 9.

178

19. 3x 2y = 0
17x 7y = 52.

20. x y = 2a
ax + by = a2 + b2

x y
21. 3 + 2 = 6
x y
2 +4 =5

x y
x 2
22. 2 + 3 = 3
23. 3 y = 1
24. ax + by = ab
y
x 3
x+6 =3
bx + ay = ab
4 +y =3
4.2 Applications of Linear Simultaneous Equations in Solving Problems
Relating to Daily Necessity:
Many of the problems relating to daily necessity can be solved by using the
concept of linear simultaneous equations. Then one unknown may exists in many
of such problems. Different symbols are used for each of the unknowns and then
the equations are formed in such a case the number of equations of which one is
independent of another is exactly equal to the number of the unknowns used. Then
by solving the equations the values of the unknowns are determined.
Example 1. Divide Tk. 50 between two persons in such a way so that three times
of one person's Taka is equal to seven times of another person's Taka.
Solution : Let, one has Tk. x and another has Tk. y.
Therefore, according to the conditions,
x + y = 50 .............. (1)
and 3x = 7y ............. (2)
from (l), y = 50 x ............. (3)
Putting the value of y from (3) in equation (2), we get,
3x = 7(50 x)
or, 3x = 350 7x
or, 3x + 7x = 350
or, 10x = 350
or, x = 35.
Putting x = 35 in equation (3), we get,
y = 50 35 = 15.
Required shares : One gets Tk. 35 and another gets Tk. 15.
Answer : Tk. 35 and Tk. 15.
Example 2. 10 years ago, father was four times as old as his son. After 4 years
father will be twice as old as his son. What are their present ages?

179

Solution : Let, the present age of the father be x years and that of the son be y
years.
Therefore, according to the first condition,
x 10 = 4(y 10) .............. (1)
and according to the second condition,
x + 4 = 2(y + 4) ................ (2)
From equation (1),
x 10 = 4y 40
or, x = 4y 40 + 10
or, x = 4y 30 ............. (3)
Putting the value of x from (3) in equation (2), we get,
4y 30 + 4 = 2y + 8.
or, 4y 2y = 8 + 30 4
or, 2y = 34
or, y = 17.
From equation (3) we get,
x = 4y 30
or, x = 4.17 30 = 68 30 = 38.
Therefore, the present age of the father is 38 years and that of the son is 17 years.
Answer : 38 years and 17 years.
Example 3. Pia and Papia had some mangoes. If 20 mangoes from Pia are given
to Papia, then the number of mangoes of Papia will be twice that of Pia. Again 10
mangoes from Papia are given to Pia, then the number of mangoes of Pia will be
three times that of Papia. How many mangoes did each of them have?
Solution : Let, x be the number of Pia's mangoes and y be the number of Papia's
mangoes.
Therefore, according to the first condition,
2(x20) = y + 20 ............. (1)
and according to the second condition,
3(y 10) = x + 10 .............. (2)
From the first equation,
y = 2x 40 20
or,
y = 2x 60 .............. (3)
Putting the values of y from (3) in equation (2), we get,
3(2x 60 10) = x + 10 or, 6x 180 30 = x + 10

180

or, 6x x = 10 + 180 + 30 or, 5x = 220 or, x = 44.


Putting the value of x in (3) we get,
y = 2(44) 60 = 88 60 = 28.
Therefore, the number of mangoes of Pia is 44 and that of Papia is 28.
Answer : 44 and 28.
Example 4. If 5 is added to the sum of two digits of a number consisting to two
digits, then the sum will be three times the digit of ten's place. But if 9 is
subtracted from the number, then the places of digits interchanged. Find the
number.
Solution : Let, x be the digit of ten's place and y be the digit of one's place.
The number is 10x + y.
Therefore, according to the first condition, x + y + 5 = 3x or, 2x y = 5 ........... (1)
According to the 2nd condition, (l0x + y ) 9 = 10y + x
or,
9x 9y = 9 or, x y = 1 .............. (2)
Subtracting equation (2) from equation (1) we get, x = 4
Putting the value of x in equation (2) we get, y = 3.
Required number is 10.4 + 3 = 40 + 3 = 43.
Answer : 43.
Example 5. The length of a rectangular region is greater than its breadth by 12
metres. If the perimeter of the rectangular region is 136 metre, find its length and
breadth.
Solution : Let, the length of the rectangular region be x metres and its breadth be
y metres. Therefore, according to the first condition,
x y = 12 ............. (1),

And according to the second condition,


2x + 2y = 136 or, x + y = 68 ............... (2)
Adding equation (2) to the equation (1), we get 2x = 80 or, x = 40
Putting x = 40 in equation (1), we get
y = 12 40 or, y = 28;
or, y = 28.
Required length = 40 metres and breadth = 28 metres.
Answer : 40 metres and 28 metres.

181

EXERCISE 4.2

1.

The sum and difference of two numbers are respectively 80 and 10. What are
these numbers ?

2.

The difference of two numbers is 45 and one is four times the other. What are
these numbers?

3.

If the first of two numbers is added to five times the second, then the sum is
52. But if the second is added to eight times the first, then the sum is 65.
What are these numbers?

4.

Father's age is 30 years more than son's age. After 10 years father will be
twice as old as his son. What are their ages?

5.

20 years ago, father was five times as old as his son. After 5 years three times
of father's age will be equal to five times of son's age. What are the present
ages of father and son?

6.

If 7 is added to the numerator of a certain fraction, then it becomes 2 and if 2


is subtracted from the denominator, then it becomes 1. Find the fraction.

7.

If 1 is added to the numerator of a certain fraction, then it becomes 2 and if 1

is added to the denominator, then it becomes 3 . Find the fraction.


8.

If 7 is added to the sum of two digits of a number consisting of two digits,


then the sum is three times the digit of ten's place. But if 18 is subtracted
from the number then places of two digits interchanged. Find the number.

9.

If 9 is added to the sum of two digits of a number consisting of two digits


then the sum is equal to six times the digit of one's place. But if nine times
the digit of one's place is subtracted from the number then the digits are
interchanged. Find the number.

10. Anik and Rafat had some mangoes. If Anik gives Rafat 30 mangoes then the
number of mangoes of Rafat becomes twice that of Anik. Again if Rafat
gives Anik 10 mangoes then it becomes three times the number of mangoes
of Anik. How many mangoes did Anik and Rafat have?
11. The length of a rectangular region is greater than its breadth by 30 metres. If
the perimeter of the rectangular region is 140 metres, find the length and
breadth of the region.

182

12. The positions of two persons are 27 kms. away. If they move at the same
time in the same direction, then they can meet after 9 hours. But if the move
towards each other then they can meet after 3 hours. Determine speed of each
of them.
13. A man and a boy can do a work in 8 days. Two men and four boys can do the
same work in 3 days. In how many days a man or a boy can do the same
work?
14. The same work which two men and two boys can do in six days can be done
by 3 men and 8 boys in three days. In how many days a man or a boy can do
that work?
15. Rahim sold to a buyer his 9 cows and 7 goats at Tk. 51000. He sold to another
customer 6 cows and 16 goats at the same price. What is the price of each of
cows and goats?
16. A man rowing in favour of current covers 70 kms and against the current he
required 70 hours to come back. Determine the speeds of rowing and current.
17. A tank is filled up in 6 minutes by two pipes. If the second pipe is closed
after 3 minutes of opening the two pipes at the same time, then it takes 5
minutes more to fill up the tank. How much time will it take to fill up the
tank by each of the pipe?
Multiple Choice Questions [Mark () on the correct answer]
1.

2.

3.

If x
(a)
(c)
If x
(a)
(c)
(i)
(ii)

y = 1, x + y = 3, then (x, y) = how much?


(1, 2)
(b) (2,1)
(1, 3)
(d) (3, 1)
2
2
y = 2a, ax +by = a +b then (x, y) = how much?
(a, b)
(b) (b, a)
(a + b, a b)
(d) (a + b, b a)
3x+2y = 5 is a simple equation.
All simultaneous equations can be solved by substitution
method.
(iii) The linear equation of one unknown variable has only one root.
Which one of the following is correct according to the above
informations?
(a) i and ii
(b) ii and iii
(c) i and iii
(d) i, ii and iii

183

Answer the questions 4, 5 in the view of the following informations:


The length of a rectangular house is 4 metre more than the breadth. The
perimeter of the house is 32 metre.
4. How much metre is the length of the house?
(a) 6
(b) 8
(c) 10
(d) 12
5. How much square metre is the area of the house?
(a) 60
(b) 96
(c) 160
(d) 192
6. The present age of the father is 30 years more than the present age
of son. After 10 years, father will be twice as old as his son. What is
the present age of the son?
(a) 40
(b) 30
(c) 20
(d) 10
7. Fathers age is four times of the age of son. The sum of the age of
father and son was 52 years before 4 years. What is the present age
of father?
(a) 32
(b) 38
(c) 48
(d) 52

1.

2.

CREATIVE QUESTIONS
A man rowing in favour of current, covers 35 km in 5 hours and against the
current he required 35 hours to come back.
(a) Express the above informations in two equations.
(b) Determine the speeds of rowing and current.
(c) How much time would he spend to go and come back for 84 km.

The sum of two digits of a number consisting two digits is 7. If 27 is


subtracted from the number then the digits interchange their position.
(a) Express the above informations in two equations.
(b) Determine the number.
(c) Add such a number with that number so that the obtained number is 1
more than five times of the sum of two digits. Determine the obtained
number.

N.M.G. -25

184

3.

A man has total 903 notes of Dolar and Euro currency. The value of the notes
in Bangladeshi taka is Tk. 77373.00 (1 dolar = Tk. 71, 1 Euro = Tk. 110).
(a) If the number of notes of Dolar is x, then express the value of that notes
in terms of x.
(b) How many notes of what type? Find the amount in Bangladeshi taka
separately.
(c) If the value of Dolar and Euro increase by Tk. 0.85 and Tk. 1.22
respectively, then what would be the percentage of increase or decrease
of the value of notes in Bangladeshi taka?

4.

Jasim and Iqbal are two friends. Tk. 152.00 is divided between them in such
a way so that three times of Iqbals share and five times of Jasims share are
equal.
(a) Express the above information in two equations.
(b) Find the amount of taka of each person.
(c) Iqbal gave some taka to Jasim, then five times of Iqbals share and three
times of Jasims share are equal. Now find the amount of taka of each of
them.

Chapter V

Drawing Of Graphs And Their Applications


5.1. Number line, Co-ordinates of a point in a Plane and Plotting of Points
By drawing geometrical figures the points of geometrical figures are expressed by
algebraic terms. Again, algebraic terms are also denoted by geometrical points.
The number corresponding to the point is the coordinate of that point. Here
coordinates of a point in the straight line and the plane are discussed.
5.1.1. Number Line
The middle point of the segment of a straight line is fixed up to correspond zero.
The length of a small segment of line considered as unit and the points situated at
the distances of 1, 2, 3, ..........units to the left and right of the point corresponding
to zero are marked. The point that corresponds to zero is called origin. The points
in the right of origin which denote 1, 2, 3, ........ are considered as positive 1, 2, 3,
........... and the points in the left of origin which denote 1, 2, 3, .............. are
considered negative 1, 2, 3, ............... The process of establishing the
relation between the points of straight line and the straight line is called
coordination of the straight line. Thus straight line coordinated is called number
line. Each real number is denoted by a point of that line. The position of the point
A in number line is completely denoted by the number related to it.

If the point, A is to the right of origin then it denotes a positive number and if it is
to the left of origin then it denotes a negative number. In the same distance from
number line, there are innumerable points out side it. The position of all these
points can not be determined by that number line only. In this case, the process of
determination of coordinates of a point in the plane is to be adopted.
Remark : For different origins or different units of length the coordinates of
points will be different. There are one to one correspondence between all real
numbers and the points on the straight line.
5.1.2 Coordination of Points on Plane.
The position of all the points on the plane can be determined by using mutually
intersecting lines. Let us consider two mutually perpendicular straight lines XOX'
and YOY' on the same plane. They intersect at the point O. The point O is the

186

origin of both the lines. Taking conveniently small segment as the unit of length,
let us make coordination of two lines, XOX' and YOY'. Now two lines XOX' and
YOY' become number lines.

The points on the right side of the origin O of the line, XOX' are positive and the
points on the left side are negative. The points on the above of the origin O of the
line, YOY' are positive and the points below are negative. XOX' is called xaxis
and YOY' is called y axis. A is on the number line XOX' and if A is the
corresponding point of 3 in that number line, then A corresponds to the pair of
numbers (3, 0). Again in the number line XOX', B the corresponding point of 1
corresponds to ( 1, 0).
If C is on the number line YOY' and C is the corresponding point of 2 in that
number line, then C corresponds to the pair of numbers, (0, 2). Again in the
number line YOY', D the corresponding point of 3 corresponds to (0, 3).

187

From A, AM and AN are drawn perpendiculars on XOX' and YOY' respectively.


The corresponding point of 3 in the line XOX' is M and the corresponding point of
2 in the line YOY' is N. A corresponds to the pair of number (3, 2).
Thus when each point, A in the plane of axes corresponds to the pair of number
(x, y), then (x, y) is called the coordinate of A.
In this case, if from A, AM is drawn perpendicular on the x axis, then M will be

5.1.3. Plotting of Points


If the coordinates of any point are
given then the placement of that
point in the plane of coordiantion is
known as plotting of the points.
Generally graph paper is used for
plotting the points.
Graph Paper
In graph paper there are some
parallel lines which have been drawn

y-axis

the corresponding point (x, o). Again, if from A, AN is drawn perpendicular on the
y a xis, then N will be the corresponding point (0, y). x of (x, y) is called
xcoordinate or abscissa and y is
called y coordinate or ordinate.

x-axis

188

at equal distances and there are also some lines perpendicular to those parallel
lines at equal distances. Thus the paper is divided into some equal small squares.
Generally, one side of the small square is considered as the unit of length. The
graph paper is also called drawing paper.
To plot points in the graph paper, two mutually perpendicular straight lines are
taken. They are denoted by XOX' and YOY'. Generally, XOX' is considered as
horizontal line and YOY' is considered as vertical line. In the direction of OX of
the line XOX' and in the direction of OY of the line YOY' are considered positive
directions.
Quadrant and Signs of Coordinates
As a result of drawing, the line XOX' or xaxis and the line YOY' or yaxis on
graph paper, the whole plane of the graph
paper has been divided into four parts and
each part is called quadrant. The first
2nd Quadrant
Ist Quadrant
quadrant by OX and OY, second quadrant
by OX' and OY, third quadrant by OX' and
OY' and the fourth quadrant by OX and
OY'.
The xcoordinate and ycoordinate of any
point in the first quadrant are both
positive.

In the second quadrant xcoordinate of


3rd Quadrant
4th Quadrant
any point is negative while ycoordinate is
positive. In the third quadrant x
coordinate and ycoordinate of any point
are both negative and in the fourth
quadrant the xcoordinate or abscissa of any point is positive, while y coordiante
or ordinate is negative.
Plotting of a Point in the Graph Paper
Let us plot a point in the graph paper with respect to the axis of XOX' and YOY'.
Suppose, the length of the side of the smallest square has been taken as unit in
case of both the axes.
Let the coordinate of the point A be (5, 6). Since the abscissa and ordinate of the
point A are both positive, hence the point is situated in the first quadrant.

189

If we move to a distance equal to


the length of 5 side of a small
square along OX to the right of
the origin O and then a distance
equal to the length of 6 sides of
small square above, then the
position of the point A is found. A
dot is placed there and A (5, 6) is
written. Let us suppose that
coordinates of the point B be (6,
5). The point B is situated in the
second quadrant. The point move
to a distance equal to the length of
6 side of a small square along OX'
to the left of the origin O and
from there move upward to a
distance equal to the length of 5
sides of a small square, then the position of the point B will be found. A dot is
placed there and B(6, 5) is written at the side of it.
Let, the coordinate of the point C be ( 4, 2). The abscissa and the ordinate are
both negative. Therefore it is situated in the third quadrant. Then position of the
point C can be located if we move along OX' to the left of the origin at a distance
equal to the length of 4 sides of a small square and from there move to a distance
equal to the length of 2 sides of a small square below. It is written as C ( 4, 2)
at the side of the dot placed there.
Let the coordinates of the point D be (3, 6). The abscissa of the point D is
positive and ordinate is negative. Therefore, it is situated in the fourth quadrant. If
we move along OX to the right of the origin O to a distance equal to the length of
3 sides of a small square and from there move to a distance equal to the length of 6
sides of a small square below, then we get the position of the point D. A dot is to
be placed there and D (3, 6) is to be written at the side of it.
Example 1. Plot the points A(3, 7), B(3, 0), C(5, 4), D (0, 4), E (4, 6), F(6, 0)
and G(0, 5) in the graph paper.
Solution : Let, XOX' and YOY represent xaxis and y axis respectively. O is the
origin. Let us suppose the length of each side of the smallest square as unit in both
the axes.

190

To find the position of the point A, we have to move to the right of the origin O at
a distance equal to the length of 3 sides of a small square along OX and then at
distance equal to the length of 7 sides of a small square above OX. Thus the point
found is A. To find the point B it is required to move to the right of the origin O to
a distance equal to the length of 3 sides of a small square along OX. (Here as the
ordinate is zero it is not required to move below or above OX).
To plot the point C, we have to
move to the left of the origin O at
distance equal to the length of 5
sides of small square along OX and
then to move above OX to a
distance equal to the length of 4
3.0
sides of small square.
To plot the point D, it is required to
move above from O along OY to a
distance equal to the length of 4
sides of small square (Here the
abscissa is zero and the abscissa of
the origin O is also zero. As the
ordinate is positive, we are to move
above directly from O along OY).
To plot the point E, we have to move along OX' left from the origin O to a
distance equal to the length of 4 sides of small square and then to move below OX'
to distance equal to the length of 6 sides of small square.
To plot the point F, the point which is obtained by moving to a distance equal to
the length of 6 sides of small square to the left of the origin along OX (Here as the
ordinate is zero, it is not required to move above or below from OX').
To plot the point G, the point which is obtained by moving to a distance equal to
the length 5 sides of a small square below the origin O along OY is the point G.
Example 2. Plot the points (1, 3), (1, 1) and (3, 1) in the graph paper. Also
show that, they are collinear (i.e. the points lie on the same straight lines).
Solution : Let XOX' and YOY' represent xaxis and yaxis respectively and O is
the origin. Let us suppose that the length of two sides of a small square is taken as
unit along the xaxis and the length of three sides of a small square is taken as unit
along the yaxis. Now, for abscissa 1 of the point (1, 3) we are to consider the

191

length equal to (1 2), or 2 sides of small square and for the ordinate 3 we have to
consider the length equal to (3 3), or 9 sides of small square.
To plot the point (1, 3), the point which
is obtained by moving to a distance
equal to the length of 2 sides of a small
square to the right from the origin O
along OX and then to a distance equal to
the length 9 sides of a small square is
the point (1, 3). To plot the point (1, 1),
the point obtained by moving to a
distance equal to the length of (1 2), or
2 small sides of squares to the left from
the origin O along OX' and to distance
equal to the length of (1 3) or 3 sides
of a small square is the point (1, 1). To
plot the point (3, 1), the point
obtained by moving to a distance equal
to the length of (3 2), or 6 sides of a small square to the left from the origin O
along OX' and then to a distance equal to the length of (1 3), or 3 sides of small
square below is the point (3, 1).
Now let us join the first and third points by a straight line. The other point lies on
this line. Therefore, the given point are collinear.
Remark : (a) Any coordinate of a point is to be multiplied by that number which
is equal to the length of number of sides of a small square that is considered as
unit. Then the points are to be placed in the graph paper according to the co
ordinates obtained by these multiplication.
(b) The unit is to be taken so that the points can be plotted in graph paper with the
help of whole numbers if any co-ordinate is fraction.
EXERCISE 5.1

1. Plot the following points in the graph paper and mention the quadrants in
which they lie:
A (2, 5); B (2, 7); C ( 4, 0); D (4, 5); E ( 5, 6); F (0,8); G (0,0); H (0, 6);
I (8,0); J (3,9) and K (0,7) .
N.M.G. -26

192

2. Taking the length of two sides of smallest square as unit, plot the following
points in the graph paper.
5 9

A (3, 4); B ( 3, 5); C ( 0, 6); D ( 3, 4); E ( 3, 0); F (0, 3) and G (2 , 2 )


3. Taking the length of three sides of the smallest square as unit, plot the
following points in the graph paper:
2 4

A ( 2, 3); B (3 , 3 ); C ( 0, 6); D ( 3, 5); E ( 5, 0) and F (3, 4).


4. Plot the following points in the graph paper. Show that they lie on the same
straight line.
(a) A (2, 1), B (6, 3) and C (2, 5)
(b) A (1, 1), B (2, 7) and C (5, 7)
(c) A (3, 1), B (6, 3) and C (3, 3)
(d) A (1, 3), B (0, 0) and C (1,3)
1 1
1 5
1 7
(e) A 2 , 4 ; B 2 , 4 and C 2 , 4 ;

5. Show that the straight line joining the points (3, 6) and (2, 4) passes
through the origin.
6. Find the coordinates of the intersecting point at which the straight line
joining the points (2, 4) and ( 4, 5) intersects with the straight line joining
the points (1,0) and (5, 4).
7. Find the coordinates
intersecting
point at which the straight line
1 of the 1
1
joining the points (3 , 3) and ( 2 , 2 ) intersects with the straight line joining
the points (3, 3) and (2, 2).
5.2. Graphs of Linear Simultaneous Equations

The existing relation between two unknown quantities, x and y can be expressed
by a figure. This figure is called graph of that relation.
The linear equation x y = 2 is satisfied by the pairs of numbers (3, 1), (4, 2),
(5, 3), (1, 3), (1, 1) ....... etc. of (x, y). These pairs of numbers are considered as
coordinates and their corresponding points are plotted. Then the figure formed by
joining these points is the graph. The relation between x and y expressed by the
above mentioned equation can be expressed by this graph. There are innumerable
points in the graph of such a equation. If some points of the graph are plotted then
a clear idea regarding the graph is found. The graph can be drawn if the points are
joined only.

193

Of the equation x y = 2 for different values of x, corresponding definite values of


y can be obtained. Again for different values of y, different definite values of x can
be determined. The values thus obtained can be written in the tabulated form as
given below :
x

Remark : Generally these values are to be taken as whole numbers.


Now let us draw axes, XOX' and YOY' in the graph paper. Let, in both the axes,
the length of each side of the smallest
square be unit. Let us plot the points
A, B, C, D, E and F which
correspond to the pairs of numbers
(3, 1), (4, 2) (5, 3), (1,1), (0, 2) and
(1, 3) mentioned above. If the
points are joined then the straight line
CF is found. This line, CF is the
graph of the equation, x y = 2
Remark : The graph of any linear
simulations equation containing two
unknown quantities is a straight line.
The coordinates of each point of this
line satisfy the equation. It is
sufficient to draw this graph from two
points only, but it is better to take three or more points.
Example 1. Draw the graph of the equation, 3x + 2y = 5.
5 3x
5 3x
Solution : 3x + 2y = 5. or, 2y = 5 3x or, y = 2 y = 2
The coordinates of some points of the graph are determined from this relation :

194

Let, XOX and YOY be x - axis and y axis respectively and O be the origin. In
both the axes, the length of each side of
the smallest square is taken to be unit.
The points (1, 1), (3, 2), (1, 4) and
(3, 7) are plotted in the graph paper.
By joining these points a straight line is
obtained. This is the graph of the
equation, 3x + 2y = 5
5.3. Solution of Linear Simultaneous
Equations containing Two Terms
with the help of Graph.
There are two linear equations in the
linear simultaneous equations containing
two unknown terms.

If graphs are drawn for two linear equations then two straight lines are obtained.
The point of intersection lies on both the lines. The x 1 coordinate and
y coordinate of this point of Intersection will be the roots of the given linear
simultaneous equations. These two equations will be simultaneously
satisfied by the values of x and y. Therefore, the only solution of the pair of linear
simultaneous equations is this abscissa and this ordinate of two straight lines.
Remark : There will be no solution of the given simultaneous equations if the two
straight lines are parallel.
Example 2: Solve with the help of graphs : 2x + 3y = 8
3x 4y = 5
8 2x
Solution : From the given first equation we get, 3y = 8 2x or , y = 3
Let us find the coordinates of some points of the graph of this equation.

Again the second equation, we get, 3x = 4y 5

195

4y 5
3
or,4y = 3x + 5
3x + 5
or, y = 4
Let us find the coordinates of some points of the graph of this equation :
or, x =

Let, XOX' and YOY' be x-axis and y-axis respectively and O be the origin. Let, in
both the axes, the length of each side of the smallest square be unit.
Let
us
plot
the
points
corresponding to (5, 6), (4, 0)
and (7, 2) in the graph paper. By
joining these points, a straight line
is obtained. Let us extend the
straight line in both directions.
Therefore, this is the graph of the
equation, 2x + 3y = 8.
Again, let us plot the points
corresponding to (3, 1), (7, 4)
and (5, 5)in the graph paper.
'

'

By joining these points, a straight


line is obtained. Let the straight
line be extended in both
directions. Therefore, this is the
graph of the equation 3x 4y =
5. This straight line intersects the previous straight line at a point A. The point A is
the solution point of both the lines. The coordinates of this point satisfy both the
equations. It is found from the graph that the abscissa and ordinate of the point, A
are 1 and 2 respectively. Hence the required solution, x = 1, y = 2,
Answer : (x, y) = (1, 2).

196

5.3.1. Solution of Linear Equations with the help of Graph


In order to find the solution of the linear equation, 5x + 7 = 8x + 1 with the help of
graph the following rules are to be followed. First of all, it is to be expressed in
linear simultaneous equations. Each of both the sides is taken to be equal to y, then
two equations y = 5x + 7 and y = 8x + 1 are obtained. If the graphs of these two
equations are drawn. then two mutually intersecting straight lines are obtained. As
the point of intersection lies on both the lines, their coordinates will satisfy both
the equations. Hence the abscissa of the point of intersection will be the root of the
given linear equation, i.e. the value of x will be the solution of the equation.
Example 3. Solve the equation, 2x + 1 = 3x 2 with the help of graph.
Solution : Let each side of 2x + 1 = 3x 2 be equal to y.
Therefore, y = 2x + 1 and y = 3x 2.
Let us find the coordinates of some points of the graph of the equation, y = 2x + l :
x

Again, let us find the coordinates of some points of the graph of the equation,
y = 3x 2
x

11

Let XOX' and YOY' be x axis and yaxis respectively and O be the origin.
Let in both the axes the length of each
side of the smallest square be unit.
Let us plot the points corresponding
to (1, 3), (1, 1) and (4, 9) in the
graph paper. By joining these points,
a straight line is obatined.
Let the straight line be extended in
both the directions.
This is the graph of the equation, y =
2x + 1. Again, let us plot the points
corresponding to (2, 4), (1, 5) and
(3, 11) in the graph paper. By joining these points, a straight line is obtained.

197

Let us extend this straight line in both the directions. This is the graph of the
equation, y = 3x 2. This straight line intersects the previous line at the point A.
The point A is the common point of both lines. The coordinates of the point satisfy
both the equations. It appears from the graph that the abscissa of the point, A is 3.
Hence, the required solution, x = 3.
Answer : x = 3.
EXERCISE 5.2
1. Draw the graphs of the following equations :
i) x + y = 4: ii) 2x + y = 8; iii) 4x + 3y = 6; iv) 2x y = 10;
v) 3x 2y = 9; vi) y = 3x 7; vii) 3x + 4y = 0: viii) x 2y 5 = 0;
ix) 2x = 63y; (x) 2x 5y + 12 = 0
2. Solve with the help of graph :
i) x + y = 5
ii) x + 4y = 11;
iii) 3x y = 5
x y = 3,
4x y =10
3x 2y = 4
iv) 3x + 2y = 12,
v) 3x + 2y = 6.
vi) 3x + 4y = 1
2x 3y = 5
4x + y = 8.
x + 2y = 1
vii) 3x 4y = 0,
viii) 4x + 3y = 8.
ix) 5x 3 = 8.
2x 3y = 1
3x + 2y = 6.
10x + 6y = 4.
3. Solve with the help of graph :
i) 3x + 4 = 5x;
ii) 3x + 2 = x 2:
iii) 7x 5 = 4x + 1;
iv) 5x 3 = x + 1;
v) 8x 3 = 5x 3;
vi) 3x 7 = 3 2x;
1
1
vii) 1 3x = 7 3x;
viii) 2 x 1 = 3 x + 1
Multiple Choice Questions [Make () on the correct answer]
1.

2.

3.

In which quadrant the point P(-3,

1
) is situated?
2

(a) 1st
(b) 2nd
(c) 3rd
(d) 4th
In how many unit of distance the poins A (2,3) is situated from the
x-axis?
(a) 1
(b) 2
(c) 3
(d) 4
Which line joining the following pair of points passes through the
origin?
(a) (2,0), (0,2)
(b) (-2, 0), (0, 2)
(c) (1, 0), (-1, 0)
(d) (1,0), (0,1)

198

4.
C(0,6)

A(0,4)

(0,0)

D(2,0)

B(4,0)

Which one of the following indicates the point of intersection of the


lines AB and CD?
(a) (3, 1)
(b) (2, 3)
(c) (3, 2)
(d) (1, 3)
5.

(i)

1
6

The point P(-2, ) is situated in the 3rd quadrant.

(ii) There is a one-one correspondence between each point on the


real line and each real number.
(iii) Using two straight lines it is not possible to determine the
position of all points.
Which one of the following is correct in the basis of the above
informations?
(a) i and ii
(b) i and iii
(c) ii and iii
(d) i, ii and iii

199

A(0,5)
D(5,5)

O
(0,0)

B (5,0)

The equation of the straight line AB is x+y = 5


Answer the questions 6-8 on the basis of the above informations.
6.

7.

8.

Which one of the following points is situated on the line x+y=5?


(a) (1,3)
(b) (0,0)
(c) (-1, -3)
(d) (6, -1)
Which one of the following is the co-ordinate of intersecting point
of lines AB and OD?
5
2
5
,0
2

5 5
2 2

(a) 0,

(b) ,

(c)

(d) (5, 5)

What type of triangle is shown in the figure above?


(a) Equilateral
(b) isosceles
(c) Right-angled
(d) acute-angled

CREATIVE QUESTIONS
1.

Observe the following points:


A(2, -1), B(1, -1), C(-2, -5), D(5,7)
(a) In which quadrant the two points A and D are situated?
(b) Show that the points P(6,3) and Q (-4, -5) are situated on the
line AC and BD respectively using graph.
(c) Find the co-ordinates of the intersecting points of the two lines
with the axes.

N.M.G. -27

200

2.

Observe the following equation:


x+3y = 3
(a) Find the co-ordinates of three points from the equation.
(b) Show that the obtained points are situated on the same line.
(c) Find the point of intersection of the obtained line and lines
joining the points (2,4) and (-4, -5); (-1,0) and (-5, -4).

3.

Observe the two equations:


2x + y = 3 ------ (i)
3x + 2y = 5 ------ (ii)
(a) Show that, the root of the equations (i) and (ii) is (1,1).
(b) Find the co-ordinates of three points from each graph of each
equation.
(c) Draw the graphs of the two equations and show that the area of
the triangle formed by the line of equation (i) with the axes is 9
square unit.

Junior Secondary Mathematics


GEOMETRY

Chapter I

Parallelism of lines, Congruency


Of triangles
1.1 Parallelism of Lines
If any straight line does not intersect the other when each of two or more straight
lines lying in the same plane are extended in both the directions, then they are said
to be parallel lines to one another.

AB and CD are two lines parallel to each other. The parallelism is denoted by the
symbol . Here: AB CD, If any two line segments are taken from two parallel
lines, then those line segments are parallel to each other. The perpendicular
distances from any point of line of two parallel lines to the other line are always
equal to another. The perpendicular distance or distance between AB and CD is
PQ. Only one straight line can be drawn parallel to a straight line through if point
not lying on this line.
Uclid's Postulates
If a point is not situated on the same line, then one and only one parallel line draw
through this point.
Playfaire's Postulates.
If two straight lines intersect then both the lines can not be parallel of a third
straight line.
1.2 Congruency of Triangles
Each triangle has six parts three sides and three angles. Of two triangles, if one is
placed on the other in a proper way and if the sides and the angles of two triangles
coincide completely, then those two triangles are congruent. Two congruent
triangles will be of the same size.

203

Two triangles will be congruent if the following parts of those two triangles are
respectively equal.
(a) Two sides and their included angle
(b) Three sides
(c) Two angles and one side
(d) One angle is a right angle, hypotenuse and one side.
Remarks : (a) Two triangles may not be congruent if three angles of one are
respectively equal to that of the other.
Three angles of innumerable triangles of different sizes may be equal. These
triangles are called similar triangles.
(b) If two triangles are congruent, then their areas will also be equal. But two
triangles may not be congruent though their areas are equal.
A
D

B
1.3 Symbols
The following symbols are used :
Symbol
Meaning

Hence, therefore
Q
=

Since
Equal to

angle
triangle
Congruent

Symbol
>

Meaning
greater than

<

less than
Greater than or equal to
less than or equal to
not equal to
perpendicular to
parallel to

1.4 For the same of continuity, the theorems studied in the previous class and
discussed again and proofs of some theorems are also included.

204

Theorem 1
If another straight line meets a line segment at its end point then the sum of two
adjacent angles thus formed is equal to two right angles.

The line AB meets the line segment CO at its end point O. Here AOC and
BOC are two adjacent angles AOC + BOC = Two right angles.
Theorem 2
If the sum of two abjacent angles is equal to two right angles, then their two
exterior sides lie in the same straight line.

AOC and BOC are two adjacent angles and AOC + BOC = two right
angles. OA and OB are their exterior sides. Hence OA and OB lie in the same
straight line.
Theorem 3
If two straight lines intersect each other, then the vertically opposite angles
formed at the point of intersection are equal to each other.

The lines AB and CD intersect at O. Hence, AOD = vericaliy opposite BOC


and AOC = vertically opposite BOD.

205

Theorem 4
If a straight line intersects two other straight Lines,
(a) two alternate angles will be equal to each other.
(b) two corresponding angles will be equal to each other.
(c) the sum of the two interior angles in the same side of the bisector will
be equal to right angles.

Proposition : Let the straight line EF intersects the parallel straight lines AB and
CD at G and H respectively, (a) It is to be proved that AGH = alternate GHD.
Proof : If AGH is not equal to GHD, then let us suppose that, KGH =
GHD. They are alternate angles.
Hence KG and CD are parallel. But AB and CD or, AG and CD have been taken
to be parallel to one another.

Each of AG and KG is parallel to CD though they intersect. But it is impossible


according to Playfaire's Postulate, Therefore, AGH and GHD are not unequal,
i.e. AGH = GHD.
(b) It is required to prove that, EGB = GHD
Proof : EGB = vertically oopposite AGH and AGH = alternate GHD
EGB =GHD.
(c) It is required to prove that, BHG + GHD = two right angles.
Proof: EGB+ BGH = two right angles, but EGB = corresponding GHD.
BGH + GHD = two right angles (proved).
Corollary. If a straight line is perpendicular to any one of two parallel lines,
then it is perpendicular to the other.

206

Theorem 5
If a straight line intersects another two straight lines and if (a) the alternate
angles are equal to each other or, (b) the corresponding angles are equal to
each other.
or, (c) the sum of the two enterior angles in the same side of the bisector be
equal to those two right angles then two lines are parallel.
.*.

'.

'

(a) Proposition : Let the straight line EF intersects AB and CD at G and H


respectively so that AGH = alternate GHD and BGH = alternate GHC. It is
required to prove that, AB and CD are parallel.
Proof : If AB and CD are not parallel then draw PQ straight line through the point
E
G and to the line CD (By uclid's postulate)
Q
Now PQ CD
G
A

P
C
D
PGH = alternate DHG
H
But given, AGH = DHG
PGH =AGH
F
But it is impossible, because between two angels one is the part of the other. So
the straight line AB and CD are parallel.
b) Proposition : Let, the straight line EF intersects AB and CD at the points. G
and H respectively so that it becomes exterior EGB = interior opposite GHD.
It is required to prove that, AB and CD are parallel.

Proof : AGH = EGB (being vertically opposite angles) and EGB = GHD
(given)
AGH = GHD. But they are alternate angles.
AB and CD are parallel.
(C) Proposition : Let the straight line EF intersects the straight lines AB and CD
at the points G and H respectively, so that in the same side of EF, interior BGH

207

+ interior GHD = two right angles.


It is required to prove that, AB and CD are parallel.

Proof: Now, AGH + BGH = two right angles (being adjacent angles). BGH
+ GHD = two right angles (given)
AGH + BGH = BGH + GHD.
Now subtracting BGH from both the sides we get, AGH = GHD. But
they are alternate angles.
AB and CD are parallel (proved).
Corollary . If each of two or more straight lines is perpendicular to given straight
line then they are parallel to one another.
Theorem 6
The straight lines which are parallel to the same straight line are parallel to.
one another.

The straight lines AB and EF are parallel and CD and EF are parallel. Hence,
the straight lines AB and CD are parallel.

N.M.G. -28

208

Theorem 7
If in two triangles each of two sides of one is equal to the corresponding side
of the other respectively and the included angle of those sides of one is equal
to that of the other, then the triangles are congruent.

Proposition: Let, in ABC and DEF, AB = DE , AC = DF and included A =


included D. It is required to prove that, ABC DEF.
Proof: Let us place ABC upon DEF so that A falls upon D. The side AB lies
along the side DE and the point C falls on that side at which side of DE, F lies.
Now, since AB = DE, the point B must fall upon the point E. Again, since the side
AB lies completely on the side DE and A = D, the side AC will be along the
side DF. Now, since AC = DF, the point C must fall upon the point F. Since the
points B and C fall upon E and F respectively, the side BC must coincide
completely with EF. Therefore, ABC is superimposed on DEF.
ABC DEF (proved).
Theorem 8
If two sides of a triangle are equal to each other, then their opposite angles
are also equal to each other.

The sides AB and AC of the triangles ABC are equal to


each other.
Hence, ACB = ABC.
Theorem 9
If two angles of any triangle are equal to each other, then their opposite sides
are equal to each other.

In the ABC, ABC = ACB.


Hence, AC = AB

209

Theorem 10
If three sides of a triangle are respectively equal to the corresponding
three sides of another triangle, then the triangles are congruent.

Proposition : Let, in ABC and DEF, AB = DE, AC = DF and BC = EF.


It is required to prove that, ABC DEF.
Proof: Let, BC and EF are the greatest sides of ABC and DEF respectively.
Let us place ABC upon DEF in such a way that B falls upon E, the side BC lies
along the side EF and the point A falls on the opposite of that side at which side of
EF, D lies. Now, since BC = EF, the point C falls upon the point F.
Let, the new position of the point A be denoted by G. As a result GEF is the new
position of ABC.
Hence, EG = BA, FG = CA and EGF = BAC. Let us join D,G.

Now, in EGD, EG = ED [Q EG = BA = ED] EDG = EGD.


Again, in FGD, FG = FD, [Q FG = CA = FD] FDG = FGD.
Hence, EDG + FDG = EGD + FGD or, EDF = EGF
i.e. BAC = EDF.
Therefore, in ABC and DEF, AB = DE, AC = DF
and included BAC = included EDF
A
ABC DEF. (proved)
D
Remark : If three angles of a triangle are equal to three
corresponding angles of another triangle, then the
triangles may not be congruent.
AB F DE, AC F DF and BC F EF.
E
Here, three angles of ABC are equal to three B
corresponding angles of DEF.

210

Theorem 11
If one side is greater than another side of a triangle the angle opposite to the
greater side is greater than the angle opposite to the smaller side.
Proposition : Let, in ABC, AC >AB
It is required to prove that, ABC > ACB.
Construction : Let us cut off AD from AC
such that, AD = AB and join B and D.
Proof : In ABD, AD = AB
ABD = ADB [Theorem 8]
But in BDC, exterior ADB > BCD
ABD > BCD or, ABD > ACB.

But ABC > ABD.[Q ABD is a part of ABC.]


Hence, ABC > ACB (proved).

Theorem 12
If one angle is greater than another angle of a triangle, the side opposite to the
greater angle is greater than side opposite to the smaller angle.

In ABC, ABC > ACB.


Hence, AC > AB.

Theorem 13
The sum of any two sides of a triangle is greater than its third side.

In ABC, the side BC is greater than both AB


and AC.
Therefore, BC is the greatest side.
Then, AB + AC > BC.

211

Theorem 14
Of all the line segments that are drawn to a straight line from any external
point of the straight line, then perpendicular is the shortest.

OC is the perpendicular to AB from any external point O of AB and OP is any


other line segment from O to any point on AB. Then, OC < OP.

Theorem 15
The sum of three angles of a triangle is equal to two right angles.
ABC, ACB and BAC are the
three angles of ABC.
Hence, ABC + ACB + BAC =
two right angles.

Theorem 16
If two angles and a sides of a triangle are equal to the corresponding
angles and side of another triangle respectively, then two triangles are
congruent.
A
D

Proposition : Let, in ABC and DEF, A = D, B = E


and the side BC = corresponding side EF.
It is required to prove that, ABC DEF.
Proof : In ABC, A + B + C = 180 [ Theorem 15]

212

and in DEF, D + E + F = 180 [Theorem 15]


A + B + C = D + E + F.
... A = D and B = E,
C = F.
Now, place ABC on DEF in such a way that the point B lies on E, the side BC
lies along EF and the point A falls on that side at which side of EF, D lies.
Since, BC = EF, therefore, the point C falls on the point F. Now since B = E,
the side BA will lie along the side ED and since C = F. the side CA will lie
along FD. Hence, the common point A of the sides BA and CA will fall on the
common point D of the sides ED and FD.
Hence, ABC is superimposed on DEF.
ABC DEF (proved).

Theorem 17
If two hypotenuses of two right angled triangles are equal to each other and
also one side of one of those triangles is equal to the corresponding side of
another, then two triangles are congruent

Proposition : Let, in the right angle triangles ABC and DEF,


the hypotenuse AC = hypotenuse DF and AB = DE.
It is required to prove that, ABC DEF.
Proof: Let us place ABC upon DEF in such a way that the point B lies on the
point E, the side BA lies along the side ED and the point C falls on the opposite of
that side at which side of DE, F lies.
Let, the new position of the point C be denoted by G.

213

Since, AB = DE, therefore, the point A lies on the point D.


Hence, DEG is the new position of ABC.
Therefore, DG = AC = DF,
DGE = ACB and DEG = ABC = one right angle.
Again, since DEF + DEG = one right angle + one right angle = two right
angle, therefore, EF and EG lie on the same straight line [Theorem 2]
Now, since in DGF, DG = DF, therefore, DFE = DGE. [Theorem 8]
Hence, DFE = ACB.
Now, in ABC and DEF, ABC = DEF, [each being one right angle]
ACB = DFE and side AB = corresponding side DE.
ABC DEF (proved).

EXERCISE 1
1.
Given that, in ABC, ABC = ACB
and DE F BC;
prove that, ADE = AED.
2.
Given that, DC F AB and DC = AB;
prove that, AD = BC.
3.
Given that, AB and CD intersect each
other at the point O; AC CD and BD
CD,
prove that, CAO = OBD.
4.
Given that, AD is a median of ABC;
prove that, AB + AC > 2AD.

214

5.

Given that, in ABC, AB = AC and D


is any point on the extension of BC;
prove that, ABD > ADB.

6.
In ABC, BAC = ABC : D is any
point on BC;
prove that, CDA > CAD.
7.
In ABC, AB = AC
and BAD = CAE;
Prove that, ADB ACE.
8.

In ABC, AB = AC and BE and CD are


the bisectors of ABC and ACB
respectively,
prove that, BDC BCE.

9.

If any straight line intersects two parallel straight lines then prove that, two
bisectors of two alternate angles are parallel to each other.
10. Prove that, sum of any two sides of a triangle is greater than twice its
median drawn on the third side.
1

11. D is a point inside ABC. Prove that, AD + BD + CD > 2 (AB + AC + BC).


12. The median AD of ABC is half of BC. Prove that, A is a right angle.
13. If one of two acute angles of a right angled triangle is twice the other, prove
that, the smallest side is half of hypotenuse.
14. Prove that, the distances of any point on the bisector of any angle from the
sides are equidistant.
15. Prove that, two perpendiculars drawn from two terminal points of the base of
an isosceles triangle to their opposite sides are equal to each other.

215

16. Prove that, if three perpendiculars drawn from three angular points of a
triangle to their opposite sides are equal to one another the triangle is an
equilateral triangle.
17. Prove that, if one angle of a triangle is equal to the sum of its other
two angles then the triangle is a right angled triangle.
18. Prove that, the sum of three medians of a triangle is smaller than its
perimeter.
19. Standing on the same bank of the river, find its breadth.
Multiple Choice Questions [Mark () on the correct answer]
1.

(i)

If two straight lines intersect one another then they cannot be


parallel with a third straight line.
(ii) More than one straight line can be drawn through two
particular points.
(iii) Two similar triangles always will be congruent.
Which one of the following is correct in view of the above
informations?
(a) i
(b) ii
(c) ii and iii
(d) i, ii and iii

2.

Which one of the following indicates the sign of congruent?


(a) =
(b)
(d) ||
(c)

3.
A

If DE || BC in ABC and ADE =


60, then which one of the following
is the value of B?

(a) 30
(c) 60
N.M.G. -29

(b) 45
(d) 90

216

Answer the questions (4 6) on the basis of the following informations:


P

In the figure, AB || CD || EF,


PQ is the intersector of them
and MNF = 60, MLR =
45.

A
M

C
E

B
R

D
F

4.

5.

6.

7.

8.

9.

Which one of the following is the value of LMR?


(b) 45
(a) 30
(c) 60
(d) 120
Which one of the following is the value of ALM?
(b) 45
(a) 30
(c) 60
(d) 90
Which one of the following is the value of LRD?
(a) 45
(b) 90
(d) 120
(c) 105
By which three line segments of the following it is impossible to
draw a triangle?
(a) 5 cm, 8 cm, 3 cm,
(b) 4 cm, 5 cm, 3 cm
(c) 2.5 cm, 3.5 cm, 3 cm
(d) 6.5 cm, 4.5 cm, 5.5 cm.
If ACB > ABC in ABC then, which one of the following is
correct?
(a) AC > AB
(b) AC AB
(c) AB > AC
(d) AB = AC
OP is the perpendicular to MN from the external point O and OC,
OD are two line segments. Which one of the following is correct?
(a) OP < OC
(b) OP > OC
(c) OD < OP
(d) OD > OP

217

10. Which one of the following is the condition for congruence of two
triangles?
(a) Two sides and one angle equal,
(b) Three angles are equal to three angles of another traingle.
(c) Two angles and one similar side are equal.
(d) The area of two triangles are equal.

CREATIVE QUESTIONS
1.

2.

3.

AB || CD, PQ is the intersector. The line PQ intersects AB and CD


at the points E and F respectively.
(a) Draw the figure on the basis of the above informations and
write two alternate angles.
(b) Using the above figure show that, BEP = PFD
(c) If two bisectors of BEF and DFE intersect at the point G
then prove that, EGF = 1 right angle.
ABC is a triangle. AD, BE and CF are medians.
(a) Draw a figure with brief descriptions on the basis of the above
informations.
(b) Show that, AB + AC > 2AD
(c) If AB = AC, then prove that, BCF BCE
The median AD of ABC is the half of BC.
(a) Draw the figure in view of the above informations.
(b) Prove that, A = 1 right angle.
(c) If AD BC, then prove that, AB = AC.

Chapter II

Theorems Related to Quadrilaterals


2.1 Quadrilateral
The closed figure bounded by four line segments is a quadrilateral. The region is
quadrilateral region.
The quadrilateral has four sides. The four line segments by which the region is
bounded are the sides of the quadrilateral.

Any three points of the points A, B, C, D are not collinear. The union of four line
segments AB, BC, CD and DA is the quadrilateral ABCD. AB, BC, CD and DA
are the sides of the quadrilateral. The points A, B, C and D are the angular points
of vertices. ABC, BCD, CDA and DAB are four angles of the
quadrilateral. The quadrilateral ABCD is sometimes denoted by the symbol "
ABCD".
The vertices A and B are the opposite vertices of C and D respectively. AB and
CD are called opposite sides to each other and AD and BC are called opposite
sides to each other. Two sides which meet at the same vertex are adjacent sides.
AB and BC are two adjacent sides. The line segments AC and BD are the
diagonals of
ABCD.
2. 2. Different kinds of Quadrilaterals
Parallelogram : If the opposite sides of a quadrilateral are parallel to each other,
then it is called a parallelogram. The region bounded by the parallelogram is called
parallelogram region.

Parallelogram

Rectangle

Rectangle : The quadrilateral in which each of all the angles be a right angle

219

is called a rectangularquadrilateral or rectangle. The region bounded by a


rectangle is called a rectangular region.
Remark : A rectangle is also a parallelogram. If one of the angles of a
parallelogram is a right angle, the parallelogram will be a rectangle.
Square : The quadrilateral whose sides are equal to each other and. each of its
angles is a right angle is called a square. The region bounded by a square is
called a square region.

Square

Rhombus

Remark : If the sides of the rectangle are equal to each other then it becomes
square.
Rhombus : The quadrilateral whose sides are equal to each other but none of its
angles is a right angle is called a rhombus.
Remark : If the sides of a parallelogram are equal to each other then it becomes a
rhombus.
Trapezium : The quadrilateral whose two opposite sides are parallel to each other
but its other two sides are not parallel to each other is called trapezium.

Trapezium

Quadrilateral

Perimeter : The total length of the line segments of a certain bounded region is
called its perimeter.
The perimeter of quadrilateral ABCD is equal to the length of
(AB + BC + CD + DA).

220

2.3. Two Theorems Related To Quadrilaterals Are Given Below :


Theorem 18
If two sides of a quadrilateral are respectively equal and parallel to each
other, the other two sides are also respectively equal and parallel to each
other.

Proposition : Let, the sides AB and DC of the quadrilateral ABCD are


respectively equal and parallel to each other. It is required to prove that, the sides
BC and AD are respectively equal and parallel to each other.
Construction : Let us join A and C.
Proof : Since AB and DC are parallel to each other and AC is their interceptor,
hence, BAC = DAC. [ being alternate angles]
Now, in ABC and ADC, AB = DC, AC is their common side
and included BAC = included DCA.
Therefore, ABC ADC [Theorem 7]
So, BC = AD and ACB = CAD.
.
Now since two alternate angles produced by the interceptor AC of the sides BC
and AD are equal to each other, the sides BC and AD are parallel to each other.
Therefore, two sides BC and AD are respectively equal and parallel to each other.
[proved]
Corollary : If two sides of a quadrilateral are respectively equal and parallel to
each other, then the quadrilateral is a parallelogram.
Theorem 19
The opposite sides and opposite angles of a parallelogram are respectively
equal to each other and each diagonal divides the parallelogram into two
congruent triangles.

221

Proposition : Let, ABCD be a parallelogram and AC and BD are its two


diagonals. It is required to prove that,
(a) The side AB = the side CD, the side AD = the side BC
(b) BAD= BCD, ABC = ADC;
(c) ABC ADC, ABD BCD.
Proof : Since AB CD and AC is their interceptor,
therefore, BAC = ACD, [being alternate angles]
Again, since BC AD and AC is their interceptor,
therefore, ACB = DAC ; [being alternate angles]
Now, in ABC and ADC,
BAC = ACD, ACB = DAC and AC is the common side.
ABC ADC
[Theorem. 16]
Therefore, AB = CD, BC = AD and ABC = ADC.
Similarly, it can be proved that, ABD BDC.
Therefore, BAD = BCD [proved]
Corollary 1. If one angle of a parallelogram is a right angle, then each of its
angels will be a right angle.

Let, in the parallelogram ABCD, A is a right angle.


Since AB and DC are parallel to each other and AD is their interceptor,
therefore, A + D = 2 right angle [Theorem 4]
Hence, D = one right angle.

222

Since ABCD is a parallelogram, therefore, C = A = one right angle


and B = D = one right angle.
Corollary 2. If one angle of a parallelogram is a right angle, then it is a rectangle.
Corollary 3. If two adjacent sides of a rectangle are equal to each other, then it is
a square.
Corollary 4. Two diagonals of a parallelogram bisect each other.

Let, two diagonals AC and BD of the parallelogram ABCD intersect each other at
O. It is required to prove that, AO.= CO, BO = DO.
.
Proof : Since AB and DC are parallel to each other, AC and BD are their
interceptor, therefore, BAC = alternate ACD and BDC = alternate ABD.
Now in AOB and COD,
OAB = OCD, OBA = ODC and AB = DC.
Therefore, AOB COD [ Thorem 16]
AO = CO and BO = DO.
.
Corollary 5. The line segment found by joining points of two sides of a triangle is
parallel to the third side and it is half length of the third side.

The points D and E are respectively the points of the sides AB and AC of ABC.
Let us join D and E and produced so that DE = EF. C and F are joined.
ADE CEF, CF || BD, DF || BC and DF = BC.
1

DE = 2 BC and DE || BC.

223

EXERCISE 2
1.

Given that, the median CO of ABC is produced


upto D so that CO = DO.
Prove that, ACBD is a parallelogram.

2.

Given that, in the quadrilateral ABCD, AB = CD


and ABD = BDC.
Prove that, ABCD is a parallelogram.

3.

Given that, in the parallelogram ABCD, BM is the


bisector of ABC and DN is the bisector of ADC.
Prove that, BMDN is a parallelogram.

4.

Given that, in the parallelogram ABCD,


AP = CR and DS = QB;
prove that, PQRS is a parallelogram.

5.

Given that, in the parallelogram ABCD, AM and


CN are both perpendicular to DB;
prove that, ANCM is a parallelogram.

6.
7.

Prove that, sum of four angles of a quadrilateral is four right angles.


Prove that, if opposite angles of a quadrilateral are equal to each other, then it
is a parallelogram.
8. Prove that, if two diagonals of a parallelogram are equal to each other, then it
is a rectangle.
9. Prove that, if two diagonals of a quadrilateral are equal to each other and
bisect each other at right angle, then it is a square.
10. Prove that, two diagonals of rhombus bisect each other at right angle.
11. Prove that, if the quadrilateral formed by joining the mid-points of adjacent
sides of a rectangle, then it is a rhombus.
12. Prove that, the bisector of any two opposite angles of a parallelogram are
parallel to each other.
13. Prove that, the bisectors of any two adjacent angles of a parallelogram are
perpendicular to each other.
N.M.G. -30

224

Multiple Choice Questions [Mark () on the correct answer]


1.
In the figure, AD = BC and AD || BC.
Again OA and OD are bisectors of
A and D respectively, then
AOD = How much?

B
O

(a) 90

(b) 60

(c) 45

(d) 30

2.
E

A and F are the mid points of DE


and CE respectively.
If DC = 8 cm, then which one of
the following is the value of AF?

3.

(a)
(c)
(i)
(ii)

4 cm
(b) 8 cm
12 cm
(d) 16 cm.
The sum of the four angles of quadrilateral is 4 right angles.
If the two adjacent sides of a rectangle are equal then the
rectangle is a square.
(iii) Every rhombus is a parallelogram.
Which one of the following is correct according to above
information?
(a) i and ii
(b) i and iii
(c) ii and iii
(d) i, ii and iii
Answer questions (4 6) in view of the following figure:
AB || DC, AD || BC and AB
B
A
E
AD, ABF = FBC in
O
quadrilateral ABCD. ADF
= FDC.
F
D

225

4.

5.

6.

How many parallelogram are there in the figure?


(a) 2
(b) 4
(c) 6
(d) 8
If AC = BD, then what would be the parallelogram?
(a) Square
(b) Rectangle
(c) Rhombus
(d) Trapezium.
(i) ADC and BCD are two adjacent angles of the
Parallelogram ABCD.
(ii) EDF and EBF are opposite angle in the parallelogram
BEDF.
(iii) The two bisectors of ADC and ABC are perpendicular to
one-another.
Which one of the following is correct in view of the above
information?
(a) i and ii
(b) i and iii
(c) ii and iii
(d) i, ii and iii

7.
C

If PA = CQ and PA || CQ in
quadrilateral PAQC. If the bisectors of
A and C are AB and CD, then
what is the name of the region P
ABCD?
(a) parallelogram
(b) rhombus
(c) rectangle
(d) square

CREATIVE QUESTIONS
1.

In above figure, ABCD is a quadrilateral.

226

(a) For what conditions the quadrilateral ABCD become a


parallelogram?
(b) If quadrilateral ABCD is a parallelogram, then show that,
ABC ACD and hence show that, ABC = ADC.
(c) If B= 90 in the parallelogram ABCD, then prove that,
ABCD is a rectangle.
2.

A
D
B

F
C

In the figure, ABC is a triangle.


(a) Prove that, BDF + DFE + FEB+EBD = 4 right angles.
(b) If D and F are middle points of AB and AC, then prove that,
DF || BC and DF =

1
BC.
2

(c) ABC is equilateral and the middle points of sides AB, AC and
BC are D, F and E, then prove that, BEFD is a rhombus.
3.
D

C
O

In figure, AB = CD and AB || CD
(a) Write the name of two triangles with base AB.
(b) Prove that, AD and BC are equal and parallel.
(c) Show that, OA = OC and OB = OD.

Chapter III

Area
3.1 Area
The measurment of a plane region is called area.
Measurement of Area
If each side of a square region is one unit, then area of the square region is one
square unit. If the length of one side of a square region is one centimetre, then the
area of the square region is one square centimetre.
3.1.1 Area of Rectangular Region
Area of rectangular region = the length of the rectangular region the breadth of
the rectangular region.
i.e. Area of the rectangular region = Product of the lengths of two adjacent sides
of the rectangular region.
The area of the rectangular region
ABCD = AB BC

3.1.2 Some Theorems Related to Areas are Included Below.


Theorem 20
If a triangular, region and a parallelogram-region stand on the same base and
lie between two parallel lines, then area of the triangular region will be half of
that of the parallelogram region.
Proposition : Let, the triangular region, ABC and the parallelogram region EBCD
stand on the same base BC and between two parallel lines BC and ED.
It is required to prove that,
1

triangular region ABC = 2 (Parallelogram region EBCD)

228

fig (a)
fig.(b)
Construction : Let us draw a line AF parallel either to EB or DC through A; the
line AF intersects either the base BC (Fig. a) or BC produced (Fig. b) at F.
Proof: AB is a diagonal of the parallelogram EBFA ;
1

region ABF = 2 (parallelogram region EBFA).

Again, AC is a diagonal of parallelogram region AFCD;


1

region AFC = 2 (

region AFCD).

In fig. (a), region ABF + region AFC


1
1
=2(
region EBFA) + 2 (
region AFCD)
1
i.e. region ABC = 2 (
region EBCD)
Again, in fig. (b), ( region ABF) ( region AFC)
1
=2 (
region EBFA
region AFCD).
1
i.e. region ABC =2 (
region EBCD)
1
Therefore, triangular region ABC = 2 (parallelogram region EBCD) (proved)

Corollary: If a triangular region and a rectangular region stand on the same


base and lie between two parallel lines then the area of the triangular region is
half of the rectangular region.

229

Area of the triangular region : A triangular region ABC stands on the base BC
and AF is its height. Let us draw a rectangular region BCDE of height equal to AF
on the base BC.
1
Then, region ABC = 2 (rectangular region BCDE)
1
1
= 2 BC CD = 2 BC AF
Remark : Hence, as a formula it can be stated :
1
Area of the triangular region = 2 base height.
Theorem 21
Areas of all the triangular regions standing on the same base and between
two parallel lines are equal to one another.

Proposition : Let, triangular regions ABC and DBC stand on the same base BC
and lie between two parallel lines BC and AD.
It is required to prove that, region ABC = region DBC.
Construction: Let us draw perpendiculars BE and CF at the points B and C of the
line BC. They intersect the line AD or AD produced at the points E and F
respectively.
Proof : According to the construction, EBCF is a rectangular region. Since
region ABC and rectangular region EBCF stand on the same base BC and lie
between two parallel lines BC and ED.
1

Therefore, region ABC = 2 (rectangular region EBCF) [ Theorem 20]


1

Similarly, region DBC = 2 (rectangular region EBCF)


region ABC = region DBC (proved).

230

Corollary : Areas of all triangular regions standing on bases equal to one another
and between the same two parallel lines are equal to one another.
Theorem 22
All triangular regions of areas equal to one another standing on the same
base and on the same side of it will lie between the same two parallel lines.

Proposition : Let, the triangular regions ABC and DBC stand on the same base
BC and lie on the same side of it and the areas of the triangular regions are equal
to each other. AE and DF are the altitudes of ABC and DBC respectively.
It is required to prove that, AD F BC.
1

Proof : region ABC = 2 (BC AE) and region DBC = 2 ( BC DF)


1

2 (BC AE) = 2 (BC DF).

Therefore, AE = DF.
Moreover, AE F DF, [Because both are perpendicular on BC]
The opposite sides of the quadrilateral AEFD, AE and DF are equal and parallel to
one another.
AD F EF, (Theorem 18)

i.e. AD F BC (proved).
Corollary : All triangular regions of areas to one another standing on equal bases
from the same straight line and lying on the same side will lie between the same
two parallel lines.

231

3.1.3 Pythagoras' Theorem

Theorem 23
In a right-angled triangle the square on the hypotenuse is equal to the
sum of the square regions on the other two sides.

Proposition : Let, ABC be a rightangled triangle. Its C = one right angle.


Hence, AB is the hypotenuse. It is required to prove that, the square region on AB
= the square region on AC + the square region on BC
i.e. AB2 = AC2 + BC2.
Construction : Let us draw square regions ABED, ACGF and BCHK on the sides
AB, AC and BC respectively. CL is drawn through C parallel either to AD or BE.
Let us suppose that it intersects AB and DE at M and L respectively. C, D and B,
F are joined.
Proof: BAD = CAF [each being one right angle]

Adding BAC to both sides, we get


BAD + BAC = CAF + BAC. CAD = BAF.

Now between CAD and BAF, CA = AF, AD = AB


and included CAD = included BAF,
CAD BAF. [Theorem 5]

Since each of ACB and ACG is a right angle.


BCG is in the same straight line.
N.M.G. -31

232

Now, since triangular region CAD and rectangular region ADLM stand on the
same base AD and lie between the parallel lines AD and CL,
hence rectangular region ADLM = 2 ( region CAD) ................ (1)
Again, since triangular region BAF and the square region ACGF stand on the
same base AF and lie between the parallel lines AF and BG,
hence square region ACGF = 2 ( region BAF) ..............(2)
Rectangular region ADLM = Square region ACGF ............... (3)
Similarly, joining C, E and A, K, it can be proved that,
Rectangular region BELM = Square region BCHK ....................(4)
Adding (3) and (4) it is obtained,
Square region ABED = Square region ACGF + Square region BCHK.
i.e. the square region on AB = the square region on AC + the square region on BC.
AB2 = AC2 + BC2 [proved]
N.B. This theorem is known as the Theorem of Pythagoras.
Alternative proof of Pythagoras Theorem:

Proposition : Let, in the triangle ABC, B = 90,


hypotenuse AC = b, AB = c and BC = a.
It is required to prove that, AC2 = AB2 + BC2 i.e. b2 = c2 + a2.
Construction : Let us produce BC upto D, so that CD = AB = c. Let us draw
perpendicular DE at D on BC produced, so that DE = BC = a.
Let us join C, E and A, E.
Proof : Between ABC and CDE, AB = CD = c, BC = DE = a
and included ABC = included CDE (each being a right angle).
Hence, ABC CDE. AC = CE = b and BAC = ECD.

233

Again, since AB BD and ED BD, therefore, AB F ED.


Therefore ABDE is a trapezium.
Moreover, ACB + BAC = ACB + ECD = one right angle.
ACE = one right angle.
Now, area of the trapezium region ABDE
= area of ( region ABC + region CDE + region AEC)
1

[Q Area of trapezium region = 2 sum of the parallel sides distance between


the parallel sides]
1

or, 2 BD ( AB + DE) = 2 ac + 2 ac + 2 b2
1
1
1
1
or, 2 (a + c) (a + c) = ac + 2 b2 or, 2 (a2 + 2ac + c2) = ac + 2 b2
1
1
1
1
1
1
or, a2 + ac + c2 = ac + b2 or, a2 + c2 = b2
2
2
2
2
2
2
or, a2 + c2 = b2. (proved)

Theorem 24
If the square-region on a side of any triangle is equal to the sum of the
squareregions on other two sides of it, then the angle included between the
last two sides is a right angle.

Proposition : Let in ABC, AB2 = AC2 + BC2.


It is required to prove that, C = one right angle.
Construction : Let us draw such a triangle so that F = one right angle, EF = BC
and DF = AC.
Proof: DE2 = EF2 + DF2 [because in DEF, F = one right angle]
= BC2 + AC2 = AB2 [supposition]
DE = AB.

234

Now between ABC and DEF, BC = EF, AC = DF and AB = DE.


ABC DEF [Theorem-10]
C = F = one right angle (proved)

EXERCISE 3
1.

O is any point inside the parallelogram ABCD. It is required to prove


1

that, region AOB + region COD = 2 (parallelogram region ABCD)


2.

Prove that, any median of a triangle divides the triangular region into two
triangular regions of areas equal to each other.

3.

In ABC, D and E are the mid-points of the sides AB and AC respectively.


1

Prove that, region CDE = 4 ( region ABC)


4.

In ABC any straight line parallel to BC intersects AB and AC at points


D and E respectively. Prove that, region DBC = region EBC
and region BDE = region CDE.

5.

D and E are the mid points of AB and AC respectively of ABC.


1

Prove that, region ADE = 4 ( region ABC) .


6.

Prove that, the diagonals of the parallelogram divide the parallelogram


region into four triangular regions equal to one another.

7.

Prove that, any square region is half of the square regions drawn on
its diagonal.

8.

In a triangle ABC, A = one right angle. D is a point on AC.


Prove that, BC2 + AD2 = BD2 + AC2.

9.

In a triangle ABC, A = one right angle. If D and E are the midpoints of


AB and AC respectively, then prove that, DE2 = CE2 + BD2.

10. In ABC, AD is perpendicular to BC and AB > AC.


Prove that, AB2 AC2 = BD2 CD2.
11. In ABC, AD is perpendicular to BC and P is any point on AD and AB > AC;
prove that, PB2 PC2 = AB2 AC2.

235

Multiple Choice Questions [Mark () on the correct answer]


1.
A

(a)

If AB = CD, AB || CD and AD =
BC, AD || BC in the quadrilateral
ABCD, which one of the
following is area of the triangle
ABD?

1
(CD BE)
2
1
(AB AD)
(c)
2

1
(DE BE)
2
1
(CD BC)
2

(b)
(d)

E
C

2.
A

AB || CD, AC BD
Which one of the following indicates
the quadrilateral ABCD?
(a) Square

(b) Parallelogram

(c) Trapezium
A

F
E

(d) Rectangle
P
B

In ABCDE polygon AE || BC, CF AE and DQ CF.


ED = 10 mm, EF = 2mm, BC = 8mm, AB = 12 mm.
Answer the questions (3 - 6) on the basis of the above informations:

236

3.

How much is square mm. area of quadrilateral ABCF?


(a) 64
(b) 96
(c) 100
(d) 144

4.

Which one of the following indicates the area of FPC?


(a) 32
(b) 48
(c) 72
(d) 80

5.

Which one of the following expresses the length of CD?


(a) 2 2

(b) 4

(c) 4 2

(d) 8

6.

Which one of the following indicates the difference of the area of


FPC and DQC?
(a) 46 square unit
(b) 48 square unit
(c) 50 square unit
(d) 52 square unit.

7.

i.

The area of a triangle and rectangle with equal bases and equal
height.

ii.

If an angle is 90 of a triangle, then the triangle would be a


right-angled.

iii. Area of rectangle = the product of the length of two adjacent


sides of the rectangle.
Which one of the following is correct on the basis of the above
informations?
(a) i and ii
(b) i and iii
(c) ii and iii
(d) i, ii and iii

237

CREATIVE QUESTIONS
A

45

N
60

In figure, ABCD is a parallelogram. The point M is mid-point of BP


and MN || BC.
(a) Find the value of BPC
(b) Show that region PBC =

1
(Parallelogram region ABCD)
2

(c) Prove that, region PBN = 2 ( region PMN).


2.

P
T
b

c
R

a
S

(a) What type of quadrilateral PQST? Explain logically in favour.


(b) Show that, PRT is a right-angled triangle.
(c) Prove that, PR2 = PQ2 + QR2

238

3.
a

C
F

H
A

In the above figure, BAD = 90


AE = BF = CG = DH = b
BE = CF = DG = AH = c
(a) Show that, ABE BCF
(b) Show that, EFGH is a square.
(c) Prove that, a2 = b2+c2.

Chapter IV

Problems
4.1 Construction of Quadrilaterals : A quadrilateral consists of four sides, four
angles and two diagonals. In case of a triangle if three sides are given, then the
definite triangle can be constructed. But if four sides of a quadrilateral are given,
then the definite quadrilateral can not be constructed.

Of two quadrilaterals ABCD and ABEF, AB = AB, BC = BE, CD = EF and DA =


FA. But if four sides and an angle of any quadrilateral are given then the definite
quadrilateral can be constructed. For drawing a quadrilateral five data independent
of one another are essential. The definite quadrilateral can be constructed if the
following five data are known :
(a) four sides and an angle;
(b) four sides and a diagonal;
(c) three sides and two diagonals;
(d) three sides and their two included angles;
(e) two sides and three angles, or
(f) parts of intercepts of two diagonals and one included angle of two diagonals.
Some times quadrilateral can also be constructed from such a given data when five
data can be obtained by logical argument.
A square can be constructed if one side is given. Here the four sides are equal to
one another and an angle is a right angle. A rectangle can be constructed if two
adjacent sides are given. Here the opposite sides are equal to each other and one
angle is a right angle. A rhombus can be constructed if a side and an angle are
given. Parallelogram can be constructed if two adjacent sides and the included
angle between them are given.
N.M.G. -32

240

4.1.1. Proof of constructions of some problems in relation to quadrilaterals


including the drawings are given.
Problem 16
The lengths of four sides and an angle of any quadrilateral are given. It is
required to construct the quadrilateral.

Let, the lengths of four sides of a quadrilateral be a, b, c, d and the included angle
between a and b be x. The quadrilateral is to be constructed.
Construction : From any line BE, let us take BC = a, EBF = x is drawn at the
point B. Let us take BA = b from BF. Taking radius equal to c and d respectively
let us draw two axes within ABC with the centres at A and C respectively.

They intersect at the point D. Let us join A, D and C, D.


Then, ABCD is the required quadrilateral.
proof : According to the construction, AB = b, BC = a, AD = c, DC = d and
ABC = x
ABCD is the required quadrilateral.

241

Problem 17
Lengths of four sides and one diagonal of any quadrilateral are given.
The quadrilateral is to be constructed.

Let, the lengths of four sides of quadrilateral be a, b, c, d and the length of its
diagonal be e, where a + b > e and c + d > e. The quadrilateral is to be constructed.

Construction : Let BD = e be taken from any straight line BE. Taking radius
equal to a and b respectively, let us draw two arcs with B and D as centres
respectively. These arcs intersect at the point A. Again, taking radius equal to d
and c respectively, let us draw two arcs with B and D as centres respectively to the
side of BD opposite to A. These two arcs intersect each other at C. A and B, A and
D, B and C, C and D are joined respectively.
Then, ABCD is the required quadrilateral.
Proof : According to construction, AB = a, AD = b, BC = d, CD = c

and diagonal BD = e.
Therefore, ABCD is the required quadrilateral.

242

Problem 18
The lengths of three sides and two included angles of any quadrilateral are
given respectively. The quadrilateral is to be constructed.

Let, the lengths of three sides of a quadrilateral be a, b, c and two angles adjacent
to the side of a and b, a and c given as x and y. The quadrilateral is to be
constructed.
Construction : Let us take BC = a from any line BE. CBF at the point B and
BCG at the piont C equal to x and y respectively are drawn.
Let us take BA = b from BF and CD = c from CG. A, D are joined.
Then, ABCD is the required quadrilateral.
Proof : According to construction, AB = b, BC = a, CD = c, ABC = x and
DCB = y,
Hence, ABCD is the required quadrilateral.
Problem 19
The length of a side of any square is given. The square is to be constructed.

Let, a be the length of a side of any square. The square is to be constructed.

243

Construction : Let us take BC = a from the line BE. BF BC at B is drawn. Let


us take BA = a from BF. Taking radius equal to a and centres at A and C
respectively two arcs are drawn within ABC. They intersect each other at the
point D. A, D and C, D are joined.
Then, ABCD is the required square.
Proof : In the quadrilateral ABCD, AB = BC = CD = DA = a
and ABC = one right angle. Hence, ABCD is a square.
Therefore, ABCD is the required square.
Problem 20
The length of a diagonal of square is given. The square is to be
constructed.

Let, a be the length of a diagonal of a square. The square is to be constructed.


Constuction : Let us take BD = a from the line BE. EBF = 45 is drawn at B.
Let us draw BDG = FBD at D. BF and DG intersect each other at A. Taking
radius equal to BA and with centres at B and D respectively, let us draw two arcs
to the opposite of A. They intersect each other at the pont C.
C, B and C, D are joined.
Then, ABCD is the required square.
Proof: In ABD, ABD + ADB = 45 + 45 = 90
BAD = 90 [ Q BAD + ABD + ADB = 180 ]
Now in the quadrilateral ABCD, AB = BC = CD = AD
and BAD = one right
angle.
The quadrilateral ABCD is a square. Its diagonal BD = a.
Hence, ABCD is the required square .

244

Problem 21
The lengths of two adjacent sides of a parallelogram and the included angle
between these two sides are given. The parallelogram is to be constructed.

Let, two adjacent sides of the parallelogram be a and b and the included angle
between them be x. The parallelogram is to be constructed.
Construction : Let us take BC = a from the line BE. EBF = x at the point B is
drawn. Let us take BA = b from BF. Taking radius equal to a, b respectively and
with the centres at the points A and C respectively, two arcs are drawn within
ABC. They intersect each other at the point D. A, D and C, D are joined.
Then, ABCD is the required parallelogram.
Proof: Let us join A and C.

Between ABC and ADC, AB = CD = b, AD = BC = a and AC is the


common side.
ABC ADC.

Therefore, B AC = DCA . But they are alternate angles.


AB F CD.

Similarly, it can be proved that, BC F AD.


Hence, ABCD is a parallelogram.
Again according to construction, ABC = x.
Therefore, ABCD is the required parallelogram.

245

Problem 22
To draw a parallelogram of which one angle is equal to a given angle and the
region bounded by it is equal to the given triangular region (area).

Let, ABC be a given triangular region and D be a given angle. A parallelogram


is to be constructed of which one angle is equal to D and the region bounded by
it is equal to region ABC.
Construction : Let us bisect BC at the point E. CEF equal to D is drawn at E
of the line segment EC. Through the point A, AG is drawn parallel to the side BC.
It intersects EF at the point F. Let us draw the line CG parallel to EF through the
point C. It intersects the line AG at the point G.
Then, ECGF is the required parallelogram.
Proof: Let us join A and E.
region ABE = region AEC [ because, BE = EC and the altitudes of both the
triangles are equal to each other.]
1

region AEC = 2 ( region ABC)

or, region ABC = 2 ( region AEC).


1

Again, region AEC = 2 (parallelogram region ECGF) [Q EC is the common


base and E C F A G ]
Parallelogram region ECGF = 2 ( region AEC)
Parallelogram region ECGF = region ABC.
Moreover, in the parallelogram ECGF, CEF = D,
The parallelogram ECGF is the required parallelogram.

246

Problem 23
To draw a triangle so that the region bounded by it is equal to a definite
quadrilateral region ( area).

Let, ABCD be a definite quadrilateral region. A triangle is to be constructed so


that the region bounded by it is equal to the quadrilateral region ABCD.
Construction : Let us join B and D.
Let us draw a line CE parallel to the line DB through the point C. It intersects the
side AB produced at E.
Let us join D, E.
Then, DAE is the required triangle.
Proof: BDC and BDE stand on the same base BD and DB CE.
region BDC = region BDE.
region BDC + region ABD = region BDE + region ABD
Or, quadrilateral region ABCD = region ADE.
ADE is the required triangle.
Problem 24
To draw a parallelogram of which one angle is equal to a definite angle and
the region bounded by it is equal to a definite quadrilateral region.

247

Let, ABCD be a definite quadrilateral region and E is a definite angle. Such a


parallelogram is to be constructed so that its one angle is equal to the given angle
E and the region bounded by it is equal to the region of ABCD.
Construction : Let us join D and B. Through the point C a line CF parallel to the
line DB is drawn. It intersects the side AB produced at the point F.
Let us find the mid-point G of AF. Let us draw GAK equal to E at the point A
of line segment AG and the line GH is drawn parallel to AK through the point G.
Let us draw a line parallel to AG through the point D. It intersects the lines AK
and GH at the points K and H respectively.
Then, AGHK is the required parallelogram.
Proof : Let us join D and F. According to construction, AGHK is a parallelogram
whose GAK = E.
Moreover, triangular region DAF = quadrilateral region ABCD
Parallelogram region AGHK = Triangular region DAF
Therefore, AGHK is the required parallelogram.

EXERCISE 4
1. Construct quadrilateral from the following given data :
a) The lengths of four sides are 3 cm., 35 cm., 28 cm., 3 cm. and one
angle is 45.
b) The lengths of four sides are 4 cm., 3 cm., 35 cm., 38 cm. and one
angle is 60
c)

The lengths of four sides are 32 cm., 35 cm., 25 cm. 28 cm. and
one diagonal is 5 cm.

d) The lengths of four sides are 32 cm., 3 cm., 35 cm., 28cm., and one diagonal
is 5 cm.
e)

The lengths of three sides are 3 cm., 35 cm., 25 cm and its two angles are
60 and 45.

f)

The lengths of three sides are 3 cm., 4 cm., 45 cm. and diagonals are 5 cm.
and 6 cm.

2.

The length of a side of a square is 4cm. Construct the square.

3.

The length of the diagonal of a square is 5cm. Construct the square.

N.M.G. -33

248

4.

The length of a side is 35cm and one angle is 75. Construct the
rhombus.

5.

The length of the adjacent sides are 3cm. and 4cm. Construct
the rectangle.

6.

The point of intersection of the diagonals of the quadrilateral, four


intercepts of two diagonals and an angle between them are given.
Construct the quadrilateral. OA = 42cm, OB = 58cm, OC = 37cm, OD =
45cm and AOB = 100

7.

The lengths of two adjacent sides are given. Construct the rectangle.

8.

The length of the diagonal and a side are given. Construct the rectangle.

9.

The length of one side and two diagonals are given. Construct the
parallelogram.

10. The length of one side and a diagonal are given. Construct the rhombus.
11. The length of two diagonals are given. Construct the rhombus.
Multiple Choice Questions [Mark () on the correct answer]
1.

2.

3.

How many independent and unique data are necessary to draw a


quadrilateral?
(a) 3
(b) 4
(c) 5
(d) 6
i. If two adjacent sides are given then rectangle can be drawn.
ii. If four angles are given then a quadrilateral can be drawn.
iii. If a side of a square is given then the square can be drawn.
Which one of the following is correct in view of the above
informations?
(a) i and ii
(b) i and iii
(c) ii and iii
(d) i, ii and iii
Which one of the following is correct for parallelogram?
(a) The opposite sides are non-parallel.
(b) If any one angle is right-angle then it is rectangle.
(c) The opposite sides are not equal.
(d) The two diagonals are equal to one-another.

249

4.

How much is the sum of the opposite angles of a square?


(b) 120
(a) 90
(c) 150
(d) 180

5.

If BCD=130 in the parallelogram ABCD, then


ABC=How much?
(b) 50
(a) 40
(c) 90
(d) 130

6.

Which one of the following is the characteristics of a rhombus?


(a) Diagonals are equal to one-another.
(b) Every angle is a right angle
(c) Two opposite angles are not equal.
(d) All sides are equal.

7.

A triangle and a parallelogram are situated on the same base and


between two parallel lines. The area of the parallelogram is
12 square cm.

Which one of the following is the area of the triangle?


(a) 6 square cm
(b) 9 square cm.
(c) 12 square cm.
(d) 24 square cm.
Answer the questions (810) on the basis of the following informations:
D

The diagonals AC and BD of the


rhombus ABCD have intersected at the
point O. Diagonal AC = 8 cm and side
AB = 5cm.
8.

9.

C
O
O
O

How much is the length of the side AO in cm?


(a) 2
(b) 3
(c) 4
(d) 5
AOB = How much?
(a) 60
(b) 90
(d) 180
(c) 120

250

10. How much is the length of the side OB in cm?


(a) 2
(b) 3
(c) 4
(d) 5

CREATIVE QUESTIONS
1.

2.

Two adjacent sides of a parallelogram are 4cm and 3cm respectively and
their included angle is 60.
(a) Express the above informations in a figure.
(b) Draw the parallelogram with the description of drawing.
(c) Draw a square with a diagonal equal to the largest diagonal of
the parallelogram mentioned above. Give the description of
drawing.
The length of four sides of a quadrilateral are 4 cm, 5 cm, 3.5 cm,
3.8 cm and one angle is 75.
(a) Express the given informations in figure.
(b) Draw the quadrilateral giving description of drawings.
(c) Draw a triangle of area equal to the area of the above
quadrilateral and verify it logically.

Answer
Arithmetic
EXERCISE - 1

1 (i) 0.5
(ii) 0.75
(ii) 1.6
(vi) 2.35 (vii) 2.28125
3
1
1
2. (i) 10
(ii) 4
(iii) 1 4
27
1
1
(vi) 2 40 (vii) 3 40 (viii) 10 200
3. (i) 5.94
(ii) 12.443 (iii) 25.8
(vi) 60.6347 (vii) 7.9667
4. (i) 3.212 (ii) 1.7799 (iii) 3.0845
(vi) 7.8514
5. (i) 14.2129
(ii) 0.0147
6. (i) 25.92 (ii) 81
(iii) 8010
(vi) 148
(vii) 40.544 (viii) 0.416
(xi) 0.735012
(xii) 0.125
(xv) 0.00016
7. (i) 2.45
(ii) 17.3
(iii) 10.8
(vi) 0.002 (vii) 0.12 (viii) 6.4
8. (i) 30.132 (ii) 3.0901 (iii) 0.015
.
.
.
9. (a) 0.16
(b) 0. 6
(c) 0. 1
.
.
.
(f) 1.83
(g) 3. 2
(h) 4.916
.
..
(k) 0.94
(l) 0.3267
2
35
152
10 (a) 9
(b) 99
(c) 333
893
16
241
(g) 2 990 (h) 8 999 (i) 4 495

(iv) 1.1875
(viii) 0.95
47
(iv) 3 100
451
(ix) 12 2000
(iv) 505.51

(v) 0.7
(ix) 3.87
1
(v) 2 8

(iv) 5.321517

(v) 0.000365

(iii) 6.543
(iv) 0.64
(ix) 0.00015
(xiii) 1.920919

(iv) 1.9473
(v) 4.16
(x) 435.52
(xiv) 0.00004

(iv) 1.22
(ix) 2250
(iv) 40.7
.
(d) 0. 5
.
(i) 0.06

(v) 0.044843

2
(d) 15
1151
(j) 6 4995

(v) 4.212101

..
(e) 0. 63
.
(j) 3.53
19
(e) 1 45

71
(f) 3 90
611
(k) 4 900

252

..
..
..
..
.
.
11. (a) 1. 33, 3. 32
(b) 2.333, 5.235
(c) 7.266, 4.237
.
.
.
.
.
.
(d) 5. 777777, 8. 343434, 6. 245245
.
.
.
.
.
.
..
..
..
(e) 3.23333333 , 9.23484848, 1.25765765 (f) 12.3200, 2.1999, 4.3256
.
.
.
.
.
12. (a) 0. 5
(b) 1. 6
(c) 0.365 (d) 0.589
(e) 17.583
.
.
.
.
.
.
.
(f) 17.1179
(g) 8.48588679
(h) 0.94937300 (i) 8.30120484
.
. .
.
.
.
.
..
13. (a) 1.31 (b) 1.6 6 5 (c) 8. 759668
(d) 3.678 85976
(e) 2.654
. .
. .
. .
. .
..
(g) 6.11062
(h) 1.92631
(i) 4.84163
(j) 3. 3892
(f) 3.1334
.
.
(k) 3.88830739
.
.
.
14. (a) 0.2
(b) 3.02
(c) 2
(d) 28.96
(e) 0.06
. .
. .
..
(f) 1.25 (g) 0.2074 (h) 12.18 5 (i) 0.112
.
.
.
15. (a) 0. 6 (b) 0.5
(c) 55
(d) 0.2
(e) 5. 21951
..
.
.
.
.
(f) 26. 36 (g) 0.28
(h) 0. 4
(i) 5. 7
(j) 7. 3
..
.
(k) 0. 12
(l) 4. 8
16. (a) 3.4641, 3.464

(b) 1.1180, 1.118

(c) 0.7453, 0.745

(d) 0.5025, 0.503

(e) 1.1595, 1.160

(f) 2.6526, 2.653

(g) 3.7875, 3.788

(h) 2.6457, 2.646

(i) 1.8539, 1.854

(b) Rational (c) Rational

(d) Irrational

(e) Rational

(f) Irrational (g) Irrational

(h) Rational

(i) Rational

(j) Rational (k) Rational

(l) Rational

(j) 2.2650, 2.265


17. (a) Rational

18. (a) 9

(b) 71.96

(c) 5

(d) 8

253

EXERCISE - 2.1

1. (a)37.5%
(b) 70 % (c) 224%
(d) 468.75%
.
(f) 258. 3 %
(g) 228.125% (h) 6%
(j) 467%
(k) 854.9%
(l) 0.03%
(n) 950%
(r) 52%
1
5
2. (a) 4
(b) 4
5
1
(g) 16
(f) 6
3. (a) 0.12 (b) 0.37
(f) 1.15625
4. (a) 31
(b) 60
(f) 240 (g) 12 pieces
(k) 8 km
5. (a) 20%

(b) 50%

(b) 320

(i) 56.7%
(m) 704.5%
.
(q) 58. 3 %

(o) 346%
(s) 78.125%
743
(c) 600
3
(h) 8
(c) 0.125
(g) 0.22625
(c) 10
(h) Tk.37.50

(p) 90%
(t) 52%
77
(d) 300
2
(i) 3
(d) 0.0825
(h) 0.1409375
(d) 67
(i) 2 kg 250 g

(e) 1.23125
(i) 0.421875
(e) 66
(j) 480 person

2
(c) 66 3 %

2
(d) 166 3 %

1
(e) 13 3 %

2
1
2
(f) 41 3 % (g) 62 2 % (h) 16 3 %
6. (a) 50

.
(e) 83. 3 %

(c) 120

297
(e) 3 700

(i) 4%

(d) 240
(e) Taka 125
1
(f) 80 km (g) 240 kg (h) Taka 233 3
(i) 300 kg
7. (a) 1 t 4 (b) 8 t 25
(c) 12 t 25 (d) 3 t 5
(e) 5 t 4
(f) 7 t 2
8. Taka 14400
9. 2000 person
10. 70%
11. Taka 102
1
12. 75%
13. Tk. 482 14. 12 2 % 15. 20%
16. Taka 4000
17. 20000 person
18. Taka 1200000 19. 200 person
20. 400 female students and at present 480 male students. 21. Taka 1500
22. 100000 person
23. 10,00,000 person
1
24. Present cost Tk. 60 and previous cost Tk. 75 25. Tk. 30
26. 9 11 %

254

1
27. 33 3 %

28. 20% 29. 23%

32. 58320 person

30. 200 person

31. 400 person

33. 1250 pieces


EXERCISE - 2.2

1. Tk. 273
6. Tk. 750
11. 40%
16. 4 years
21. 4%

2. Tk. 119
7. Tk. 730
12. Tk. 475
17. 3 years
22. 6 years

3. Tk. 30
8. Tk. 9000
13. 30%
18. Tk. 775
23. Tk. 225

4. Tk. 2502
5. Tk. 1270.50
9. 14%
10. 6%
14. 6 years
15. 10 years
19. Tk. 500
20. Tk. 1600
24. 10 years
25. Tk. 365000
1
1
26. Capital Tk. 400 and rate of interest 7 2 % 27. 8 years
28. Tk. 533 3
29. 40 years 30. Tk 1200 and 7%
31. 6 years
32. Tk. 1230
3
3
33. 4%
34. 20 portion
35. 6 4 %
EXERCISE - 2.3

1. Tk. 25 Loss
5. 25 Pieces
9. Tk. 40
13. 50% Profit
17. 12 Pieces
21. 25% Loss
25. Tk. 225
pant Tk. 350

2. Tk. 12.50 Profit


6. 60 kg
10. Tk. 25 Loss
14. Tk. 16
18. 4% Profit
22. Tk. 2.25
1
26. 33 3 %
29. 12% profit

31. There will be no loss or Profit


34.Tk. 33

35. Tk. 3.36

38. Tk. 561000 39. Tk. 181.92

3. Tk. 10 Profit
7. Tk. 20 Loss
11. 20% Loss
15. 5% Profit
19. Tk. 792
23. 50% Profit

4. Tk. 10 Profit
8. Tk. 25 Profit
12. 8% Profit
16. 25% Profit
20. Tk. 10
24. Tk. 396

27. Tk. 300

28. Shirt Tk. 175 and

30. Horse Tk. 6000 and cow Tk. 4000


16
32. 4 21 % Loss 33. Tk. 22
1
36. Tk. 12.50
37. 33 3 %

255

EXERCISE - 3

1. 53650 Person

2. 30 Person will be reduced

4. Tk. 80

6. 420 pieces

5. Tk. 2.50

3. 132 pieces

7. 10000 metre 8. 6400 times

9. 625 metre 10. 3000 metre


11. 100 metre towards the same direction
and 3100 metre towards the opposite direction
12. 54.17 km(app) 13. 280kg
14. 26.67 metric ton (app) 15. 145 kg 952 gm 450mg 16. 1 quintal 26 kg 500gm
17. 821.92 metric ton (app)

18. 549 kg rice and 172.5 kg salt

19. 200 days 20. 0.64 litre (app)

21. 0.07 litre (app) 22. 16 litre

23. 320000 sq.cm.

24. 25 metre

26. 636sq.m. 27. 402.31 metre(app) 28. 60 metre

29. 155 sq.m

30. 520.8 sq.m

31. 3264 sq.m

34. 2717.44 gm

35. 673.5470 cu.cm 36. 1240 cu. metre

37. 450 kg

32. 24 metre

25. 208 sq.m


33. 2.5 metre

38. 60000 litre and 60000 kg


EXERCISE - 4.1

1. (a) Tk. 49.40


2. (a) 75.02 (b) 0.02
6. 44.5 kg

(b) Class interval 5, Tk. 49.13


3. Tk. 821.331
4. 45.2, 44.66

5. 23.31

.
7. Tk. 42.36 hundred 8. Tk. 2230.63
EXERCISE 4.2

1. Mean 22.21, Median 23, Mode 19


2. Median 45, Mode 38 and 55; Median 46 and Mode 41 from class interval
3. Mean Tk. 74.64, Median Tk. 74 and Mode Tk. 69.
4. Mean Tk. 150.43, Median Tk. 150, Mode Tk. 140 and Tk. 156
5. Mean Tk. 110.93, Median Tk. 112.60 and Mode Tk117.15
6. Mean Tk. 65.18, Median Tk. 64.55 and Mode Tk. 63.68
7. Mean 66.1, Median 66.83 and Mode 66.714
N.M.G. -34

256

8. Mean 11.44 years, Median 11.97 years and Mode 12.2 years.
9. Mean marks 65.125, Median 64 and the marks obtained by the highest number
of students is 64.
10. Mean Tk. 66.65, Median Tk. 65 and the daily wages of the maximum number
of labourers is Tk. 63.
11. Mean 22.25 years, Median 17.92 years and Mode 5 years.
12. Population growth rate 22%, Annual population growth rate 2.2%, Growth rate
of male population 21.8%, Growth rate of female population 22.2%, Mean of
population 96717478.5, Mean of male population 49823770.5 and Mean of female
population 46893708.

ALGEBRA
EXRCISE - 1.1

1. (i) 25a2+70ab+49b2

(ii) 49x2-126xy+81y2

(iv) 36x4y2-60x3y3+25x2y4 (v) 16x6+24x3y4+9y8

(iii) 16a2b2+40ab2c+25b2c2
(vi) x4-6x2+9

(vii) x6+2x4y+x2y2

(viii) 121a2-264ab+144b2 (ix) a2x2-2abxy+b2y2

(x) x2+2xy+y2

(xi) a2-2ab+b2

(xii) 2y2z2+2abcxyz+a2b2c2

(xiii) a4x6-2a2b2x3y4+b4y8

(xiv) a2+b2+c2-2ab+2ac-2bc

(xv) x4+y4+z4-2x2y2-2x2z2+2y2z2

(xvi) x4y2z2-2x3y3z2+y4z2x2

(xvii) x2y2+y2z2+z2x2+2xy2z-2x2yz-2xyz2
(xx) 356409
(xxi) 998001

(xviii) 90601
(xxii) 1004004

(xxiii) 25a2+36b2+49c2-60ab-70ac+84bc
(xxiv) 49a4+64b4+25c4+112a2b2-70a2c2-80b2c2
(xxv) a2+b2+c2+d2-2ab-2ac-2ad+2bc+ 2bd+2cd
2. (i) 9a2
(vi) 16

(ii) 36x4

(iii) 324

(iv) 64b2

(v) 4z4

6. 11

7. 194

13. 29

14. 178, 40

(vii) 10,00,00,000

3. 576

4. 196

5. 18225

10. 4,34

11. 289,169 12. 36, 90

(xix) 367236

257

15. 482, 240 16. (i) x2-y2 (ii) 25x2-49y2 (iii) 49a2-121 (iv) a2-b2+2bc-c2
(v) 25a2+4b2+20ab-9c2

(vi) a2x2-b2y2+2bcyz-c2z2 (vii) a8-b8

(viii) x2-4x-140

(ix) 9a2-45a+50

(x) 36x2+24x-221

(xi)a2x2+2abxy+b2y2+8ax+8by+15
17.(i) (6a+4b)2-(a+2b)2

(ii) (5x)2-(13)2

(iii) (5x)2-(2x-5y)2

(iv) (8b-a)2-(b+7a)2
EXERCISE - 1.2

1. (i) 27x3+108x2y+144xy2+64y3

(ii) x9+3x6y2+3x3y4+y6

(iii) a6b3+3a4b2c2d+3a2bc4d2+c6d3

(iv) a3b3+3a2b3c+3ab3c2+b3c3

(v) 512x6+2112x4y3+2904x2y6+1331y9

(vi) 343m3 + 735m2n+525mn2+125n3

(vii) x3+y3+z3+3x2y+3xy2+3x2z+3y2z+3xz2+3yz2+6xyz (viii) x9+3x6y3+3x3y6+y9


(ix) 8x3-60x2y+150xy2-125y3

(x) p6-3p4q2+3p2q4-q6

(xi) 1331a3-4356a2b+4752ab2-1728b3

(xii) x9+6x6+12x3+8

(xiii) x18-24x12+192x6-512
(xiv) 8x3-27y3-z3-36x2y+54xy2-12x2z-27y2z+6xz2-9yz2+36xyz
(xv) x6-y6+z6-3x4y2+x2y4+3x4z2+3y4z2+3x2z4-3y2z4-6x2y2z2
(xvi) a6b3-3a4b5c+3a2b7c2-b9c3

(xvii) x3y3-6x2y3z+12xy3z2-8y3z3

(xviii) a6b6-3a4b4c2d2+3a2b2c4d4-c6d6

(xix) x9-6x6y3+12x3y6-8y9

(xx) 343x6-1323x4y2+1701x2y4-729y6
2. (i) 216a3

(ii) 64y3

3. 370

4. 793

5. 1900

10. 722

11. 125

9. 1
6

16. (i) a +b

(ii) x +8

(vi) 512x3-27y3

(iii) a3-3a2b+3ab2-b3 (iv) 8x3 (v) 8x3

6. 33614

8. 0

12. 512
3

(vi) 8x3

15. 140
3

(iii) 8a +27b (iv) 343a +64b3

(v) 64a3-27b3

(vii) a3x3-b3y3 (viii) x6+a6 (ix)x6-a6

(x) 64a6-1

(xi) 15625a6-729b6
17. (i) (a+2) (a2-2a+4)
(ii) 8(x+3y) (x2-3xy+9y2) (iii) (2x+7) (4x2-14x+49)
(iv) b3(3a+4c) (9a2-12ac+16c2)
(v) a (2a+3b) (4a2-6ab+9b2)

258

(vi) (4b-5) (16b2+20b+25) (vii) (3a-2) (9a2+6a+4)


(viii) 3(2x-3y) (4x2+6xy+9y2)

(ix) 7(2x-3y) (4x2+6xy+9y2)

(x) (9a-4bc2) (81a2+36abc2+16b2c4)


EXERCISE - 1.3

1. (2x+y) (2x-y)

2. (x+12y) (x-12y)

3. 3x(1+5x) (1-5x)

4. a(x2+4x+8) (x2-4x+8)

5. (2a2+6a+9) (2a2-6a+9)

6. (x+y-1) (x-y+1)

7. (x2+2x-1) (x2-2x-1)

8. (x2+x+1) (x2-x+1)

9. (a-1) (a-2b+1)

10. (a2+ab+b2) (a2-ab+b2)

11. 8(2x-y) (4x2+2xy+y2)

12. (x-y+z) (x2-2xy+y2-xz+yz+z2)

13. (a2+b2) (a4-a2b2+b4)

14. (a+b) (a-b) (a2-ab+b2) (a2+ab+b2)


15. (x+9) (x-8)

16. (x+14) (x+4)

17. (x-15) (x+7)

18. (x-5) (x-3)

19. (x-26) (x-25)

20. (x+7) (5-x)

21. (x+10) (x+4)

22. (a+4b) (a+3b)

23. (x+15) (x-8)

24. (x+14) (x-3)

25. (x+8y) (x-5y)

26. (p+10q) (p-8q)

27. (x2-x+8) (x2-x-5)

28. (a-2) (a+2) (a+5) (a+9)

29. (x-4) (x+2) (x+4) (x+10)

30. (3x-5) (2x+3)

31. (x+4a-2) (x-3a+2)

32. (x+a+b) (x+2a+3b)

33. (x+14) (x+4)

34. (x+4) (3x-1)

35. (2x+3) (4x-5)

36. (x-6) (3x+2)

37. (x-7) (2x+5)

38. (a+5b) (2a-3b)

39. (3a-4b) (3a+4b) (a2+2b2)

40. (x+y-2) (2x+2y+1)

41. (3x-4y) (5x+3y)

42. (x-2y) (2x-y)

43. (2p+3q) (5p-2q)

44. (x+6) (2x-7)

45. (x+a) (ax+1)

46. 5a2b2c2

47. 18a2c2

48. 6

49. 8x2y2z3

50. x+y

53. x-y

54. x+2

55. 2 (x+y)

56. ab(a2+ab+b2)

51. x-3

58. 36x2y2z3 59. 72a3b2c3d3

60. 60x4y4z2

63. (a-b)2 (a+b)3 (a2-ab+b2)2

64. (x2-1) (x+2) 65. (a6 -1)

66. x(x+2)2 (x+3)

67. a2b2(a6-b6)

52. x-2
57. x(x+2)

61. 120a3b3c262. 30a2b3c3


68. (2x-1) (3x+1) (x+2)

259

EXERCISE - 2.1

3y
1. (i) 4x2z2

7c4
(ii) 6ab2

2(a+b)
(a-b)
(vi) 3a(a-b) (vii) (a+b)2

5a3
(iii) 4b3x2

11q2r3
(iv) 16

x
(v) y2

(a-3)
(viii) (a+3)

(x-1)
(ix) (x+9)

(2x+3)
(x) (3x+4)

(a-b-c)
(2a-b)
(2x2-1)
(xi) (3x2+1) (xii) (a+b-c) (xiii) (a2-1)
21b3
15c3
35a3
2. (i) 420abc , 420abc , 420abc
a2(a+b) ab(a-b) c(a-b)
(iii) a(a2-b2) , a(a2-b2) , a(a2-b2)

15a2b2d3 10a2c2d2 6b2c2


(ii) 60abcd5 , 60abcd5 , 60abcd5
c(a-b) a(b-c) b(c-a)
(iv) abc , abc , abc

(a+b) (a3+b3)
(a-b)3
a(a b) (a2 ab + b2)
(v) (a-b)2 (a3+b3) , (a-b)2 (a3+b3) ,
(a-b)2 (a3+b3)
(vi)

a(a2+ab+b2) b(a-b)
c
,
3 3
3 3 ,
3 3
(a -b )
(a -b ) (a -b )

2(x+3)2
3(x+1) (x+3)
4(x-2) (x+1)
(vii) (x-2) (x+1) (x+3)2, (x-2) (x+1) (x+3)2, (x-2) (x+1) (x+3)2
(a-b)2(c-a)
(b-c)2(a-b)
(b-c) (c-a)2
(viii) (a-b) (b-c) (c-a), (a-b) (b-c) (c-a), (a-b) (b-c) (c-a)
x2+y2+z2
a2+2ab-b2
2(4x2+9y2)
x2+y2
(ii) xyz
(iii) a2-b2
(iv) 4x2-9y2
3. (i) xy
3a2+b2
3
2x4-1
(v) (a-b) (a+b)3
(vi) (x-1) (x-3)
(vii) x6-1
3(x-1)
(viii) (x-2) (x-3) (x+2)
-2b
ab
6xy
6x
4x
(iv) x2-9y2 (v) (x3-8)
4. (i) x2-16 (ii) a(a2-b2) (iii) a3+b3
2x
(vi) (x4+x2+1)
3a2b-3bc2+a2c-ac2+ab2-b2c
-2c
2(a-c)
(ii)
(iii)
5. (i) ac
(a+b) (b+c) (c+a)
(b-c) (c-a)
2ab
(iv) a2-b2

(v) 0

16x3y
(vi) (x4-16y4)

6ab2
(vii) (a2-b2) (4a2-b2)

260

2x
12a4
(viii) (x4-1) (ix) (a6-64)

(x) 0

(a-2b)
(xiii) (a2+b2-c2-2ab)

12y
(xiv) 4x2-9y2

64x4
(xi) (x8-256)

2
(xii) (c-b)

EXERCISE - 2.2
4 3

5 3

2a b
1. 45c2d4

8y z
2. 9x2

(1-b)
6. x

7. 1

3. 1

x
4. (x+3)

(a+b) (a2+b2)
5. (a3-b3) (a2-ab+b2)

8. 1

9. a+b

(1-b)
10. (x+y)

a(a2-ab+b2)
11. (a-b)2

(b2+ac) (bd+ac)
12.
abcd

x
15. (1+x) (1-x3)

6x2
16. x2-y2

1
13.- x2

2x
14. (x+y+z)

EXERCISE - 2.3

32a3c
1. 27b3
x-7
6. x-5
10. 1
a(2a-3b)
2a-b
3a
20. 2b

15.

5ac3
2. 9b

x2y2z2
3. 9a2b2c2

3b
5. 2a

7. (a-b)2

x2-8x+12
8. x2-10x+21

25a
4. a+b

x
11. x-1

12. 1

1
13. a2+b2

14. a+b

16. x2-y2

1
17. a

18. x-3

19. 1

(x+5) (x2+x-56)
9. (x+12) (x2-36)

21. 2a(a+b)2

1. - 4

2. - 8

5
6. -55 7

4
7. 2 19

EXERCISE - 3.1
1
3. 2
4. 3 5
79
11
8. 18
9. 26

5. - 20
1
10. - 2 24

261

-24
11. 55
16. - 25

-242
12. 71
2
17. 5

21. a + b + c

22. a + b + c

-11
25. 7

5
26. 3

-295
13. 96
23
18. - 31
23.
1
27. 5

14. 73

a+b
15. 2

19. 15

20. ab + ac + bc

ab+bc+ca
3
7
28. 2

3
24. 2
-5
29. 6

EXERCISE - 3.2

1. 40 and 60 2.19 and 42


3. Present age of father is 60 years and the present age of son is 20 years.
3
4. 15 and 60 5. 4
6. Chaity's portion Tk. 48, Pio's Tk. 32 and Rafat's portion Tk. 24.
7. 240 pieces 8. 30 male and 20 female. 9. 180 person
10. 12 metre
11. The ratio of prices of sugar at Tk 40 and Tk. 30 is 1 t 4.
12. The number of 25 paisa coin is 20 pieces and 50 paisa coin is 80 pieces
13. 1km. away from Rampura
14. 45 Kg. with one person and 35 kg with another person. Each of both can carry
20 kg without any cost.
EXERCISE - 4.1

1. (2, 1)
2. (3, 2)
6. (6, -5)
7. (3, 2)
2
(b +ac) (ab-c)
11.{ (a2+ab) , (a2+ab) }
1
15. (-3, 0)
16. ( 2 , -3)

3. (2, 2)
8. (3, 10)

4. (2, 3)
9. (2, 3)

5.(2, 3)
10. (1, 2)

12. (2, 3)

13. (5, 2)

14. (2, 1)

17. (3, 2)

18. (7, 4)

19.(8, 12)

20. (a+b, b-a) 21. (6, -8)

22. (2, 6)

23. (6,2)

ab ab
24. ( a+b , a+b )

262

EXERCISE - 4.2

1. 45,35

2. 60,15 3. 7,9
4. 50 years and 20 years
3
3
5. 45 year and 25 year 6. 5
7. 8
8. 53
9.63
10. 62 and 34 mangoes respectively; Total number of mangoes 96.
11. 50 and 20 metre. 12. 6 km and 3km 13. Men by 12 days or boys by 24 days.
14. Male by 15 days or boys by 60 days.
15. The price of cow is Tk. 4500 and the price of goat is Tk. 1500.
16. Speed of rowing is 4 km. and the speed of current is 3 km.
17. By 1st tube in 10 minutes and by 2nd tube in 15 minutes.
EXERCISE - 5.1

1. 1st Quadrant, 2nd Quadrant, x-axis to the right, 3rd Quadrant, 4th Quadrant,
y- axis above, Origin, y-axis below, x-axis to the left, 1st Quadrant and y-axis
above.
6. (0,1)
7. (-1,-1)
EXERCISE - 5.2
2. (i). (4,1)
(vi). (-3,2)
3. (i) 2
(vi) 2

(ii). (3,2)
(vii). (4,3)
(ii) -2
(vii) -1

(iii). (2,1)
(viii). (2,0)
(iii) 2
(viii) 12

(iv). (2,3)
(ix). (1,-1)
(iv) 1

THE END

(v). (2,0)
(v) 0

You might also like